Добавил:
Upload Опубликованный материал нарушает ваши авторские права? Сообщите нам.
Вуз: Предмет: Файл:
крок 1_ЗЛП_2009.DOC
Скачиваний:
220
Добавлен:
01.05.2019
Размер:
10.29 Mб
Скачать

E Паралічі зліва

30. В експерименті у тварини зруйнували середній мозок в ділянці чорної субстанції. Синтез якого медіатора припинився?

A * Дофаміну

B Ацетилхоліну

C Серотоніну

D Гістаміну

E Норадреналіну

31. Що лежить в основі механізму формування патологічної детермінанти?

A.*Формування генератора патологічно посиленого збудження.

B. Феномен випадіння

C. Позамежове гальмування

D. Перезбудження

E. Парабіоз

ПАТОЛОГІЧНА АНАТОМІЯ

Патологія клітини. Дистрофії. Некроз. Смерть.

1. При черевному тифі некротизовані пейєрові бляшки тонкої кишки, забарвлюються у жовто-коричневий колір. Який пігмент просякає некротизовану тканину?

A. * Білірубін

B. Гемоглобін

C. Ліпофусцин

D. Індол

E. Меланін

2. Хвора 70 років прооперована з приводу “гострого живота”. Під час операції виявлено близько 80 см клубової кишки чорного кольору, очеревина тьмяна, просвіт верхньої брижової артерії обтурований тромбом. Який процес розвинувся в кишці?

A. *Гангрена

B. Пролежень

C. Білий інфаркт

D. Білий інфаркт із геморагічним вінчиком

E. Коагуляційний некроз

3. На розтині трупа на підставі характерних макроскопічних змін діагностована паренхіматозна жирова дистрофія міокарда серця. Яка образна назва серця при цій дистрофії?

A. *Тигрове серце

B. Волове серце

C. Волосате серце

D. Панцирне серце

E. Легеневе серце

4. У хворого цукровим діабетом з'явився різкий біль у правій стопі. При огляді великий палець стопи чорного кольору, тканини стопи набряклі, осередки відшарування епідермісу, виділення з неприємним запахом. Яка клініко-морфологічна форма некрозу розвилася в хворого?

A. *Гангрена волога

B. Пролежень

C. Секвестр

D. Гангрена суха

E. Інфаркт

5. У хворого з вадою мітрального клапану з'явився кашель, мокротиння ржавого кольору. Який пігмент обумовив такий колір мокротиння?

A. *Гемосидерин

B. Меланін

C. Гемоглобін

D. Гемомеланін

E. Сірчасте залізо

6. У хворого виразковою хворобою шлунка з кровотечею при ендоскопії в шлунку знайдена рідина кольору кавової гущавини. Який пігмент обумовив такий колір вмісту шлунка?

A. *Солянокислий гематин

B. Гемосидерин

C. Білірубін

D. Феритин

E. Порфірин

7. Жінка віком 68 років хворіє на хронічний фіброзно-кавернозний туберкульоз легень протягом 20 років. Поступив у нефрологічне відділення з явищами уремії. Прижиттєва проба на наявність у нирках амілоїду виявилася позитивною. Про яку форму амілоїдозу йде мова в даному випадку?

A. *вторинний системний

B. первинний системний

C. обмежений (місцевий)

D. сімейний вроджений

E. сенильний (старечий)

8. На розтині трупа чоловіка віком 57 років, який помер від висипного тифу, виявлено, що м’язи передньої черевної стінки і стегон щільні, білуваато-жовтого кольору, нагадують стеаринову свічу. Проявом якого патологічного процесу є виявлені зміни?

A. *восковидного некрозу

B. апоптозу

C. фібриноїдного некрозу

D. колікваційного некрозу

E. казеозного некрозу

9. У померлого 58 років на розтині мітральний клапан деформований, потовщаний, змикається не до кінця. Мікроскопічно: вогнища колагенових волоконець еозінофільні, дають плюсову реакцію на фібрін. Найвірогідніше це:

A. *фібріноїдне набухання

B. фібрінозне запалення

C. мукоїдне набухання

D. гіаліноз

E. амілоїдоз

10. При розтині чоловіка 49 років, який поступив у стаціонар з картиною гепатотропної інтоксикації і раптово помер, печінка збільшена, дрябла, жовто-коричневого кольору; на поверхні розрізу печінки і лезі ножа помітні краплини жиру. Мікроскопічно: гепатоцити периферії класичних печінкових часточок вміщують масу дрібних крапель, які виповнюють цитоплазму і відсовують ядро на периферію. Який процес найімовірніше має місце в печінці ?

A. *Жирова дистрофія печінки

B. Цереброзидліпідоз (хвороба Гоше)

C. Сфінгомієлінліпідоз (хвороба Німанна-Піка)

D. Гангліозидліпідоз (хвороба Тея-Сакса)

E. Генералізований гангліозидоз(хвороба Нормана-Ландінга)

11. У хворого на гостру виразкову хворобу шлунка, що ускладнилась шлунковою кровотечею, блювотні маси забарвлені в темно-коричневий колір, що описується як блювота “кавовою гущею”. Наявність якого пігменту в блювотних масах визначає таке їх забарвлення ?

A. *Солянокислого гематину

B. Гемоглобіну

C. Білірубіну

D. Гемомеланіну

E. Сульфіду заліза

12.У померлого при розтині знайдено тромбоз лівої середньої мозкової артерії і велике вогнище сірого пом’якшення тканин лівої півкулі мозку. Який патологічний процес розвився в головному мозку?

A. * Ішемичний інфаркт

B. Коагуляційний некроз

C. Абсцесс

D. Волога гангрена

E. Секвестр

13. У лежачого хворого з недостатністю кровообігу, нерухомого після перенесеного інсульту, шкіра і м’які тканини над крижами почорніли і набрякли, після відторгнення епідермісу в чорних тканинах відкрились виразки. Який процес розвився у хворого?

A. *Пролежні

B. Суха гангрена

C. Флегмона

D. Інфаркт

E. Абсцес

14.У померлого від хронічної серцево-судинної недостатності на розтині виявлене “тигрове серце”. З боку ендокарду помітне жовтувато-біле покреслення, міокард тьмяний, глинисто-жовтий. Який процес зумовив дану патологію?

A. * Жирова паренхіматозна дистрофія.

B. Вуглеводна дистрофія

C. Гіаліново-краплинна дистрофія.

D. Жирова судинно-стромальна дистрофія.

E. Амілоїдоз

15. Чоловік, 70 років, зі скаргами на біль у дрібних суглобах рук і ніг. Суглоби деформовані, болючі. Виявлений підвищений рівень солей сечової кислоти в крові та сечі. Про порушений обмін яких речовин йде мова?

A. *Нуклеопротеїдів.

B. Кальція.

C. Хромопротеідів.

D. Ліпопротеїдів.

E. Калію.

16. При мікроскопічному дослідженні тканини печінки було виявлено, що деякі клітини розпалися на невеликі фрагменти з окремими органелами та залишками ядра, оточені мембраною. Запальна реакція відсутня. Виберіть патологічний процес, для якого характерні описані зміни.

A. *Апоптоз

B. Некроз

C. Каріорексис

D. Плазмоліз

E. Плазморексис

17.У хворого, який тривалий час страждав на переміжну кульгавість, тканини пальців стопи сухі, чорного колоьру, нагадують мумію. На невеликій відстані від почорнілої ділянки розташована двоколірна лінія (червоний колір прилягає до практично незмінених тканин, а біло-жовтий колір – до змінених тканин). Який вид некрозу у даного хворого?

A. *Гангрена.

B. Інфаркт.

C. Секвестр.

D. Пролежень.

E. Мацерація.

18.У хворої 77 років із защемленою паховою килою при лапаротомії стінка кишки ціанотичного кольору, роздута, набрякла, вкрита нитками фібрину, перистальтика не виявляється. Який патологічний процес розвинувся у стінці кишки внаслідок пристінкового защемлення кили?

A. * Волога гангрена.

B. Суха гангрена.

C. Коагуляційний некроз

D. Колікваційний некроз

E. Пролежень

19.У дитини діагностовано в клініці дифтерія зіву. Померла від гострої серцевої недостатності. На розтині виявлено, що порожнини серця розширені в поперечнику. М’яз серця тьмяний, в’ялий, на розрізі строкатий, з жовтуватими ділянками. У цитоплазмі деяких кардіоміоцитів зі збереженою цитоплазмою виявляються дрібні вакуолі. На заморожених зрізах вакуолі забарвлюються суданом 3 у помаранчовий колір. Який вид дистрофії виявлений у кардіоміоцитах?

A. *Жирова

B. Вуглеводна

C. Балонна

D. Гіаліново-крапельна

E. Гідропічна

20. Чоловік тривалий час хворів гемобластозом. На розтині виявлено, що кістковий мозок, селезінка, печінка, лімфатичні вузли коричневого кольору. Проведено гістохімічну реакцію Перлса. Встановлено, що ретикулярні, ендотеліальні і гістіоцитарні елементи цих органів містять гранули синього кольору. Який пігмент виявлено при застосуванні зазначеної методики?

A. *Гемосидерин

B. Білірубін

C. Гематоїдин

D. Гематоїдин

E. Гематопорфірин

21. Чоловік 38 років раптово помер, на розтині у задній стінці лівого шлуночку серця знайдено інфаркт міокарду. Які найбільш вірогідні зміни у будові міокардіоцитів можна побачити у вогнищі інфаркту мікроскопійно?

A. *Каріолізис

B. Жирова дистрофія

C. Вуглеводна дистрофія

D. Обвапнення

E. Білкова дистрофія

22. Хворий 66 років за 10 років до смерті переніс перітоніт. На розтині капсула печінки та селезінки місцями різко потовщана, ущільнена, напівпрозора. Найвірогідніше це:

A. *гіаліноз

B. некроз

C. мукоїдне набухання

D. фібріноїдне набуханняя

E. амілоїдоз

23. У харкотинні хворого з мітральною вадою серця виявлені клітини, які вміщують бурий пігмент. Реакція Перлса позитивна. Який це пігмент ?

A. *Гемосидерин

B. Гематоідин

C. Меланін

D. Порфірин

E. Білірубін

24. У чоловіка 62 років, що помер при наростаючих явищах серцевої недостатності, на розтині знайдено збільшене в об’ємі серце. Серце дряблої консистенції, камери розтягнуті, міокард на розрізі тьмяний, глинисто-жовтий. З боку ендокарда видно жовто-білу посмугованість, яка особливо виражена в сосочкових м’язах. Який патологічний процес найбільш ймовірний?

A. *Жирова дистрофія міокарда

B. Ожиріння серця

C. Дилятаційна кардіоміопатія

D. Міомаляція

E. Кардіосклероз

25. При розтині жінки 40 років, яка страждала ревматоїдним артритом знайдено збільшену щільну селезінку. На розрізі її тканина коричнево-червоного кольору зі збільшеними фолікулами, які мають вигляд напівпрозорих сірувато-білуватих зерен. Вкажіть, який з перелічених патологічних процесів має місце.

A. * Сагова селезінка.

B. Глазурна селезінка.

C. Сальна селезінка.

D. Гіаліноз селезінки.

E. Порфірна селезінка.

26.У хворого 71 років з атеросклерозом, з'явився біль в лівій стопі. До лікаря не звертався. На момент огляду стопа збільшена в об'ємі, тканини дряблі, чорного кольору, мацеровані. Демаркаційна зона не виражена Діагностуйте патологічний процес.

A. Волога гангрена.

B. Муміфікація.

C. Коагуляційнний некроз.

D. Суха гангрена.

E. Секвестр.

27. При автопсії чоловіка середніх років, що тривало страждав бронхоектатичною хворобою, виявлено, що обидві надниркові залози різко збільшені в об'ємі за рахунок кіркової речовини. Надниркові залози щільні, бліді, сального вигляду. Мікроскопічно по ходу ретикулярної строми, в стінках судин - відкладення безструктурних, гомогенних, еозинофільних мас і позитивних для конго-рота мас. Ці зміни характерні для:

A. *Амилоїдозу

B. Мукоїдного набухання

C. Фібриноїдного набухання

D. Ліпідозу

E. Гіалінозу

28. Після перенесеного геморагічного інсульту у хворого розвинулася кіста головного мозку. Через 2 роки помер від післягрипозної пневмонії. На секції трупа виявлено в мозку кісту із стінками біло-ржавого відтінку,реакція Перлса позитивна. Який з процессів найбільш вірогідний у стінці кісти?

A. *Місцевий гемосидероз

B. Загальний гемосидероз

C. Місцевий гемомеланоз

D. Інфільтрація білірубіну

E. Первинний гемохроматоз

29.У померлого 45 років від раптової зупинки серця знайдено симетричний тип ожиріння ІІІ ступеню, розрив стінки правого шлуночку з гемоперікардом; під епікардом збиткове відкладення жиру. Мікроскопічно - жирова тканина з епікарду проникає у міокард з атрофією м’язових волокон. Вкажіть, який з перелічених патологічних процесів найбільш вірогідний?

A. * Просте ожиріння серця.

B. Жирова дистрофія міокарду.

C. Ішемічна хвороба серця.

D. Гіпертонічна хвороба.

E. Гострий інфаркт міокарду.

30. При розтині 56-річного чоловіка, який страждав фіброзно-кавернозним туберкульозом легень, знайдено збільшену у розмірах щільну селезінку. На розрізі тканина селезінки коричнево-рожевого кольору, гладка, з вісковидною поверхнею. Який з перелічених патологічних процесів найбільш вірогідний?

A. * Сальна селезінка.

B. Глазурна селезінка.

C. Порфірна селезінка.

D. Сагова селезінка.

E. Ціанотична індурація

31.У чоловіка 34 років з туберкульозом надниркових залоз шкіра на розтині сірувато-коричневого кольору, артеріальний тиск (прижиттєво) був знижений, спостерігалась адинамія та зниження рівня 17-оксикортикостероїдів в сечі та плазмі крові. Порушення якого пiгменту обумовило клiнiчнi прояви у хворого?

A. *Меланiну

B. Бiлiрубiну

C. Лiпофусцину

D. Лiпохрому

E. Гемосидерину

32. У шматочку шкіри 1х2 см, який доставлений для гістологічного дослідження, знайдено новоутворення бурого кольору діаметром 0,5 см. Мікроскопічно пухлина складається з невусних клітин у вигляді тяжів та гнізд, розташованих у дермі, з бурим пігментом у цитоплазмі, який дає негативну реакцію Перлса. Який пігмент найбільш вірогідний?

A. *Меланін

B. Гематоїдин

C. Гемосидерин

D. Білірубін

E. Гемомеланін

33.У дівчини 18 років з'вились різкий біль при ковтанні, збільшення лімфатичних вузлів шиї, підвищення $t^o$ до $39^o$С. На слизовій оболонці мигдаликів – біло-жовті плівки, які важко відділяються з утворенням дефекту. Стан прогресивно погіршувався. Хвора померла на 8-й день захворювання при наростаючих явищах серцевої недостатності. Які гістологічні зміни в кардіоміоцитах найімовірніше будуть виявлені?

A. *Жирова дистрофія

B. Гідропічна дистрофія

C. Гіаліново-крапельна дистрофія

D. Балонна дистрофія

E. Слизова дистрофія

34. Хворий 65 років, що страждав на атеросклероз, госпіталізований до хірургічного відділення з приводу розлитого гнійного перитоніту. Під час операції діагностовано тромбоз брижових артерій. Яка найбільш імовірна причина перитоніту?

A. *Геморагічний інфаркт

B. Ішемія ангіоспастична

C. Ішемічний інфаркт

D. Стаз

E. Ішемія компресійна

35. У чоловіка, що за життя страждав на бронхоектатичну хворобу, пневмосклероз з вираженими явищами кахексії, на розтині серце зменшене в розмірах, стінки стоншені, в'ялої консистенції, на розрізі тканина бурого кольору. Відкладання якого пігменту спостерігається в міокарді?

A. *Ліпофусцин

B. Гемосидерин

C. Гематоїдин

D. Цероїд

E. Ліпохроми

36.Макроскопічно печінка збільшена в розмірах, ущільнена, тканина сірувато-жовтого кольору, з сальним блиском. Який патологічний процес лежить в основі писаних змін?

A. *Амілоїдоз

B. Гіаліноз

C. Жирова дистрофія

D. Мукоїдне набухання

E. Гемахроматоз

37.У жінки віком 45 років, яка померла від хронічної алкогольної інтоксикації, на аутопсії виявлена різко збільшена печінка, тістоподібної консистенції, жовтоватого кольору. Мікроскопічно в цитоплазмі гепатоцитів при фарбуванні гаматоксилино та еозином виявляються різних розмірів оптично порожні вакуолі. Який вид дистрофії має місце?

A. *Паренхіматозна жирова

B. Вуглеводна паренхіматозна

C. Гіаліново-крапельна

D. Мезенхімальна жирова

E. Гідропічна

38. При ультраструктурному вивченні біоптату тканини встановлено, що в цитоплазмі макрофага, який входить до складу запального інфільтрату, виявляється значне збільшення кількості лізосом. Це свідчить про:

A. *Активацію фагоцитозу

B. Проліферативну активность клітини

C. Сповільнення фагоцитозу:

D. Активацію апоптозу

E. Сповільнення апоптозу

39.У жінки з важкою інтоксикацією, зумовленою сепсисом, який і послужив безпосередньою причиною смерті, на розтині виявлене “тигрове серце”. Мікроскопічно в цитоплазмі кардіоміоцитів виявлені ліпіди. Який морфогенетичний механізм розвитку переважно лежить в основі даної дистрофії?

A. *Декомпозиція

B. Інфільтрація

C. Трансформація

D. Спотворений синтез

40. Чоловік віком 63 роки хворіє на хронічний фіброзно-кавернозний туберкульоз легень протягом 24 років, поступив у нефрологічне відділення з явищами уремії. Прижиттєва проба на наявність у нирках амілоїду виявилася позитивною. Яка форма амілоїдозу в даному випадку має місце?

A. *Вторинний системний

B. Первинний системний

C. Обмежений (місцевий)

D. Сімейний вроджений

E. Сенильний (старечий)

41. У дитини, після перенесеного кору, при огляді виявлено у м’яких тканинах щік і промежини нечітко відмежовані, набряклі, червоно-чорного кольору ділянки, які злегка флуктують. Яке ускладнення розвинулося у дитини?

A. *Волога гангрена (нома)

B. Суха гангрена

C. Газова гангрена

D. Пролежень

E. Трофічна виразка

42.У новонародженої дитин при огляді виявлено дефект в ділянці твердого піднебіння у вигляді порожнини. Внаслідок якого процесу він утворився?

A. *Порушення апоптозу

B. Атрофії

C. Некрозу

D. Дистрофії

E. Запалення

43. При електронномікроскопічному дослідженні клітини було встановлено, що вона загинула внаслідок некрозу, а не апоптозу, оскільки для некрозу характерним є:

A. *Дифузна локалізація хроматину в клітині

B. Фагоцитоз апоптозних тілець

C. Відсутність запальної відповіді

D. Збереження цілісності органел

E. Розщеплення ДНК

44. При електронномікроскопічному дослідженні біоптату печінки виявлено, що між чисельними мітохондріями знаходиться велика кількість плоских цистерн і міхурців із секреторними гранулами, стінки яких утворені мембранами. Про гіперплазію складових якої ультраструктури йде мова?

A. *Апарату Гольджі

B. Піноцитозних міхурців

C. Ендоплазматичного ретикулуму

D. Лізосом

E. Мікротрубочок

45.Під час розтину тіла жінки, яка померла внаслідок пухлинної дисемінації муцинозної цистаденокарциноми і тривалий час мала вимушене положення в ліжку, були знайдені велики ділянки некрозу шкіри та підлеглих м’яких тканин крижової ділянці. Діагностуйте форму некрозу.

A. *Пролежень

B. Інфаркт

C. Секвестр

D. Сирнистий некроз

E. Воскоподібний (ценкеровський) некроз

46. Під час розтину трупа чоловіка 55 років, який протягом останніх восьми років хворів на хронічну форму тропічної малярії, виявлено, що сіра речовина головного мозку та селезінка аспідно-сірого кольору. Який пігмент зумовив таке забарвлення?

A. *Гемомеланін

B. Ліпофусцин

C. Гематопорфірин

D. Меланін

E. Гемосидерин

47.У дівчинки 6 років, яка перенесла кір, при огляді лікар виявив у м’яких тканинах щік нечітко відмежовані, набряклі, червоно-чорного кольору ділянки. Назвіть ускладнення кіру

A. *Волога гангрена [нома]

B. Суха гангрена

C. Газова гангрена

D. Пролежень

E. Трофічна виразка

48. Жінка 36 років, яка хворіла на дифтерію, померла від гострої серцевої недостатності. На розтині порожнини серця розширені, м’яз серця тьмяний, строкатий, на розрізі з жовтуватими ділянками. Який процес виявлений у кардіоміоцитах?

A. *Жирова дистрофія

B. Вуглеводна дистрофія

C. Балонна дистрофія

D. Гіаліново-крапельна дистрофія

E. Гідропічна дистрофія

49.У хворого на туберкульоз в біоптаті нирки при гістологічному дослідженні у вогнищі казеозного некрозу виявлені безпорядно розсипані дрібні зерна хроматину. Наслідком чого є виявлені зміни?

A. * Каріорексису

B. Каріолізису

C. Пікнозу ядер

D. Мітотичної активності ядер

E. Апоптозу

50.У хворого в харкотинні та біоптаті легень у великій кількості були знайдені клітини з внутрішньоклітинними коричневими гранулами, що давали позитивну реакцію на залізо. Проявом якого процесу є наявність коричневих гранул у цитоплазмі?

A. * Фагоцитозу

B. Макропіноцитозу

C. Мікропіноцитозу

D. Апоптозу

E. Резорбції

51.У хворого о 8 годині ранку з’явилися болі за грудиною, о 9 годині ранку в приймальному відділенні за даними ЕКГ встановлений інфаркт міокарда. Через 10 хвилин хворий помер. Яку найбільш достовірну морфологічну ознаку інфаркту міокарда знайдуть при гістологічному дослідженні після розтину трупа?

A. * Зникнення глікогену в кардіоміоцитах

B. Вакуольну дистрофію кардіоміоцитів субендокардіальної зони

C. Жирову інфільтрацію кардіоміоцитів

D. Некроз міокардіоцитів

E. Розслаблення міофібрил кардіоміоцитів

52. При зовнішньому огляді новонародженого виявлялися виражені зміни шкірних покривів всього тіла. Шкіра суха, тьмяна із нерівною поверхнею та з наявністю сірих пластин, які відшаровуються. З яким видом дистрофії пов’язана ця картина?

A. *Роговою

B. Гідропічною

C. Гіаліново-крапельною

D. Фібриноїдним набуханням

E. Мукоїдним набуханням

53. У хворого, з двобічним ураження надниркових залоз, з’явилися темне коричневе пофарбування шкірних покривів, при гістохімічному дослідженні шкіри реакція Перлса негативна. Який пігмент зумовив зміну кольору шкіри?

A. *Меланін

B. Гемосидерин

C. Порфірин

D. Ліпофусцин

E. Білівердін

54. На аутопсії жінки, яка хворіла на хронічну дизентерію, в стромі та паренхімі міокарду, нирок, в слизовій оболонці шлунку, у сполучній тканині легень виявлені фіолетового кольору аморфні відкладання, позитивні при реакції Косса.. Який патологічний процес розвинувся у внутрішніх органах хворої?

A. *Метастатичне звапніння

B. Дистрофічне звапніння

C. Метаболічне звапніння

D. Амілоїдоз

E. Гіаліноз

55.При дослідженні біоптату шкіри хворого алергічним васкулітом знайдено: стінка судин потовщена, гомогенна, пікрофуксином забарвлена в жовтий колір, ШИК-позитивна. Який патологічний процес розвинувся в стінках судин?

A. *Фибриноїдне набухання

B. Амілоїдоз

C. Мукоїдне набухання

D. Гиаліноз

E. Ліпідоз

56.При розтині трупа чоловіка, який помер від черевного тифу,виявлено, що прямі м’язи передньої черевної стінки щільні, білуватого кольору, нагадують стеаринову свічку. Який патологічний процес має місце?

A. * Воскоподібний некроз

B. Фібриноїдний некроз

C. Колікваційний некроз

D. Казеозний некроз

E. Апоптоз

57. У чоловіка 58 років, померлого при наростаючих явищах хронічної серцевої недостатності, діагностовано ревматичний гранулематозний міокардит. Мікроскопічно в міокарді спостерігаються гранульоми, що складаються з макрофагів з гіперхромними ядрами та світлою цитоплазмою, в центрі осередок некрозу. Який характер носить некроз в середині гранульом?

A. *Фібриноїдний.

B. Ценкеровський.

C. Казеозний.

D. Колікваційний.

E. Жировий.

58. До хірурга звернувся чоловік 60 років, що тривалий час страждав на сахарний діабет. Тканини правої стопи були чорного кольору, щільні з чіткими краями. Який діагноз поставив лікар?

A. *Суха гангрена.

B. Волога ганрена.

C. Газова гангрена.

D. Пролежень.

E. Трофічна виразка.

59. Під час огляду трупу відмічено: мутні роговиці, сухі шкірні покрови із жовто-бурими пергаментного вигляду плямами. Встановіть вид посмертних змін.

A. *Трупне висихання

B. Перерозподіл крові

C. Трупні плями

D. Трупне задубіння

E. Охолодження трупа

60. При мікроскопічному дослідженні інтими аорти (забарвлення суданом) було виявлено: відкладання ліпідів у вигляді крапель і кристалів холестерину, включення жиру спостерігаються в ксантомних клітинах, серед відкладань жирів – розростання сполучної тканини. Дайте визначення гістологічному процесу

A * Стромально-судинний ліпідоз

B Паренхіматозний диспротеїноз

C Паренхіматозний ліпідоз

D Стромально-судинний диспротеїноз

E Амілоїдоз

61.При гістологічному дослідженні нирок молодої жінки 25 років, померлої. Під час пологів, в епітелії канальців головних відділів нефрону виявлені конденсація хроматину ядер, розпад їх на глибки і лізис, а також плазморексис і цитоліз. Який патологічний процес виявлено в епітелії канальців нирок?

A * Некроз

B Гідропічна дистрофія

C Амілоїдоз

D Жирова дистрофія

E Гіаліноз

62.При морфологічному дослідженні печінки хворого, який довго страждав лімфогранульоматозом і помер від гепатаргії, орган збільшений, на розрізі сального вигляду. Гістологічно - між зірчатими ендотеліоцитами синусоїдів, по ходу ретикулярної строми судин, протоків і портальних трактів виявлені конгофільні маси. Гепатоцити частково атрофовані. Який патологічний процес виявлений в печінці?

A * Амілоїдоз

B Гіаліноз.

C Жирова дистрофія

D Відкладання глікогену

E Гіаліново -крапельна дистрофія

63.У хворого з клінікою нефротичного синдрому в біоптаті нирки виявлено в клубочках відкладення однорідних еозинофільних мас, які при забарвленні гістологічних зрізів методом Конго-червоний набували червоного кольору. Який вид дистрофії виник у даному випадку?

A * Амілоїдоз.

B Мукоїдне набухання.

C Фібриноїд.

D Гіаліноз.

E Вуглеводна дистрофія.

64.У 43-річного чоловіка, що тривало хворів на остеомієліт і помер від прогресуючої ниркової недостатності, при патоморфологічному дослідженні нирок виявлено відкладення гомогенних рожевих мас в мезангіумі клубочків, навколо артеріол та артерій. Діагностуйте вид патологічного процесу.

A * Амілоїдоз.

B Гіаліноз.

C Склероз.

D Некроз.

E Кальциноз.

Пухлини.

1. В шкірі виявлена щільна, рухома, чітко відмежована від оточуючих тканин пухлина. На розрізі вона білого кольору, представлена волокнистою тканиною. Мікроскопічно – хаотично переплетені колагенові волокна, клітин мало. Назвіть пухлину?

A. * фіброма

B. міома

C. Гістіоцитома

D. Дерматофіброма

E. Десмоїд.

2. При гістологічному дослідженні новоутворення шкіри виявлено: паренхіма сформована з покривного епітелію із збільшеним числом шарів. Строма разом з розростаннями епітелію формує сосочки. Вкажіть вид атипізму.

A. * Тканинний

B. Клітинний.

C. Гістохімічний

D. Функціональний.

E. Метаболічний.

3. Для гістологічного дослідження доставлене очне яблуко, у судинній оболонці якого виявлене пухлиноподібне утворення 1 х 0,4 см чорного кольору.. У клітках – множинні патологічні мітози і у цитоплазмі багатьох з них визначається пігмент жовто-бурого кольору. Ваш діагноз?

A. *Меланома

B. Неврінома

C. Ангіосаркома

D. Нейробластома

E. Гангліонейробластома

4.При мікроскопічному дослідженні біоптату з товстої кишки виявлена пухлина з призматичного епітелію, що формує атипові залозисті структури різної форми і розміру. Клітини поліморфні, ядра гіперхромні, є патологічні мітози. Базальна мембрана залоз зруйнована. Ваш діагноз.

A. *Аденокарцинома

B. Базально-клітинний рак

C. Солідний рак

D. Слизовий рак

E. Недиференційований рак

5.При мікроскопічному дослідженні біоптата шийки матки виявлена клітинна і ядерна атипія багатошарового плоского епітелію, патологічні мітози, а також рогові перлини в глибині епітеліальних шарів. Ваш діагноз:

A. *Плоскоклітинний рак зі зроговінням

B. Перехідноклітинний рак.

C. Плоскоклітинний рак без зроговіння

D. Залозистий рак.

E. Анапластичний рак.

6. При гістологічному дослідженні зішкріба слизової оболонки матки у хворої 54 років з клінічним діагнозом порушення оваріально-менструального циклу виявлено – розростання атипових залозистих структур, що складаються з клітин з гіперхромними ядрами, фігурами мітозів, атипією. Атипові залозисті структури вростають в міометрій..Для якого патологічного процесу характерні виявлені мікроскопічно зміни?

A. * Аденокарцинома матки.

B. Залозиста гіперплазія ендометрію.

C. Гострий ендометрит.

D. Плацентарний поліп.

E. Хоріонепітеліома матки.

7.У жінки 46 років під час паліативної операції з приводу раку шлунку встановлена наявність крукенбергівських метастазів в яєчники (“крукенбергівський рак яєчників”). Який з наведених шляхів метастазування призвів до ураження яєчників?

A. *Лімфогенний ретроградний

B. Лімфогенний ортоградний

C. Гематогенний

D. Імплантаційний

E. Каналікулярний

8.При гістологічному дослідженні пухлини шлунка виявлено багато перстневидних клітин. Назвіть гістологічний варіант рака.

A. *Слизовий

B. Солідний

C. Аденокарцинома

D. Саркома

E. Карциноїд

9. При розтині дитини 3 років у мозочку виявлена пухлина, що не має чітких меж з оточуючою тканиною, гістологічно побудована з атипових дрібних клітин з гіперхромними ядрами. Найвірогідніше це:

A. *медулобластома

B. медулосаркома

C. метастаз раку

D. метастаз саркоми

E. гліобластома

10.У хворого, з'явилася охриплість голосу, під час лярінгоскопії виявлена пухлина гортані, сіро-білого кольору, з сосочковою поверхнею. Мікроскопійно: розростання сполучної тканини, що вкрите багатошаровим плоским епітелієм зі збільшеним ороговінням, без клітинного атипізму. Найвірогідніше це:

A. *папілома

B. фіброма

C. поліп

D. ангіома

E. ангіофіброма

11. У хворого 75 років з позачеревної клітковини видалена пухлина розмірами 16х8х6см. Мікроскопійно: анаплазовані жирові клітини з ознаками клітинного атіпізму, поліморфізму. Зустрічаються величезні спотворені клітини, що мають у цитоплазмі жирові крапельки. Який найбільш імовірний діагноз?

A. *ліпосаркома

B. ліпома

C. фібросаркома

D. міосаркома

E. мезотеліома

12. При розтині померлого хворого виявлена гіперплазія кісткового мозку плоских і трубчастих кісток (піоїдний кістковий мозок), спленомегалія (6 кг), гепатомегалія (5 кг), збільшення всіх груп лімфатичних вузлів. Якому захворюванню відповідають виявлені зміни?

A. * Хронічний мієлолейкоз

B. Хронічний лімфолейкоз

C. Мієломна хвороба

D. Справжня поліцитемія

E. Лімфогрануломатоз

13. У чоловіка віком 62 роки видалено нирку, у якій при макроскопічному дослідженні виявлено пухлину у вигляді вузла діаметром до 8см. Тканина пухлини на розрізі строката, з множинними крововиливами, некрозами. Гістологічно: пухлина складається із світлих клітин, які утворюють альвеолярні і сосочкові структури, помірно виражений інвазивний ріст. У багатьох клітинах пухлини визначаються патологічні мітози, гіперхромні ядра. Діагностуйте виявлену пухлину нирки.

A. * Світлоклітинний рак

B. Світлоклітинна аденома

C. Аденокарцинома

D. Нефробластома

E. Ацидофільна аденома з малігнізацією

14.При мікроскопічному дослідженні бронхобіопсії виявлена пухлина, яка побудована з гніздних скупчень атипових клітин багатошарового плоского епітелію, місцями із характерними “перлинами”. Ваш діагноз?

A. * Плоскоклітинний рак із ороговінням.

B. Плоскоклітинний рак без ороговіння.

C. Солідний рак.

D. Слизовий рак.

E. Скір.

15.У хворого видалено новоутворення шкіри, яке має вигляд вузла з сосочковою поверхнею, що нагадує цвітну капусту, щільної консистенції. Мікроскопічно пухлина складається з багатьох сосочків. Паренхіма сформована з покривного епітелію із збільшеним числом шарів. В епітелії збережена полярність клітин, стратифікація, цілісність власної мембрани. Строма пухлини розташована в центрі сосочків. Який найбільш імовірний діагноз?

A. *Папілома

B. Фіброма

C. Аденома

D. Фіброаденома

E. Цистоаденома

16. При гістологічному дослідженні збільшеного шийного лімфатичного вузла відмічено, що загальний рисунок вузла нечіткий за рахунок розростання атипових гістіоцитарних клітин, з наявністю гігантських клітин Березовського-Штернберга, маються ділянки некрозу, склерозу, гіалінозу. Для якого захворювання характерні виявлені морфологічні зміни в лімфовузлі?

A. *Лімфогранулематозу

B. Гострого мієлолейкозу

C. Хронічного мієлолейкозу

D. Грибовидного мікозу

E. Туберкульозу

17.У хворого 60 років при торакотомії в передньому середостінні знайдені збільшені в розмірах та спаяні між собою лімфатичні вузли, з яких взято біоптат. Мікроскопічно виявлено атипові клітини, серед яких переважають клітини Ходжкіна і гігантські клітини Рід-Березовського-Штернберга; склероз відсутній. Яке захворювання найбільш відповідає цим ознакам ?

A. *Лімфогранулематоз з пригніченим розвитком лімфоїдної тканини

B. Лімфогранулематоз з переважанням нодулярного склерозу

C. Лімфогранулематоз з переважанням лімфоїдної тканини

D. Лімфосаркома

E. Змішано-клітинний варіант лімфогранульоматозу

18. При томографії у хворого в середостінні знайдені збільшені лімфатичні вузли. При гістологічному дослідженні в лімфовузлі виявлені циркулярні розростання сполучної тканини, яка оточувала гранульомоподібні утворення з лімфоцитів, плазмоцитів і великих двоядерних клітин. Ваш діагноз.

A. *Лімфогрануломатоз

B. Лімфосаркома

C. Туберкульоз

D. Саркоїдоз

E. Лімфолейкоз

19.Для гістологічного дослідження доставлена видалена на операції матка. Під слизовою оболонкою визначені численні округлої форми вузли, які чітко відмежовані від оточуючої тканини. Мікроскопічно пухлина побудована з пучків гладкої мускулатури із явищами тканинного атипізму. Ваш діагноз?

A. *Лейоміома.

B. Рак матки.

C. Фіброміоми.

D. Хоріонепітеліома

E. Лейоміосаркома

20. При мікроскопічному дослідженні пухлини верхньої губи, виявлено, що вона побудована з числених щілиноподібних порожнин, стінка яких вистелена сплощеним ендотелієм, заповнених рідкою кров`ю і згортками. Поставити діагноз.

A. *Капілярна гемангіома.

B. Венозна гемангіома.

C. Кавернозна гемангіома.

D. Гемангіоперицитома.

E. Гломус-ангіома.

21. У хворого 17 років інтраопераційно на нижній поверхні печінки виявлена пухлина розмірами 4,5х5,0х3,5см. з субсерозною локалізацією, темночервоного кольору, на розрізі представлена порожнинами значним вмістом крові. Поставити попередній діагноз.

A. *Кавернозна гамангiома.

B. Капiлярна гемангіома.

C. Гемангiоперицитома.

D. Гемангiоендотелiома.

E. Лiмфангiома.

22.При гістологічному дослідженні злоякісної пухлини молочної залози виявлено, що пухлина побудована з низькодиференційованих атипових клітин епітеліального походження, які утворюють трабекули, відокремлені між собо прошарками сполучної тканини, співвідношення клітин і строми приблизно 1:1. Назвіть гістологічний варіант рака.

A. *Солідний

B. Аденокарцинома

C. Плоскоклітинний

D. Фіброзний

E. Дрібноклітинний

23.У молодої людини збільшилися шийні лімфатичні вузли. При біопсії в лімфовузлі виявлені вогнища проліферації лімфоїдної тканини з наявністю велетенських клітин Березовського-Штернберга і еозинофілів, вогнища некро-з та фіброзу. Яке захворювання найбільш вірогідно буде мати місце?

A. *Лімфогрануломатоз

B. Хронічний лімфолейкоз

C. Мієломна хвороба

D. Лімфосаркома

E. Гістіоцитоз

24.У молодої людини в м'яких тканинах лівого стегна з'явилось безболюче новоутворення без чітких меж. В біоптаті тканини новоутворення нагадують риб'яче м'ясо, складаються з незрілих фибробластоподібних клі-тин численими мітозами, які прорастають в м'язи. Ваш діагноз?

A. *Фібросаркома

B. Міосаркома

C. Фіброма

D. Рак

E. Міома

25. У хворого внаслідок обтурації середньодольового бронха вузлом м'яких тканин утворився ателектаз середньої долі правої легені. При бронхобіопсії в зоні обтурації виявлені розростання залозоподібного атипового епітелію з патологічними мітозами, який прорастає в підлеглі тканини і хрящ. Якому захворюванню найбільш вірогідно відповідають ці дані?

A. *Бронхогенний рак легень

B. Деформуючий бронхіт

C. Запальний поліп

D. Дисплазія епітелію бронха

E. Саркома бронха

26. При мікроскопічному дослідженні біоптату з легень виявлено атипові клітини, які формують множинні ацинарні структури і продукують слиз. Яка гістологічна форма раку легень має місце у хворого?

A. *Високодиференційована аденокарцинома.

B. Помірно диференційована аденокарцинома.

C. Недиференційований рак.

D. Помірнодиференційований плоскоклітинний рак.

E. Залозистоплоскоклітинний рак.

27. У хворого при гастроскопічному дослідженні в ділянці малої кривизни виявлено пухлинне утворення діаметром 1,5 см на ніжці. Який характер росту пухлини?

A. *Екзофітний.

B. Експансивний.

C. Інфільтруючий

D. Апозиційний

E. Ендофітний

28. При мікроскопічному дослідженні біоптату з ділянки пухлини слизової оболонки правого бронху, встановлено клітинний та тканинний атипізм, появу структур у вигляді 'ракових перлин'. Визначити характер патологічного процесу.

A. *Злоякiсна пухлина.

B. Доброякiсна пухлина.

C. Гiперплазія.

D. Метаплазія.

E. Гiпоплазія.

29. При гістологічному дослідженні пухлини шкіри виявлені різних розмірів часточки з жирової тканини, відмежовані нерегулярними прошарками сполучної тканини. Поставити діагноз.

A. *Лiпома.

B. Фiброма.

C. Гiгрома.

D. Папiлома.

E. Гемангiома.

30. При дослідженні пухлини матки (післяопераційний матеріал) макроскопічно вона м'якої консистенції, з крововиливами та ділянками вогнищевого некрозу, на розрізі нагадує 'риб'яче м'ясо'. При гістологічном дослідженні встановлено ознаки вираженого клітинного та тканинного атипізму, зустрічаються клітини з патологічними фігурами мітозів. Поставити діагноз.

A. *Саркома.

B. Аденокарцинома.

C. Ангiома.

D. Фiброма.

E. Лiпома.

31. При гістологічному дослідженні біоптату піхвової частини шийки матки у хворої 47 років з тривало не заживаючою ерозією виявлені ознаки клітинного атипізму, базальна мембрана - без змін. Поставити діагноз.

A. *Карцинома in situ.

B. Ерозiя.

C. Аденокарцинома.

D. Папiлома.

E. Ендометрiоз.

32.У хворої 45 років знайдені збільшенними надключичні лімфатичні вузли. При дослідженні біопсійного матеріалу із лімфатичного вузла виявлений метастаз перстненодыбно-клітинного раку. Виберить найбільш ймовірну локалізацію первинної пухлини.

A. *Рак шлунка.

B. Рак стравохода.

C. Рак щитовидної залози.

D. Рак легень.

E. Рак шийки матки.

33.У хворої 55 років появилися рецидивуючі маткові кровотечі. Виконано діагностичне вишкребання матки. В зіскобі ендометрію серед елементів крові видно залозисті елементи різної величини і форми, утворені атиповим клітинами з гіперхромними ядрами, з багаточисленними мітозами (в тому числі і патологічними). Про який процес можна думати?

A. *Аденокарцинома

B. Залозиста гіперплазія ендометрію.

C. Хоріонепітеліома

D. Аденоматозний поліп.

E. Ознаки перерваної вагітності.

34.Пухлина у капсулі діаметром 2 см, оперативно видалена з ампутаційної культі нижньої кінцівки, мікроскопічно складається із веретеноподібних клітин мономорфного вигляду з паличкоподібними ядрами, які утворюють разом з волокнами “палісадні” структури. Який з перелічених видів пухлин найбільш вірогідний?

A. *Доброякісна неврілемомма

B. нейрофіброма

C. Злоякісна неврілемомма

D. М'яка фіброма

E. Фібросаркома

35. При гістологічному дослідженні лімфатичного вузла - дифузне розростання лімфоїдних клітин з домішкою еозинофілів та атипових гістіоцитів, поодинокі гігантські клітини з двома та більш ядрами, осередки некрозу та склерозу. Найбільш вірогідний діагноз:

A. *Лімфогранульоматоз

B. Лімфолейкоз

C. Лімфома Беркіта

D. Саркоїдоз

E. Мієлолейкоз

36. На розтині померлого знайдено системне збільшення лімфатичних вузлів з утворенням пухлинних конгломератів. Селезінка збільшена, на розрізі строката: на червоному тлі пульпи множинні дрібні осередки жовтувато-білого та сіруватого кольору. Якому захворюванню найбільше відповідають описані зміни?

A. *лімфогранульоматоз

B. саркоїдоз

C. лімфосаркома

D. рак легенів

E. лімфолейкоз

37. Для патогістологічного дослідження доставлене очне яблуко: у сітківці ока на проєкції окового нерву пухлина 1х1см, м(якої консистенції, коричневого кольору. Мікроскопійно пухлина складаэться з поліморфних клітини з прозорою цитоплазмою. сгрупованих у альвеолярні структури, наявныстью патологічних мітозів, подекуди з накопиченням бурого о пігменту. Найвірогідніше це:

A. *меланома

B. шваннома

C. гломусна пухлина

D. парагангліома

E. ангіосаркома

38. При бронхоскопії у хворого 62 років у початковому відділі верхньодольового бронху правої легені знайдено поліпоподібне утворення діаметром до 1,0 см. При гістологічному дослідженні виявлено пухлину, яка побудована з дрібних лімфоцитоподібних клітин з гіперхромними ядрами, клітини ростуть пластами та тяжами. Вкажіть, який з перелічених видів пухлин найбільш вірогідний?

A. * Недиференційований дрібноклітинний рак.

B. Недиференційований великоклітинний рак.

C. Плоскоклітинний рак.

D. Аденокарцинома.

E. Залозисто-плоскоклітинний рак.

39. У товщі шкіри макроскопічно знайдена щільна пухлина, рухлива. При мікроскопії вона представлена хаотично розташованими пучками колагенових волокон з невеликою кількістю веретеноподібних клітин. Яка пухлина видалена?

A. *Щільна фіброма.

B. Лейоміома.

C. Меланома.

D. Ліпома.

E. Гломус-ангіома.

40. У хворого на шкірі обличчя поступово розвилась бляшка з некрозом і виразкою в центрі. При патогістологічному дослідженні біоптату виявлено розростання атипових епітеліальних клітин з великою кількістю патологічних мітозів. Який найбільш вірогідний діагноз?

A. *Рак шкіри

B. Саркома

C. Папілома

D. Трофічна виразка

E. Фіброма

41.У хворого 21 року видалена пухлина лобної частки правої півкулі головного мозку діаметром 5 см, яка була нечітко відмежована від довколишньої тканини. На розрізі - однорідного вигляду, гістологічно - складається із клітин, численні відростки яких утворюють густі сплетіння. Яка пухлина мала місце у хворого?

A. *Астроцитома

B. Олігодендрогліома

C. Гангліоневрома

D. Епендімома

E. Хоріоїдпапілома

42. При дослідженні біоптату лімфатичного вузла дано заключення "Лімфогранульоматоз". На підставі наявності яких ознак був поставлений діагноз?

A. *Клітин Березовського-Штернберга

B. Клітин Тутона

C. Клітин Пирогова-Лангханса

D. Клітин Вірхова

E. Клітин Мікулича

43. При розтині жінки 33 років знайдено потовщення стінки шлунка в пілоричному відділі (на розрізі шари стінки розрізняються) з розростанням щільної білуватої тканини в підслизовому шарі і дрібними тяжами її в м'язовому шарі. Рельєф слизової оболонки збережений, складки ригідні, нерухомі. Яка макроскопічна форма пухлини в даному випадку?

A. *Інфільтрат

B. Вузол

C. Виразка

D. Кіста

E. Інфільтративно-виразкова форма

44.У збільшеному шийному лімфатичному вузлі дівчинки 14 років, мікроскопічно були знайдено, що тканинна будова вузла порушена, лімфоїдні фолікули відсутні, є ділянки склерозу та вогнища некрозу, клітинний склад вузла поліморфний, присутні лімфоцити, еозинофіли, атипові клітини великих розмірів з багаточасточковими ядрами (клітини Березовського-Штернберга) та одноядерні клітини також великих розмірів. Поставте діагноз.

A. *Лімфогранулематоз

B. Гострий лімфолейкоз

C. Хронічний лімфолейкоз

D. Лімфома Беркіта

E. Грибовидний мікоз

45. При розтині трупу чоловіка, померлого від ракової інтоксикації, при макроскопічному дослідженні стінка шлунку потовщена до 1,2 см, слизова оболонка нерухома, її складки не виявляються. На розрізі однорідна, білісувата, хрящоподібної щільності. Для якої макроскопічної форми пухлини характерні описані зміни?

A. *Інфільтрат

B. Вузол

C. Виразка

D. Виразково-інфільтративна

E. Кіста

46. В біоптаті пухлини молочної залози виявляються солідні пласти, побудовані з дрібних епітеліальних клітин з поліморфними ядрами, з великою кількістю патологічних мітозів. Строми дуже мало, з лімфоцитарною інфільтрацією. Встановити діагноз.

A. *Медулярний рак

B. Скірозний рак

C. Хвороба Педжета

D. Аденома

E. Аденофіброма

47.Під шкірою піднижньощелепної ділянки у жінки 55 років виявлено з чіткими межами тістуватої консистенції рухоме утворення 1,0x0,7 см з повільним ростом. При гістологічному дослідженні утворення виявлені ліпоцити, які формують часточки різні за формою та розміром, розмежовані тонкими прошарками сполучної тканини із судинами. Встановити діагноз.

A. *Ліпома

B. Фіброма

C. Ангіома

D. Ліпосаркома

E. Фібросаркома

48. Під час огляду стоматологом хлопчика 16 років були знайдені збільшені підщелепові та шийні лімфатичні вузли. Була проведена біопсія. Мікроскопово в лімфатичних вузлах було знайдено: типова будова стерта, клітинна популяція гетерогенна, присутні великі клітини з багатолопасним ядром, множинні одноядерні клітини великого розміру, еозинофільні та нейтрофільні лейкоцити, лімфоцити, окрім того знайдені ділянки некрозу та вогнища склерозу. Діагностуйте захворювання.

A. *Лімфогранулематоз

B. Гіперплапзія лімфатичного вузла

C. Гранулематозний лімфаденіт

D. Гнійний лімфаденіт

E. Неходжкінська лімфома

49. У жінки 22 роки виявлені збільшені лімфатичні вузли. При гістологічному дослідженні в лімфатичному вузлі спостерігаються лімфоцити, гістіоцити, ретикулярні клітини, малі та великі клітини Ходжкіна, багатоядерні клітини Березовського-Штернберга, поодинокі осередки казеозного некрозу. Для якого захворювання характерні такі зміни?

A. *Лімфогранулематоз.

B. Лімфосаркома.

C. Хронічний лейкоз.

D. Гострий лейкоз.

E. Метастаз рака легень

50. У чоловіка 46 років на шкірі визначалась пляма, яка вибухала і мала темний колір та не спричиняла турбот. З часом пляма почала збільшуватись, з’явилась біль, колір став чорно-коричневим і почав пальпуватися вузлик. На гістологічному дослідженні видаленої тканини визначалися веретеноподібні і поліморфні клітини, цитоплазма яких вміщувала пігмент бурого кольору. Про яку пухлину йдеться?

A. *Меланома.

B. Базаліома.

C. Гемангіома.

D. Гематома.

E. Карциноїд.

51.У чоловіка 36 років в заочеревинному просторі виявлено пухлиноподібне утворення розміром 17?15 см з простанням його в брижу, яке цілком вилучити хірург не зміг. Макроскопічно тканина утворення на розрізі жовтувата, м’яка, з осередками некрозу і ослизніння. При мікроскопічному дослідженні виявлено клітини з вираженим поліморфізмом ядер, з наявністю патологічних мітозів, цитоплазма клітин світла, дрібновакуолізована, при фарбуванні суданом вакуолі дають позитивну реакцію. Визначте пухлину.

A *Ліпосаркома

B Ліпома

C Фібросаркома

D Фіброма

E Гібернома

52.Жінці 32 років проведено секторальну резекцію молочної залози з наявністю пухлинного вузла діаметром 2,0 см, щільного, з чіткими контурами, на розрізі біло-рожевого кольору. У гістологічних зрізах паренхіму пухлини представлено залозами різної форми, які вкриті одним або кількома шарами дрібних темних епітеліоцитів без клітинного атипізму. Строма пухлини різко переважає над паренхімою, представлена щільною волокнистою сполучною тканиною. Вкажіть найбільш вірогідний гістологічний діагноз пухлини.

A *Фіброаденома

B Аденома

C Фіброма

D Аденокарцинома

E Фіброзний рак

53.Під час операції з приводу запаленого апендиксу, що розташований під печінкою, хірург випадково виявив у стінці шлунку хворого чітко відмежований вузол щільної консистенції, сірого кольору, волокнистий на розрізі, діаметром до 3,0 см. При мікроскопічному дослідженні виявлено: тканина вузла (з забарвленням пікрофуксином) складається з пучків веретеноподібних клітин без клітинного атипізму, що йдуть у різних напрямах і мають жовтий колір. Між ними зустрічаються невеликі прошарки сполучної тканини червоного кольору. Для якої пухлини шлунку характерні ці морфологічні зміни?

A *Лейоміома

B Фіброміома

C Лейоміосаркома

D Рабдоміома

E Рабдоміосаркома

54.Хворий помер від ракової кахексії при первинній локалізації раку у шлунку. На розтині тіла померлого виявлено різко збільшену печінку з нерівною поверхнею з безліччю вузлів, що випинаються і мають на розрізі чіткі межі, округлу форму, сіро-рожевий колір, різну щільність, місцями з осередками некрозу. Який патологічний процес розвинувся у печінці?

A *Метастази раку

B Абсцеси

C Регенераторні вузли

D Інфаркт

E Рак печінки

55.Хворій 57 років виконана резекція шлунку з приводу хронічної виразки, що кровоточить. При гістологічному дослідженні зрізів зі значно потовщеного краю виразки: хаотичні розростання залозистих структур різної форми та величини, що поширюються в м’язовий шар стінки шлунка. Залози утворені клітинами з незначними ознаками клітинного атипізму, у багатьох визначається багаторядність епітелію, руйнування базальних мембран. Який гістологічний діагноз найбільш імовірний?

A *Високодиференційована аденокарцинома

B Низькодиференційована аденокарцинома

C Недиференційований рак

D Аденома

56.В патогістологічну лабораторію для дослідження доставлено матку. У товщі міометрію виявлено 5 вузлів різного розміру, з чіткими межами, щільної консистенції, на розрізі сірого кольору, волокнистої будови. Мікроскопічно при забарвленні пікрофуксином видно різної товщини пучки веретеноподібних клітин, що йдуть у різних напрямах і забарвлені у жовтий колір. Між ними широкі прошарки сполучної тканини червоного кольору. Така морфологічна картина найбільш відповідає:

A *Фіброміомі

B Лейоміомі

C Лейміосаркомі

D Ракові

E Рабдоміомі

57.У дитини на шкірі передпліччя синюшний вузлик з бугристою поверхнею. Мікроскопічно вузлик складається з розгалужених судин капілярного типу з вузькими просвітами, базальною мембраною і декількома рядами ендотеліальних клітин. Назвіть пухлину.

A * Капілярна гемангіома

B Лімфангіома

C Венозна гемангіома

D Кавернозна гемангіома

E Папілома

58.При мікроскопічному дослідженні біоптату нирки виявлена пухлина, що складається з світлих полігональних і поліморфних клітин з множинними мітозами, що містять ліпіди. Клітини утворюють альвеоли і часточки, залозисті і сосочкові структури, які розділені незначною стромою з судинами. Ваш діагноз?

A * Нирково-клітинний рак

B Нефробластома

C Темноклітинна аденома

D Светлоклітинна аденома

E Ацидофільна аденома

59.Хворий зваернувся до лікаря з приводу пухлиноподібного утвору на передній поверхні грудної стінки, який пальпується у вигляді рухливого вузла. Пухлина видалена. Макроскопічно вона має чіткі межі, вилущується із капсули, на розрізі волокнистої будови, сіро-рожевого кольору. Гістологічно виявлений тканинний атипізм. Вкажіть тип пухлини за її клітинним перебігом.

A *Доброякісна

B Злоякісна

C З місцеводеструюючим ростом

D Апозиційна

E Експансивна

60. Під час бронхоскопії в правому головному бронху виявлена пухлина, яка майже повністю перекриває його просвіт. Вкажіть характер росту цієї пухлини по відношенню до просвіту бронха.

A *Екзофітний

B Експансивний

C Ендофітний

D Апозиційний

E Деструюючий

61. У пухлині, видаленій з черевної порожнини, виявлене волосся і зуби. До яких пухлин за гістогенезом віднесено цей утвір?

A *Тератом

B Мезенхімальних

C Ембріональних

D Епітеліальних

E Органоспецифічних

62. В матці, видаленій оперативним шляхом, виявлена щільна пухлина, яка має чіткі межі, псевдокапсулу і тканинний атипізм. Якою є ця пухлина за клінічним перебігом?

A *Доброякісна

B Диспластична

C “In situ”

D Злоякісна

E Незріла

63. При гістологічному дослідженні видаленого новоутвору шкіри знайдені гніздні скупчення і тяжі атипових плоских епітеліальних клітин, що проростають в підлеглу тканину. Ваш діагноз?

A * Плоскоклітинний рак без ороговіння

B Плоскоклітинний рак з ороговінням

C Рак на місті

D Папілома

E Аденома

64.При гістологічному дослідженні матеріалу після трансуретральної резекції простати виявлено переважання залоз різної форми та величини, утворених атиповими, гіперхромними клітинами з патологічними мітозами. Діагностуйте захворювання

A * Аденокарцинома.

B Аденома.

C Аденоматозна нодулярна гіперплазія.

D М’язово-фіброзна гіперплазія.

E Хронічний простатит.

65.У біоптаті з правого головного бронха 63-річного чоловіка, курця, виявлено пухлину, що складається з груп атипових епітеліальних клітин, які проникають поза межі базальної мембрани слизового шару, формують “гнізда” та тяжі, в центральних частинах яких розташовані концентричні, яскраво-еозинофільні маси. Діагностуйте захворювання.

A * Плоскоклітинний роговіючий рак.

B Плоскоклітинна роговіюча папілома.

C Плоскоклітинний нероговіючий рак.

D Аденокарцинома.

E Дрібноклітинний рак.

Порушення кровообігу.

1. На розтині померлого, хворівшого вадою серця виявлена збільшена в розмірі печінка строкатого виду, з малюнком мускатного горіха на розрізі. Назвіть вид порушення кровообігу

A. *загальне венозне повнокрів'я

B. Загальне артеріальне повнокрів'я

C. Недокрів'я

D. Крововилив

E. Кровотеча

2. На секції виявлено: множинні геморагічні інфаркти легень, у деяких судинах легень буруватого кольору щільні маси, які не прикріплені до стінки судин, варикозне розширення вен нижніх кінцівок, в яких наявні тромби. Про який патологічний процес йдеться?

A. * Тромбемболія судин легеневої артерії.

B. Жирова емболія судин легеневої артерії.

C. Тканинна емболія судин легеневої артерії.

D. Застійний тромбоз судин легеневої артерії.

E. Геморагічна бронхопневмонія.

3.На розтині померлого після абдомінальної операції в венах малого тазу були знайдені численні тромби. Клінічно був зафіксований тромбоемболічний синдром. Де слід шукати тромбоемболи?

A. *Легеневі артерії

B. Портальна вена

C. Лівий шлуночок серця

D. Головний мозок

E. Вени нижніх кінцівок

4. Хворий 35 років скаржиться на багаторазову блювоту, пронос, падіння АТ, серцебиття. Свій стан пов´язує з прийомом недоброякісної їжі. Дослідження показало інфікування сальмонелами. В аналізі крові - збільшення числа еритроцитів в одиниці об´єму. Який розлад кровообігу міг мати місце у даного хворого?

A. *Згущення крові

B. Гемоліз еритроцитів і компенсаторне підсилення гемопоезу

C. Загальне артеріальне повнокрів´я

D. Поліцитемія

E. Гіперхромна анемія

5.Під час емоційно напруженої роботи раптово помер молодий мужчина. На розтині виявлено нерівномірне кровонаповнення міокарда. Гістохімічно – зниження вмісту глікогену. Електронно мікроскопічно –деструкція мітохондрій, контрактури міофібрил. Вкажіть ймовірний розлад кровообігу?

A. *Гостра ішемія

B. Хронічна ішемія

C. Вакатна артеріальна гіперемія

D. Гостра венозна гіперемія

E. Ангіоневротична артеріальна гіперемія

6. Під час дорожно-транспортної пригоди водій отримав поранення в шию розбитим склом. Кровотеча була невелика, але через декілька хвилин потерпілий помер при явищах гострої ядухи. При розтині серця померлого у заповненій водою порожнині перикарду виділяються пухирці. Вкажіть ймовіринй патологічний процес.

A. *Повітряна емболія

B. Газова емболія

C. Жирова емболія

D. Тромбоемболія

E. Емболія сторонніми тілами

7. Запис в акті судово-медичного експерта про обстеження трупа потерпілого з тупою травмою обох нирок, у якого після травми спостерігалась гостра ниркова недостатність: “В обох сечоводах виявлені червоні тромби довжиною до 10 см., що обтурують простір сечоводів”. Начальник судово-медичного бюро прочитав і не погодився з цим записом. Він поступив так тому, що:

A. *В сечоводах не буває тромбів.

B. В сечоводах можуть бути лише білі тромби.

C. Це тромбоемболи.

D. Тромби в сечоводах не бувають обтуруючими.

E. В сечоводах можуть бути лише змішані тромби.

8. Хворий, якому швидко видалили біля 10 літрів асцитичної рідини з черевної порожнини, раптом втратив свідомість. Яка причина цього явища?

A. *Недокрів'я головного мозку

B. Тромбоз церебральних артерій

C. Крововилив у головний мозок

D. Вакатна гіперемія

E. Тромбоз церебральних вен

9. У хворого після відкритого перелому ключиці раптово наступила смерть. При ранньому розтині в правому шлуночку серця і легеневих артеріях виявлена піниста кров. Що стало причиною смерті?

A. *Повітряна емболія

B. Бактеріальна емболія

C. Кровотеча

D. Тканинна емболія

E. Жирова емболія

10.Чоловіка з кесонною хворобою омер з ознаками гострих порушень мозкового кровообігу в басейні A. meningea media лівої півкулі головного мозку. На розтині виявлено вогнище сірого розм'якшення мозгу зазначеній області розмірами 6х7х3,4 см. Визначити характер процесу, що викликав смерть людини.

A. *Газова емболiя.

B. Жирова емболiя.

C. Тромбоз.

D. Тромб емболiя.

E. Атеросклероз судин.

11. При розтині померлого чоловіка 73 років, який довго страждав ішемічною хворобою серця з серцевою недостатністью, знайдено: “мускатна” печінка, бура індурація легень, цианотична індурація нирок та селезінки. Вкажіть, який з видів порушення кровообігу найбільш вірогідний?

A. * Хронічне загальне венозне повнокрів’я.

B. Артеріальна гіперемія.

C. Гостре загальнне венозне повнокрів’я.

D. Гостре малокрів’я.

E. Хронічне малокрів’я.

12. Чоловік 44 років, хворий на інфаркт міокарду, помер від лівошлуночкової недостатності. При розтині знайдено:, набряк легень, дрібнокраплинні крововиливи у серозних та слизових оболонках. Мікроскопічно: дистрофічні та некробіотичні зміни епітелію проксимальних канальців нирок, у печінці – центролобулярні крововиливи та осередки некрозу. Який з видів порушення кровообігу найбільш вірогідний?

A. * Гостре загальнне венозне повнокрів’я.

B. Артеріальна гіперемія.

C. Хронічне загальне венозне повнокрів’я.

D. Гостре малокрів’я.

E. Хронічне малокрів’я.

13. При мікроскопічному дослідженні пупочної вени новонародженого, померлого від інтоксикації, знайдено: стінка судини з дифузною запальною інфільтрацією, у просвіті її – обтуруючий тромб з великою кількістю нейтрофільних лейкоцитів з явищами каріорексису та колоніями бактерій. Який з наслідків тромбу найбільш вірогідний?

A. * Септичний аутоліз.

B. Асептичний аутоліз.

C. Організація та каналізація тромбу.

D. Перетворення у тромбоембол.

E. Петрифікація тромбу.

14. У хворої 65 років з тромбофлебітом глибоких вен гомілки в поліклініці, в черзі на прийом до лікаря, раптово наступила смерть. На розтині трупа в загальному стовбурі і біфуркації легеневої артерії знайдені вільно лежачі червоні рихлі маси з тьмяною гофрованою поверхнею. Який патологічний процес в легеневій артерії знайшов патологоанатом?

A. *Тромбоемболію

B. Тромбоз

C. Тканинну емболію

D. Емболію сторонніми тілами

E. Жирову емболію

15. На розтині померлого виявлено, що печінка збільшена в розмірах, щільна, краї закруглені, на розрізі тканина жовтувато-коричневого кольору з темно-червоними крапками та смужками, що нагадує малюнок мускатного горіху. Який патологічний процес лежить в основі таких змін печінки?

A. *Хронічне венозне повнокрів’я

B. Гостре венозне повнокрів'я

C. Артеріальне повнокрів’я

D. Артеріальне недокрів’я

E. Хронічна кровотеча

16. Хворий 70 років, який страждав цукровим діабетом та переніс інфаркт міокарда, помер при явищах прогресуючої серцево-судинної недостатності. На розтині – ціанотична індурація селезінки та нирок, бура індурація легень та мускатна печінка. Який вид порушення кровообігу обумовив зміни внутрішніх органів?

A. *Загальна хронічна венозна гіперемія

B. Загальна гостра венозна гіперемія

C. Загальна артеріальна гіперемія після анемії

D. Артеріальна ішемія в резульаті перерозподілу крові

E. Місцева хронічна венозна гіперемія

17.Хворий, який страждав ішемічною хворобою серця і переніс повторний інфаркт міокарда, помер при явищах прогресуючої серцево-судинної недостатності. На розтині виявлена збільшена щільна селезінка, темно-вишневого кольору на розрізі. При мікроскопічному дослідженні органа встановлені склероз пульпи і атрофія фолікулів. Яким терміном визначають зміни?

A. *Ціанотична індурація селезінки

B. “Сагова” селезінка

C. “Сальна” селезінка

D. Порфирова селезінка

E. Септична селезінка

18. У хворого 29 років з діагнозом: багатооскольчастий перелом правого стегна на 3 добу від одержання травми з’явилися скарги на біль у грудній порожнині зправа, утруднене дихання. Через добу на фоні прогресуючої серцево-дихальної недостатності настала смерть. При гістологічному дослідженні у кровоносних судинах легень та головного мозку виявлені суданофільні краплини оранжевого кольору, які повністю перекривали просвіти судин мікроциркуляторного руслу. З яким ускладненням пов’язана смерть хворого?

A. * Жировою емболією

B. Газовою емболією

C. Медикаментозною емболією

D. Мікробною емболією

E. Тромбоемболією

19.У льотчика, який загинув внаслідок розгерметизації кабіни літака. При гістологічному дослідженні внутрішніх органів у судинах виявлено велику кількість пухирців, у печінці – жирова дистрофія. В головному і спинному мозку – множинні дрібні ішемічні осередки сірого розм’якшення. Вкажіть найбільш ймовірну причину таких змін.

A. *Газова емболія

B. Повітряна емболія

C. Жирова емболія

D. Тромбоемболія

E. Тканинна емболія

20. У хворого 75 років, який довгий час страждав на атеросклероз церебральних судин, на аутопсії у правій тім’яно-скроневої ділянці головного мозку виявлено вогнище неправильної форми, в’ялої консистенції, сірого кольору. Яка найбільш вірогідна причина розвитку цього процесу?

A. *Тромбоз правої середньої мозкової артерії

B. Тромбоз правої передньої мозкової артерії

C. Тромбоз правої задньої мозкової артерії

D. Тромбоз базилярної артерії

E. Тромбоз судини м’якої мозкової оболонки

21. У хворого з цирозом печінки, після видалення з черевної порожнини 10 літрів асцитичної рідини розвинувся колапс і гіперемія очеревини. Визначте вид артеріальної гіперемії очеревини.

A. * Гіперемія після анемії

B. Запальна

C. Вакатна

D. Колатеральна

E. Внаслідок шунтування крові

22. На секції тіла жінки 76 років були знайдені ознаки хронічного бронхіту та легеневого серця, печінка збільшена в розмірах, щільна, тканина на розрізі строката. Мікроскопово – просвіти центральних вен печінкових часточок дилятовані, синусоїди центрів часточок повнокрівні, центри часточок склерозовані, а гепатоцити мають ознаки дистрофії, некрозу та атрофії; на периферії часточок гепатоцити збережені. Який вид розладу кровообігу викликав зміни печінки?

A * Загальна венозна гіперемія.

B Місцева венозна гіперемія.

C Загальна артеріальна гіперемія.

D Місцева артеріальні гіперемія.

E Загальне малокрів’я.

23. У хворого на тромбофлебіт нижніх кінцівок раптово виникла задишка, різкий біль у грудях, ціаноз, набухання шийних вен. Яке найбільш імовірне порушення кровообігу виникло у хворого?

A * Тромбоемболія легеневої артерії

B Тромбоемболія судин головного мозку

C Тромбоемболія вінцевих судин

D Тромбоемболія мезентеріальних судин

E Тромбоемболія ворітної вени

24. Після перенесенного ДТП у потерпілого: ниткоподібний пульс, стійка гіпотонія, часте та поверхневе дихання, блідість шкіри, холодний піт, свідомість - сопор. Яка стадія шоку наймовірніше розвинулась у хворого?

A * Торпідна.

B Термінальна

C Еректильна

D Предшок

E Агонія

25. Хворий скаржиться на задишку в спокої, запаморочення в голові. Акроціаноз, частота дихання 24 в хв. Кров: еритропенія, гіпогемоглобіноз, гіпохромія, мікроанізоцитоз, пойкілоцитоз. Про який вид гіпоксії можна думати?

A * Гемічна гіпоксія

B Гіпоксична гіпоксія

C Циркуляторна гіпоксія

D Гістотоксична гіпоксія

E Тканинна гіпоксія

26. У хворого із емболією легеневої артерії розвинувся респіраторний алкалоз. Що є безпосередньою причиною цього?

A * Гіпервентиляція

B Гіповентиляція

C Гіперкапнія

D Артеріальна гіпоксемія

E Брадипное

27. Що є проявом ДВС – синдрому?

A *Утворення кров’яних згустків внутрішньосудинно

B Анемія

C Лейкоцитоз

D Еритроцитоз

E Дефіцит протромбіну

Запалення.

1.Слизова оболонка товстої кишки у померлого при дизентерії на розтині повнокровна, покрита плівкою сірого кольору, що відривається із зусиллям. Який вид запалення розвинувся в кишці у хворого?

A. *Дифтеритичне запалення

B. Крупозне запалення

C. Геморагічне запалення

D. Серозне запалення

E. Катаральне запалення

2. При розтині померлого хворого від хронічної ниркової недостатності в слизуватій оболонці товстої кишки виявлені сіро-жовті, щільно з'єднані плівки, які відокремлюються з утворенням виразок. Уточніть вид запалення.

A. *Дифтеритичне

B. Серозне

C. Катаральне

D. Крупозне

E. Гнійне

3.Чоловік віком 55 років тривалий час хворів на хронічний гломерулонефрит. Помер при явищах хронічної ниркової недостатності.. На поверхні епікарда і перикарда виявляються сірувато-білуваті ворсинчасті нашарування. Який процес патологічний процес мав місце в перикарді?

A. *фібринозне запалення

B. організація

C. проліферативне запалення

D. геморагічне запалення

E. артеріальне повнокрів’я

4. Дівчинка віком 6 років захворіла на дифтерію і померла на третю добу від асфіксії На аутопсії слизова оболонка трахеї і бронхів стовщена, набрякла, тьмяна, покрита сіруватими плівками, які легко відокремлюються. Про який вид запалення свідчать морфологічні зміни.

A. *крупозне запалення

B. Серозне запалення

C. Геморагічне запалення

D. Дифтеритичне запалення

E. Катаральне запалення

5. Хворий 67 років мав важку форму грипу з летальним наслідком. На секції зміни в легенях були подібні змінам “великих строкатих легень”. При мікроскопічному дослідженні виявлено: різке повнокрів’я судин, крововиливи, набряк легеневої тканини, в просвіті бронхів і альвеол ексудат, який містить переважно еритроцити. Про який характер запалення легень свідчать ці морфологічні ознаки?

A. * Геморагічна бронхопневмонія.

B. Катаральна бронхопневмонія.

C. Гнійна бронхопневмонія.

D. Десквамативна бронхопневмонія.

E. Фібрінозна пневмонія.

6.У хворго після операційного втручання (опікова хвороба) в умовах різкого зниження реактивності організму став прогресувати сепсис, що привело до смертельного наслідку. В області передньої стінки черевно порожнини на розтині мікроскопічно спостерігається дифузна інфільтрація сегментоядерними лейкоцитами міжм'язових проміжків, набряк тканини, лізис м'язових волокон. Визначити характер патологічного процесу.

A. *Флегмона.

B. Дифтеритичне запалення.

C. Абсцес.

D. Некроз.

E. Катаральне запалення.

7. На розтині померлого 34 років від ревматизму т - поверхня епікарду ворсиста, покрита плівками сірого кольору, що легко відділяються. Після їх відділення визначається набрякла повнокровна поверхня епікарду. Найвірогідніший діагноз:

A. *фібринозий перикардит

B. гнійний перикардит

C. геморагічний перикардит

D. проліферативний перікардит

E. катаральний перикардит

8. На розтині у хворого в черевній порожнині виявлено близько 2.0 л гнійної рідини. Очеревина тьмяна, з сіруватим відтінком, на серозній оболонці кишки сіруватого кольору нашарування, що легко знімаються. Найвірогідніше це:

A. * фібринозно-гнійний перитоніт

B. геморагічний перитоніт

C. серозний перитоніт

D. туберкульозний перитоніт

9. При мікроскопічному дослідженні міокарду чоловіка, померлого від серцевої декомпенсації, виявлено склероз перісудинної сполучної тканини та дифузну інфільтрацію її лімфоцитами, макрофагами, плазмоцитами та поодинокими нейтрофілами. Вкажіть, який з перелічених видів запалення найбільш ймовірний.

A. * Інтерстиційне продуктивне.

B. Гранульоматозне.

C. Альтеративне.

D. Ексудртивне дифузне.

E. Ексудативне осередкове.

10.При мікроскопічному дослідженні біопсії нирки виявлено вогнища, в центрі яких знаходяться зернисті еозинофільні маси, оточені інфільтратом з лімфоцитів, епітеліоїдних клітин та поодиноких клітин Пирогова-Лангханса. Виберіть патологічний процес, що найбільш повно відповідає описаним змінам.

A. * Гранулематозне запалення

B. Коагуляційний некроз

C. Казеозний некроз

D. Альтеративне запалення

E. Проліферація та диференціювання макрофагів

11.У хворого 28 років мигдалики значно збільшені, повнокровні, болючі, на їх поверхні – щільні брудно-сірі плівки, які поширюються на тверде піднебіння, щільно зв’язані з підлеглими тканинами, при спробі відокремити їх – розвивається кровотеча. Який патологічний процес обумовлює дані морфологічні зміни?

A. *Дифтеритичне ексудативне запалення.

B. Крупозне ексудативне запалення.

C. Катаральне ексудативне запалення.

D. Гнійне ексудативне запалення.

E. Геморрагічне ексудативне запалення.

12. У хворого, померлого від серцевої недостатності, при патоморфологічному дослідженні виявлено: стулки мітрального клапана деформовані, потовщені, зрослися по краях; у сполучній тканині міокарда - дифузно розкидані вузлики, які складаються з ділянок фібриноїдного некрозу, навколо яких скупчуються макрофагоцити, що нагадують гігантські багатоядерні клітини. Подібні вогнища оточені лімфоцитами та поодинокими плазматичними клітинами.. Яка з перерахованих гранульом має місце у даного хворого?

A. * Ревматична.

B. Туберкульозна.

C. Актиномікотична.

D. Сифілітична.

E. Лепрозна

13. Хворий хронічним пієлонефритом помер від хронічної ниркової недостатності. При житті аускультативно відмічено феномен “шум тертя перикарду”. На розтині виявлено, що епікард тьм’яний, шорсткий, ніби покритий волосяним покривом. Який перикардит за характером запалення має місце?

A. *Крупозний

B. Дифтеритичний

C. Гнійний

D. Гнильний

E. Серозний

14.При розтині тіла дівчинки, яка померла від асфіксії, виявлено, що слизова трахеї і бронхів покрита біло-сірою плівкою, яка рихло з’єднана з підлеглими тканинами і легко знімається пінцетом. Просвіт сегментарних бронхів виповнений рихлими масами сіро-білого кольору. Який трахеобронхіт за характером ексудату відмічений при розтині?

A. *Крупозний

B. Катаральний

C. Дифтеритичний

D. Гнійний

E.Гнильний

15.Слизова оболонка трахеї тьмяна, повнокровна, з нашаруванням сіро-білих плівок. Найвірогідніша форма запалення:

A. *фібринозне

B. гнійне

C. серозне

D. проліферативне

E. змішане

16. Губчастий та кортикальний шари гомілкової кістки на окремих ділянках у стані розпаду, порожниниі, що створилися, заповнені вершкоподібними масами зеленувато-жовтого кольору. Найвірогідніша форма запалення:

A. *гнійне

B. катаральне

C. серозне

D. проліферативне

E. змішане

17.У розтині у печінці померлого 62 років виявлено вогнище розпаду тканини діаметром 4 см, заповнене рідиною жовтувато-зеленого кольору. Найвірогідніший діагноз:

A. *абсцес

B. флегмона

C. карбункул

D. емпієма

E. гранульома

18.В біоптаті слизової носа знайдені епітеліоїдні клітини, плазмоцити, клітини Мікуліча, еозинофільні тільця Русселя. Ваш діагноз:

A. *Ріносклерома

B. Сифіліс

C. Туберкульоз

D. Респіраторносинцитіальна інфекція

E. Алергічний риніт

19. Гістологічно в апендиксі в усіх шарах знайдені в значній кількості поліморфноядерні лейкоцити, повнокрів’я, стази. Така картина є характерною для:

A. *Флегмонозного апендициту

B. Гангренозного апендициту

C. Поверхневого апендициту

D. Простого апендициту

E. Хронічного апендициту

20.При розтині хворого, який помер від крупозної пневмонії, в плевральній порожнині непрозора рідина, на вісцеральній плеврі сіруватого кольору плівка. Визначити вид запалення на вісцеральнiй плеврi.

A. *Фiбринозне

B. Катаральне

C. Гнiйне

D. Гранульоматозне

E. Геморагiчне

21.У дитини підвищилась температура тіла, з’явились болі при ковтанні. При огляді піднебінні мигдалики збільшені, темно-червоні, вкриті сіро-жовтими плівками, щільно спаяними з поверхнею мигдаликів. Яке запалення розвилося в мигдаликах?

A. * Дифтеритичне

B. Крупозне

C. Геморагічне

D. Гнійне

E. Катаральне

22. У хворого з перитонітом виявлено в черевній порожнині 200 мл в´язкої жовто-зеленої рідини. Визначте форму ексудативного запалення

A. *Гнійне

B. Серозне

C. Фібринозне

D. Геморагічне

E. Гнилісне

23. У хворого після термічного опіку в шкірі сформувалися болючі пухирі, заповнені каламутноватою рідиною. Яка морфологічна форма запалення виникла у хворого?

A. *Серозне

B. Продуктивне

C. Крупозне

D. Гранульоматозне

E. Дифтеритичне

24. При гістологічному дослідженні біоптату шкіри у хворого 24 років виявлено казеозний некроз, оточений клітинним інфільтратом з лімфоцитів, серед яких зустрічаються окремі велетенські клітини, має місце розростання сполучної тканини, ендоваскуліти. Який характер патологічного процесу?

A. *Продуктивне гранульоматозне запалення.

B. Продуктивне iнтерстицiальне запалення.

C. Абсцес.

D. Катаральне запалення.

E. Iхорозне запалення.

25. У хворого зі скаргами на задуху проведена біопсія слизової оболонки носової порожнини. Встановлено дiагноз: риносклерома. Якi клiтини типовi для даного захворювання при мiкроскопiчному дослiдженi?

A. *Клiтини Мiкулiча

B. Клiтини Пирогова-Лангханса

C. Плазмоцити

D. Лімфоцити

E. Тiльця Шаумана

26. При розтині чоловіка 60 років у легенях та печінці знайдено багато білуватих вузликів розміром з пшонину. Мікроскопічно виявлені гранульоми з осередком некрозу у центрі, по периферії його - епітеліоїдні, лімфоїдні, плазматичні клітини, а також макрофаги і велика кількість клітин Пирогова-Лангханса, які переважають у інфільтратах. Яка з перелічених гранульом має місце?

A. *Гігантоклітинна

B. Макрофагальна

C. Фагоцитомна

D. Епітеліоїдно-клітинна

E. Гранульома чужорідних тіл

27. При розтині чоловіка 60 років у легенях та печінці знайдено багато білуватих вузликів розміром з пшонину. Мікроскопічно виявлені гранульоми з осередком некрозу у центрі, по периферії його - епітеліоїдні, лімфоїдні, плазматичні клітини, а також макрофаги і велика кількість клітин Пирогова-Лангханса, які переважають у інфільтратах. Яка з перелічених гранульом має місце?

A. *Гігантоклітинна

B. Макрофагальна

C. Фагоцитомна

D. Епітеліоїдно-клітинна

E. Гранульома чужорідних тіл

28. При огляді зіву у хворого на ангіну визначається гіперемія слизової оболонки піднебіння, мигдалики збільшені в розмірах, червоні, на їх поверхні помітні дрібні біло-жовті осередки. Який клініко-морфологічний варіант ангіни найбільш імовірний у даному випадку?

A. *Лакунарна

B. Катаральна

C. Гнійна

D. Фібринозна

E. Некротична

29. Дівчинка 4 років на 3 добу від початку захворювання на дифтерію померла від справжнього крупу. На аутопсії слизова оболонка гортані, трахеї та бронхів потовщена, набрякла, тьмяна, вкрита сіруватою плівкою, що легко відокремлюється. Визначити вид ексудативного запалення гортані

A. *Фібринозне

B. Серозне

C. Гнійне

D. Змішане

E. Катаральне

30.При розтині чоловіка 28 років, який хворів підгострим гломерулонефритом і помер при явищах прогресуючої уремії, на поверхні перикарду, вісцеральному та пристінковому листках плеври, очеревини спостерігалися нашарування у вигляді білуватих ниток, які легко відокремлювалися від підлеглої тканини. Вказати різновид запалення в серозних оболонках.

A. * Крупозне фібринозне запалення

B. Дифтеритичне фібринозне запалення

C. Гнійне запалення

D. Дифузне проліферативне запалення

E. Серозне запалення

31. На слизовій оболонці мигдаликів та м’якого піднебіння виявляються білувато-сірого кольору плівки, які щільно з’єднані з підлеглою тканиною, при спробі зняти плівку на її місці виникає глибокий дефект тканини. Визначити патологічний процес, який виник на слизовій оболонці мигдаликів та м’якого піднебіння

A. *Дифтеритичне запалення

B. Серозне запаленя

C. Крупозне запалення

D. Гнійне запалення

E. Змішане запалення

32. У померлого від гострого трансмурального інфаркту міокарда на розтині на поверхні перикарда виявлено: ниткоподібні відкладення білувато-коричневого кольору, які з’єднували парієтальний та вісцеральний листки перикарда між собою. Вказати різновид запалення в перикарді?

A. *Крупозне запалення

B. Дифтеритичне запалення

C. Серозне запалення

D. Гнійне запалення

E. Гранульоматозне запалення

33. У жінки віком 34 років після наобережного поводження з праскою на правому вказівному пальці з’явився різкий біль, почервоніння, припухлість. Через кілька хвилин виник міхур, заповнений прозорою рідиною солом’яно-жовтого кольору. Проявом якого патологічного процесу є описані зміни?

A. *Ексудативного запалення

B. Травматичного набряку

C. Альтеративного запалення

D. Проліферативного запалення

E. Вакуольної дистрофії

34.Хворий віком 34 роки, звернувся до лікаря зі скаргами на локальний біль в області потилиці, підвищення температури тіла у цій ділянці. Макроскопічно визначається інфільтрат конусоподібної форми багряно-синюшного кольору з жовтувато-зеленуватою верхівкою, яка піднімається над поверхнею шкіри. Поставте діагноз.

A. *Фурункул

B. Флегмона

C. Абсцес

D. Карбункул

E. Емпієма

35. Дівчинка віком 5 років захворіла на дифтерію. На третю добу померла від асфіксії внаслідок справжнього крупу. На розтині встановлено, що слизова оболонка гортані, трахеї та бронхів стовщена, набрякла, тьмяна, покрита сіруватими плівками, які легко відокремлюються. Про який патологічний процес свідчать морфологічні зміни у гортані?

A. *Крупозне запалення

B. Серозне запалення

C. Гнійне запалення

D. Дифтеритичне запалення

E. Катаральне запалення

36.При мікроскопічному дослідженні легень хворого 52 р. виявлені вогнища некрозу, оточені валом епітеліоїдних клітин та лімфоцитів. Між лімфоцитами та епітеліоїдними клітинами розміщені великі клітини округлої форми з великою кількістю ядер, розміщених на периферії. Як називається виявлене утворення?

A. * Туберкульозна гранульома

B. Саркоїдозна гранулема

C. Сифілітична гранульома

D. Лепрозна гранулема

37. При огляді дитини 7 р., яка поступила в інфекційне відділення зі скаргами на різкий біль у горлі, затруднення при ковтанні, підвищення температури тіла до 390С, набряк шиї виявлено: мигдалики збільшені, їх слизова оболонка повнокрівна, вкрита великою кількістю плівок білувато-жовтого кольру, які щільно прилягають до слизової оболонки. При спробі зніти плівку залишається глибокий дефект, який кровоточить. Який вид запалення має місце?

A. * Дифтеритичне

B. Гнійне

C. Серозне

D. Крупозне

E. Геморагічне

38. 50-річний хворий протягом 10 років хворів на мембранозно-проліферативний гломерулонефрит і постійно отримував сеанси гемодіалізу з метою корекції ниркової недостатності. Останнього півроку уникав лікування. Доставлений у відділення гемодіалізу у вкрай важкому стані, без свідомості, із запахом сечовини від тіла, набряками, ознаками вираженого плевриту, перикардиту та перитоніту, що було розцінено як уремія. Який вид запалення найбільш вірогідний?

A. *Фібринозне

B. Гнійне

C. Катаральне

D. Геморагічне

E.Серозне

39. Чоловік скаржиться на свербіж та почервоніння шкіри у ділянці щоки, котре з’явилося після гоління. Об’єктивно: на ділянці гіперемованої шкіри щоки виявлені пухирці, заповненні прозорою рідиною. Який характер рідини у пухирцях?

A. *Серозний ексудат

B. Трансудат

C. Гнійний ексудат

D. Слизовий ексудат

E. Геморагічний ексудат

40.Під час розтину дитини, яка померла при ознаках асфіксії, були виявлені в трахеї і бокових бронхах фібринозні плівки, які вільно лежали в просвіті дихальних шляхів, нагадуючи їх зліпки. Вкажіть вид запалення.

A *Крупозне

B Дифтеритичне

C Катаральне

D Змішане

E Дифтерійне

41.У 5-річної дитини, доставленої в лікарню в стані асфіксії, в гортані виявлені біло-жовті фібринозні плівки, які з великим зусиллям знімаються з утворенням виразок. Вкажіть вид запалення.

A *Дифтеритичне

B Катаральне

C Змішане

D Крупозне

E Дифтерійне

42.У дитини, що хворіла дифтерією і померла від гострої серцевої недостатності, на розтині на слизовій оболонці зіву і мигдаликів знайдені плівки сірого кольору, щільно з´єднані з підлеглими тканинами, при спробі зняти плівку утворюється глибокий виразковий дефект. Який вид ексудативного запалення в зіві і мигдаликах?

A *Дифтеритичне

B Крупозне

C Гнійне

D Катаральне

E Геморагічне

43.При гістологічному дослідженні тканини легень виявлено порожнину, виповнену лейкоцитами, стінки якої представлені грануляційною тканиною, іззовні оточеною фібротизованою сполучною тканиною. Діагностуйте патологічний процес.

A * Гнійне запалення.

B Фібринозне запалення.

C Кістоутворення.

D Гематома.

E Пухлинний ріст.

44.На автопсії тіла чоловіка 43 років, який помер від серцево-легеневої недостатності, знайдено у нижній частці правої легені порожнину діаметром 3 см, заповнену тягучим зелено-сірим вмістом. При гістологічному дослідженні стінка описаного утвору представлена сполучною та молодою грануляційною тканиною, в просвіті - нейтрофільні лейкоцити та продукти їх розпаду. Діагностуйте вид запалення.

A * Хронічний абсцес.

B Гострий абсцес.

C Емпієма.

D Фурункул.

E Карбункул.

Імунопатологічні процеси.

1. При гістологічному дослідженні діагностичної біопсії лівого вушка серця виявлено ревматичну гранулему, яка побудована із вогнища фібриноїдного некрозу, навколо якого розміщені базофільні макрофаги, лімфоцити, фібробласти і поодинокі плазмоцити. Вкажіть ймовірний тип запальної реакції на імунній основі.

A. * Гіперчутливість сповільненого типу

B. Нормергічний тип

C. Гіперчутливість негайного типу

D. Вроджений імунодефіцит

E. Набутий імунодефіцит

2. Дитина 3 років з множинними порушеннями розвитку кісток лицевого відділу черепа. Причина смерті – сепсис, який розвився на фоні бронхопневмонії. В крові вміст імуноглобулінів в межах фізіологічної норми. На розтині встановлена відсутність тимусу. Назвати головну причину виявлених змін.

A. *Синдром недостатності клітинного імунітету

B. Синдром комбінованого імунодефіциту

C. Вторинний імунодефіцитний синдром

D. Гострий лімфолейкоз

E. Синдром хронічної інтоксикації

3. У хворої бронхіальною астмою вірусне інфікування спровокувало астматичний статус зі смертельним наслідком. При гістологічному дослідженні легень виявлено спазм і набрякання бронхіол, в їх стінках виражена інфільтрація лімфоцитами, еозинофілами і іншими лейкоцитами, а також дегрануляція лаброцитів. Який механізм гіперчутливості лежить в основі виявлених змін?

A. *Реагінова реакція гіперчутливості

B. Запальний

C. Аутоімунний

D. Імунокомплексний

E. Імуннозумовлений клітинний цитоліз

4. Біля інфікованої рани збільшилися регіонарні лімфовузли. При гістологічному дослідженні в них виявлено збільшення кількості макрофагів, лімфоцитів і лімфатич-них фолікулів в кірковому шарі, а також велику кількість плазматичних клітин. Який процес в лімфатичних вузлах відображають виявлені гістологічні зміни?

A. *Антигенну стимуляцію

B. Набуту недостатність лімфоїдної тканини

C. Природжену недостатність лімфоїдної тканини

D. Пухлинну трансформацію

E. Реакцію гіперчутливості

5. При розтині дитини віком 6 міс., померлої від сепсису, виявлена відсутність вилочкової залози, зменшення розмірів та маси селезінки. При мікроскопічному дослідженні селезінки виявилася відсутність периартеріальних Т-залежних зон фолікулів зі спустошенням червоної пульпи; у лімфатичних вузлах - відсутність перикортикальної зони, яка в основному представлена Т-лімфоцитами. В-зони у периферійних імунних органах розвинуті нормально. Який патологічний процес иає місце?

A. *Синдром Дайджорджи (недостатність клітинного імунітету)

B. Синдром Гланцмана-Рінікера (недостатність клітинного та гуморального імунітету)

C. Синдром Брутона (недостатність гуморального імунітету)

D. Акцидентальна інволюція тимусу

E. ВIЛ-інфекція

6. При пункційній біопсії в трансплантованій нирці виявлена дифузна інфільтрація строми лімфоцитами, плазмоцитами, лімфобластами, плазмобластами, а також некротичний артеріїт. Який патологічний процес розвинувся у трансплантаті?

A. *Імунне відторгнення

B. Гломерулонефрит

C. Iшемічне пошкодження нирки

D. Тубулонекроз

E. Пієлонефрит

7. Чотирьохрічній дитині зроблена реакція Манту. Через 60 годин після введення у шкіру туберкуліну з’явилося вогнищеве затвердіння і почервоніння шкіри, діаметром 15 мм, що було розцінено як позитивний тест. Який вид реакції гіперчутливості лежить в основі цього тесту?

A. * Гіперчутливость сповільненого типу

B. Імунокомплекс-опосередкована гіперчутливість

C. Комплімент-опосередкована цитотоксична

D. Гіперчутливість негайного типуІ

8. 14-річна інфантильна дівчинка раптово померла після перенесеної гострої респіраторної вірусної інфекції. Під час розтину виявлено наступні зміни: різко збільшений в розмірах тимус, генералізовану гіперплазію лімфатичних вузлів, гіпоплазовані яєчники.Діагностуйте захворювання.

A. *Тиміко-лімфатичний стан

B. Акцидентальна інволюція тимуса

C. Гіпоплазія тимуса

D. Атрофія тимуса

E. Тимома

9.Смерть семирічного хлопчика наступила внаслідок гострої постгеморагічнї анемії, зумовленою профузною кровотечею із шлунково-кишкового тракту. В ході патологоанатомічного дослідження виявлено: макроскопічно – малокрів’я внутрішніх органів, збільшення різних груп лімфатичних вузлів, тимомегалія, помірно виражена гепато-спленомегалія, яскраво-червоний кістковий мозок; мікроскопічно – гіперцелюлярний кістковий мозок з мономорфним інфільтратом з бластних клітин, дифузно-вогнищеві пухлинні інфільтрати в печінці, селезінці, лімфатичних вузлах, оболонках та речовині головного мозку. Діагностуйте захворювання.

A. *Гострий лімфобластний лейкоз

B. Гострий міелобластний лейкоз

C. Гострий недиференційований лейкоз

D. Гострий монобластний лейкоз

E. Гострий плазмобластний лейкоз

10. При обстеженні вагітної жінки з резус-негативною групою крові знайдено високий рівень антиеритроцитарних антитіл, для зниження якого їй було підшито шкіряний лоскут її резус-позитивного чоловіка. Через 2 тижні лоскут відторгся, мікроскопічно в ньому знайдено порушення кровообігу, набряк, клітинну інфільтрацію перважно лімфоцитами, нейтрофілами та макрофагами. Який з перелічених патологічних процесів найбільш вірогідний?

A. *Трансплантаційний імунітет

B. Реакція гіперчутливості негайного типу

C. Реакція гіперчутливості сповільненого типу

D. Гранульматозне запалення

E. Інтерстиційне запалення

11. При дослідженні тимуса дитини 5 років, що померла від гострої деструктивної стафілококової пневмонії, виявлено зменшення маси залози до 3,0 г. При гістологічному дослідженні в тимусі найдено: зменшення дольок залози, значне зменшення кількості лімфоцитів з колапсом строми, інверсія шарів, кістоподібне збільшення тілець Гасаля. Який з перелічених діагнозів найбільш вірогідний?

A. *Акцидентальна інволюція

B. Тимомегалія

C. Гіпоплазія тимусу

D. Дисплазія тимусу

E. Агненезія тимусу

12. Експериментальній тварині після попередньої сенсибілізації підшкірно введено дозу антигену. У місці ін?єкції розвинулось фібринозне запалення з альтерацією стінок судин, основної речовини та волокнистих структур сполучної тканини у вигляді мукоїдного на фібриноїдного набухання, фібриноїдного некрозу. Яка імунологічна реакція має місце?

A. *Гіперчутливість негайного типу

B. Гіперчутливість сповільненого типу

C. Реакція трансплантаційного імунітету

D. Нормергічна реакція

E. Гранульоматоз

13. У померлого від задухи чоловіка, який багато років страждав на бронхіальну астму, при гістологічному дослідженні легень виявлено: в просвітах бронхіол та дрібних бронхів багато слизу з домішкою еозинофілів, склероз міжальвеолярних перегородок, розширення просвітів альвеол. Який з механізмів розвитку реакції гіперчутливості має місце?

A. *Реагінова реакція

B. Цитотоксична реакція

C. Імунокомплексна реакція

D. Цитоліз, обумовлений лімфоцитами

E. Гранульоматоз

14. Хворому 39 років у зв’язку із лікуванням гнійного пульпіту була проведена інфільтраційна анестезія розчином ультракаїну з адреналіном. Після чого раптово виникли почервоніння, набряк шкіри з пухирцями та свербіж. Який тип гіперчутливості має місце у хворого?

A. *Анафілактичний

B. Цитотоксичний

C. Імунокомплексне пошкодження

D. Гіперчутливість сповільненого типу

E. Грануломатозний

15. У хворого з грибковим ураженням шкіри виявлено порушення клітинного імунітету. Зниження яких показників найбільш характерні при цьому?

A * Т-лімфоцитів

B Імуноглобулінів G

C Імуноглобулінів E

D В-лімфоцитів

E Плазмоцитів

16. Після пересадки нирки хворому проведено курс гормонотерапії. Які гормони були застосовані?

A * Глюкокортикоїди

B Мінералокортикоїди

C Катехоламіни

D Статеві

E Тиреоїдні

17. У резус негативної вагітної жінки можливий імунний конфлікт між мамою і плодом. Наявність яких з перерахованих імуноглобулінів може привести до цього явища?

A *Ig G

B Ig М

C Ig А

D Ig Е

E Ig Д

18. У хворої при обстеженні діагностовано аутоімунний тиретоксикоз. Для цього захворювання характерний такий тип алергічної реакції за Кумбсом і Джелелом:

A *Стимулюючий

B Цитотоксичний

C Імунокомплексний

D Анафілактичний

E Стимулюючий

Пристосування і компенсація (адаптація).

1. Хворий, що страждав раком шлунка, помер внаслідок ракової кахексії. На розтині трупа були знайдені характерні зміни в серці. Як назвати таке серце?

A. *Буре атрофічне

B. Волосате

C. Панцирне

D. Тигрове

E. Волове

2. Хворий, що страждав хронічною обструктивною емфіземою легень, помер від легенево-серцевої недостатності. Які зміни можна виявити в серці?

A. *Гіпертрофію правого шлуночка серця

B. Гіпертрофію лівого шлуночка серця

C. Амілоїдоз

D. Великоосередковий кардіосклероз

E. Розрив серця

3. Підліток скаржиться на потоншення м’язів і зменшення об’єму гомілки, які виникли після тривало незагоювавшогося перелому стегнової кістки без пошкодження нервів. Як називається така атрофія м’язів.

A. *Дисфункціональна

B. Нейротична

C. Визвана недостатнім кровопостачанням

D. Визвана здавлюванням

E. Від дії фізичних факторів

4. У хворої, що страждала вторинним сифілісом, з'явилися вогнища депігментації шкіри у верхніх відділах спини. Назвіть патологічний процес у шкірі.

A. *Лейкодерма

B. Метаплазія

C. Лейкоплакія

D. Дисплазія

E. Пара кератоз

5. У померлої жінки, 86 років, яка страждала на атеросклероз судин головного мозку, на розтині виявлена атрофія кори головного мозку. Як називається ця атрофія відносно причини?

A. * Від недостатнього кровопостачання.

B. Від тиску.

C. Від дії фізичних та хімічних факторів.

D. Нейротична.

E. Дисфункціональна.

6. В анамнезі у хворого 62 років з дитинства хронічні запальні хвороби легень. Харкотиння скупе, біле. Хворий потрапив до лікарні з ознаками легеневої та серцевої недостатності і при наростанні останньої помер. Які зміни в серці було знайдено на розтині?

A. *Гіпертрофію та дилятацію правого шлуночка

B. Гіпертрофію та дилятацію лівого шлуночка

C. Гіпертрофію обох шлуночків

D. Дилятацію обох шлуночків

E. Серце без макроскопічних змін

7. При гістологічному дослідженні біоптату слизової оболонки прямої кишки встановлено розростання сполучної тканини та залоз.Макроскопічно: дрібні вузли на тонкій ніжці. Про який процес йде мова?

A. *Гиперпластичний поліп.

B. Гiпертрофiя.

C. Метаплазія.

D. Атрофiя.

E. Склероз.

8. У хворого, який помер в результаті легенево-серцевої недостатності серце збільшене в розмірах, стінка правого шлуночку на розтині потовщена, порожнина розширена. Визначити характер патологічног процесу.

A. *Гiпертрофiя.

B. Гiперпластичні розростання запальної природи.

C. Метаплазія.

D. Атрофiя.

E. Склероз.

9. У хворого – глибока рвана рана із нерівними краями, вкрита гноєм. У крайових відділах – сочна грануляційна тканина, яка не здіймається над рівнем рани. Назвіть вид загоєння рани.

A. *Загоювання вторинним натягом.

B. Загоювання первинним натягом.

C. Загоювання під струпом.

D. Безпосереднє закриття дефекту епітеліальної тканини.

E. Організація рани.

10. На розтині чоловіка, який помер від хроніосепсису, виявлено атрофію скелетних м’язів, буру атрофію міокарда, печінки. Порушення обміну якого пігменту виявлено у померлого?

A. *Ліпофусцину

B. Ліпохрому

C. Гемосидерину

D. Гемомеланіну

E. Меланіну

11. На розтині у чоловіка 73 років виявлено збільшену, м’яку, еластичну, злегка горбкувату передміхурову залозу, яка на розрізі складається з окремих вузлів, розділених прошарками сполучної тканини. При мікроскопії відмічено збільшення кількості залозистих елементів. Величина часточок і кількість залозистих елементів в них - різні. Який процес має місце у померлого?

A. *Залозиста нодулярна гіперплазія

B. М’язово-фіброзна (стромальна) нодулярна гіперплазія

C. Змішана нодулярна гіперплазія

D. Аденокарцинома

E. Недиференційований рак

12. У 40-річного чоловіка із стенозуючим (без метастазів) раком стравоходу виявляються наступні зміни: атрофія скелетних м`язів, жирової клітковини. Шкіра землисто-коричневого забарвлення, епідерміс потоншаний, серце зменшене за розмірами. Міокард та печінка бурого кольору. Поставте діагноз.

A. *Аліментарна кахексія.

B. Міастенія.

C. Ракова кахексія

D. Бура атрофія.

E. Хвороба Аддісона

13. В біоптаті бронха хворого, зловживаючого палінням, в потовщеній слизовій оболонці виявлено хронічне запалення і трансформація одношарового війчастого епітелія в багатошаровий плоский епітелій. Який з процесів найбільш вірогідний?

A. *Метаплазія

B. Гіперплазія епітелія

C. Плоскоклітинний рак

D. Лейкоплакія

E. Гіпертрофія епітелія

14. У дитини 12 років, хворої поліоміелітом, соматичні м'язи слабкі, обсяг їх зменшений, шкіра суха, бліда. При морфологічному дослідженні біоптату мяких тканин виявлені характерні морфологічні зміни Визначити характер патологічного процесу мяких тканин.

A. *Атрофiя.

B. Гiпертрофiя.

C. Гiперплазія.

D. Метаплазія.

E. Гiпоплазія.

15. У хворого, який на протязі тривалого часу зловживав табакокурінням з'явився кашель з виділенням в'язкого слизу, слабкість після незначних фізичних навантажень, блідість шкіряних покровів, за останні дв місяці схуд на 12,0 кг. При ендоскопічному дослідженні біоптату діагноз: плоскоклітинний рак. Визначити характер патологічного процесу, який передував виникненню пухлини.

A. *Метаплазія.

B. Гiпоплазія.

C. Гiперплазія.

D. Некроз.

E. Склероз.

16. Хворий на рак шлунку з множинними метастазами помер від ракової кахексії. Виділіть характерні зміни серця, виявлені на розтині.

A. *Бура атрофія міокарду.

B. Амілоїдна кардіомегалія.

C. Дилятаційна кардіоміопатія.

D. Гіпертрофічна кардіоміопатія.

E. "Тигрове" серце.

17. У біоптаті слизової оболонки бронху хворого 50 років, який 20 років страждал хронічним бронхітом, знайдено витончення її, кистовидне перетворення слизових залоз, осередки заміни призматичного епітелію на багатошаровий плоский.Який з перелічених патологічних процесів найбільш вірогідний?

A. * Метаплазія.

B. Гіперплазія.

C. Гетеротопія.

D. Гетероплазія.

E. Дисплазія.

18. До патоморфологічного відділення надісланий шматочок тканини з краю хронічної виразки шлунку. При гістологічному дослідженні в ньому виявлено: некроз, грануляційна тканина, надмірний розвиток волокнистої сполучної тканини та метаплазія епітелію. Який вид компенсаторно-пристосувального процессу має місце?

A. *Спотворена регенерація з порушенням зміни фаз.

B. Гіпертрофія.

C. Фізіологічна регенерація.

D. Репаративна регенерація(субституція).

E. Репаративна регенерація(реституція).

19. Після травматичного пошкодження печінки згодом відбулося повне відновлення будови та функції печінки. Як називається такий вид регенерації?

A. *Реституція

B. Неповна регенерація

C. Патологічна регенерація

D. Звичайна фізіологічна регенерація

E. Субституція

20. У біоптаті слизової оболонки бронха в хворого на хронічний бронхіт були знайдені ділянки багатошарового плоского незроговілого епітелію. Діагностуйте патологічний процес в слизовій оболонці бронха.

A. *Метаплазія

B. Атрофія

C. Гіперплазія

D. Дисплазія

21. Жінка 45 років звернулася до гінеколога зі скаргами на ацикличні, надмірні, маткові кровотечі. При досліджені біоптату виявлено збільшення кількості залоз, кістозне її розширення. Для якого процесу характерні ці зміни?

A. *Гіперплазії ендометрію

B. Атрофія ендометрію

C. Гіпертрофії ендометрію

D. Метаплазії

E. Організації

22. При розтині трупа жінки, 69 років яка тривалий час страждала на гіпертрофічну хворобу, паталогоанатом виявив, що обидві нирки щільної консистенції, значно зменшені у розмірах, поверхня їх дрібнозерниста. Ці зміни свідчать про:

A. *Атрофію від недостатнього кровопостачання

B. Атрофію від тиску

C. Старечу атрофію

D. Атрофію дисфункціональну

E. Гіпоплазію

23. У хворого 10 років тому було видалено праву легеню з приводу пухлини, після чого об'єм лівої легені збільшився на 50%. Який процес розвинувся в лівій легені?

A. *Вікарна гіпертрофія

B. Нейрогуморальна гіпертрофія

C. Несправжня гіпертрофія

D. Робоча гіпертрофія

E. Гіпертрофічні розростання

24. При гістологічному дослідженні біопсії стінки бронха виявлено, що одношаровий миготливий епітелій заміщений на багатошаровий плоский. Який загальнопатологічний процес діагностував лікар?

A *Метаплазія

B Регенерація

C Гіпертрофія

D Організація

E Гіперплазія

25. У хворого в ділянці післяопераційного рубця відмічається посилене розростання грануляційної тканини. З метою гальмування проліферативного запалення йому місцево призначили глюкокортикоїдну мазь. Який механізм гальмування клюкокортикоїдами процесів проліферації?

A *гальмування проліферації фібробластів

B активація проліферації макрофагів

C стимуляція резорбції колагену еозинофілами

D збільшення синтезу колагенових волокон

E активація колагеназ, що руйнують колаген

Хвороби системи крові.

1. У хворого на анемію при дослідженні крові виявлено переважання еритробластів, нормобластів та мегалобластів. Такі ж клітини знайдено в червоному кістковому мозку. Для якого виду анемії характерна така картина крові?

A. *B12 -фолієво-дефіцитна

B. Гемолітична

C. Залізодефіцитна

D. Післягеморагічна

E. Апластична

2. При патогістологічному дослідженні біопсійного матеріалу печінки чоловіка 70 років, у якого в крові виявлена велика кількість лимфоцитів і пролімфоцитів, встановлено: множинні скупчення зазначенних клітин переважно проміж печінковими часточками. Для якого захворювання характерні вищеописані зміни?

A. *Хронічний лімфолейкоз

B. Гострий лімфолейкоз

C. Лімфогрануломатоз

D. Хронічний персистуючий гепатит

E. Гепатоцелялярний рак печінки

3. У хворого при огляді порожнини рота визначається атрофія слизової оболонки язика з червоними плямами (Гунтеровський глосит). Склери жовтуваті. В аналізі крові – кольоровий показник більше одиниці. Для якої анемії характерні ці зміни?

A. * В12-фолієводефіцитна

B. Залізодефіцитна

C. Гостра постгеморагічна

D. Хронічна постгеморагічна

E. Гемолітична

4. У дитини 7 років з’явилась слабкість, носові кровотечі, численні петехії на шкірі, лихоманка, до якої приєдналась явища менінгіту. У крові кількість лімфоцитів збільшена до 50 тис., з них 80% – лімфобласти. Ваш діагноз?

A *Гострий Т-лімфобласний лейкоз

B Гострий В-лімфобласний лейкоз

C Хронічний мієлоцитарний лейкоз

D Хронічний лімфоцитарний лейкоз

E Хронічний моноцитарний лейкоз

5. При розтині трупа чоловіка 48 років виявлено, що кістковий мозок плоских кісток, діафізів та епіфізів трубчастих кісток соковитий, сіро-червоний або сіро-жовтий гноєподібний(піоїдний кістковий мозок). Селезінка масою 7кг. На розрізі вона темно червоного кольору, з ішемічними інфарктами. Всі лімфатичні вузли збільшені, м’які, сіро-червоного кольору. В печінці жирова дистрофія і лейкемічні інфільтрати. Ваш діагноз?

A *Хронічний мієлоїдний лейкоз

B Хронічний лімфоїдний лейкоз

C Гострий мієлоїдний лейкоз

D Гострий лімфоїдний лейкоз

E Лімфогранулематоз

6. При клінічному обстеженні 15-річного підлітка у периферичній крові – лейкоцитоз (до 70 тис. лейкоцитів) з різким збільшенням числа бластних клітин і наявністю поодиноких зрілих елементів при відсутності перехідних дозріваючих форм, анемія, тромбоцито- та нейтропенія. На шкірі і слизових оболонках – петехіальні крововиливи. Незважаючи на антибактеріальну терапію, стан хворого прогресивно погіршувався, і при наростаючих явищах інтоксикації він помер. На аутопсіїї кістковий мозок губчастих і трубчастих кісток з зеленуватим (гноєподібним) відтінком. Лімфатичні вузли не збільшені, маса селезінки біля 600г. Ваш діагноз?

A *Гострий мієлобластний лейкоз, лейкемічний варіант

B Гострий лімфобластний лейкоз, лейкемічний варіант

C Гострий мієлобластний лейкоз, алейкемічний варіант

D Гострий лімфобластний лейкоз, алейкемічний варіант

E Хронічний лімфобластний лейкоз

7. При розтині трупа чоловіка 65 років виявлено, що кістковий мозок плоских і трубчастих кісток червоного кольору. Лімфатичні вузли всіх груп збільшені, на розрізі червоного кольору. Маса селезінки 900г., вона м’ясистої консистенції, на розрізі червонувата. Печінка збільшена, щільна, на розрізі сіро-коричнева, з наявністю дрібних сіро-білих вузликів. Нирки збільшені, щільні, сіро-коричневого кольору на розрізі, малюнок стертий. У всіх перерахованих органах при мікроскопічному дослідженні виявлена лейкозна інфільтрація. Ваш діагноз?

A *Хронічний лімфоїдний лейкоз

B Хронічний мієлоїдний лейкоз

C Гострий мієлобластний лейкоз

D Гострий лімфобластний лейкоз

E Лімфогрануломатоз

8. У хворого, в результаті харчовго отруєння, розвилась невгамовна блювота. рН крові 7,50, рСО2 артеріальної крові 50 мм.рт.ст., буферні основи – 38 ммоль/л, зсув буферних основ +6 ммоль/л. Яке порушення кислотно-основного стану має місце?

A * Негазовий алкалоз

B Газовий алкалоз

C Негазовий ацидоз

D Газовий ацидоз

E Нирковий ацидоз

9. У пацієнта після важкої травми розвинулася нирково-печінкова недостатність. Залишковий азот – 60 ммоль/л, сечовина – 8 ммоль/л, аміак – 300 мкм/л, креатинін – 380 мкм/л. Яке порушення азотистого балансу?

A * Змішана азотемія

B Ниркова азотемія

C Печінкова азотемія

D Позитивний азотистий баланс

E Негативний азотистий баланс

10. Після перенесеного інфекційного захворювання у хворого в аналізі крові збільшилась запальна кількість лейкоцитів до 30х109/л, зсув лейкоцитарної формули вліво, в лейкоцитах тільця Князькова-Деле, незначна анемія. Ваш діагноз?

A * Лейкемоїдна реакція

B Хронічний лімфолейкоз

C Гострий лімфолейкоз

D Гострий мієлолейкоз

E Хронічний моноцитарний лейкоз

11. У хворого зі спадковою серпоподібною аномалією еритроцитів, захворювання пневмонією супроводжувалося гемолітичним кризом із розвитком анемії. Що є безпосередньою причиною гемолітичного кризу?

A *Гіпоксія, викликана пневмонією

B Зміна осмолярності крові

C Гіпероксія

D Гетерозиготність за Нb S

E Мутація структурного гена

12. Хворий звернувся до лікаря зі скаргами на загальну слабкість, гематоми на тілі. В крові: лейкоцитів 9,0х109/ л. Лейкоцитарна формула: Б-0, Е-1, мієлобласти-5%, П-2, С-55, Л-20, М-4. Про яку патологію з нижченаведених можна думати?

A * Гострий мієлолейкоз

B Хронічний мієлолейкоз

C Гострий лімфолейкоз

D Хронічний лімфолейкоз

E Лейкемоїдна реакція

13. У хворого на рак сечового міхура під час проведення цитостатичної хіміотерапії в аналізі крові встановлено: лейкоцити – 0,8х109/л, гранулоцити – 0,6х109/л. Яке ураження білої крові має місце у хворого?

A *Мієлотоксичний агранулоцитоз.

B Імунний агранулоцитоз.

C Алейкія.

D Лейкопенія.

E Лейкоцитоз.

14. У хворого відмічається різке збільшення печінки і селезінки. Селезінка займає майже половину черевної порожнини. В периферичній крові 120000 лейкоцитів в 1 мл. Основну масу білих кровяних тілець складають промієлоцити і мієлоцити. Про яке захворювання йде мова?

A *Мієлоцитарний лейкоз

B Мієлобластний лейкоз

C Лімфоцитарний лейкоз

D Лімфобластний лейкоз

E Лімфосаркома

15. У хворого виявлена спленомегалія, геморагічний діатез. Периферична кров не змінена. При пункції груднини кістковий мозок представлений тільки мієлобластами. Про яке захворювання йде мова?

A *Мієлобластний лейкоз

B Лімфобластний лейкоз

C Лімфосаркома

D Мієлоцитарний лейкоз

E Лімфоцитарний лейкоз

16. У хворого виявлено геморагічний синдром з крововиливами та кровотечами, збільшені лімфатичні вузли, селезінка не збільшена. В кістковому мозкузбільшена кількість мегакаріоцитів. В периферичній крові кількість тромбоцитів 90х109/л. Про яке захворювання йде мова ?

A *Тромбоцитопатія

B ДВЗ-синдром

C Анемія Аддісон -Бірмера

D Мегакаріобластний лейкоз

E Лімфоцитарний лейкоз

17. У хворого з анемією виявлено лейкопенію, в мазку крові - атипові нейтрофіли гігантських розмірів з гіперсегментованими ядрами. Вони характерні для:

A12 - дефіцитної анемії

B Залізодефіцитної анамії

C Серповидноклітинної анемії

D Токсико-гемолітичної анемії

E Анемії Мінковського-Шоффара

Хворобо серцево-судинної системи. Ревматичні хвороби.

1. На розтині померлої виявлено морфологічні прояви стеноз лівого атріовентрикулярного отвору, недостатність мітрального клапана. Гістологічно в міокарді – вогнищевий кардіосклероз, наявність квітучих гранулем Ашоф-Талалаєва. Який з перерахованих нижче діагнозів найбільш імовірний?

A. * Ревматизм.

B. Склеродермія.

C. Дерматоміозит.

D. Вузликовий периартериїт.

E. Системний червоний вовчак.

2. На аутопсії трупа чоловіка 39 років, який раптово помер, в інтимі черевного відділу аорти визначаються ділянки жовтуватого кольору, які не підвищуються над її поверхнею. Гістологічно – в цих ділянках серед глад-ком’язових клітин і макрофагів виявляються клітини з пінистою цитоплазмою. При забарвленні суданом Ш відзначається їх яскраво-жовто-гарче забарвлення. Для якої стадії атеросклерозу найхарактерніша описана картина?

A. *ліпоїдозу

B. Ліпосклерозу

C. Атероматозу

D. Виразки

E. Атерокальцинозу

3, Мікроскопійне дослідження коронарної артерії у померлого 53 років виявило звуження прозору судини за рахунок фіброзної бляшки з домішком ліпідів. Найвірогідніша форма атеросклерозу:

A. *ліпосклероз

B. ліпоїдоз

C. доліпідна

D. атероматоз

E. виразкування

4. При розтині померлого від набряку легень в міокарді знайдено велике вогнище жовто-сірого кольору, а в коронарній артерії – свіжий тромб. Уточніть діагноз:

A. *Інфаркт міокарда

B. Кардіосклероз

C. Міокардит

D. Амілоїдоз

E. Кардіоміопатія

5. При гістологічному дослідженні стулки мітрального клапана серця виявлено: мукоїдний набряк, пошкодження ендотелію, утворення по замикаючому краю фібринових тромбів. Яка форма ревматичного ендокардиту спостерігається?

A. *Гострий бородавчастий ендокардит

B. Дифузний ендокардит

C. Фібропластичний ендокардит

D. Поворотньо-бородавчастий ендокардит

E. Поліпозно-виразковий ендокардит

6. Під час розтину в міокарді макроскопічно виявлено крупновогнищеву щільну ділянку сірого кольору, що гістологічно складається із грубоволокнистої сполучної тканини, оточену гіпертрофованими м’язевими волокнами. Які зміни виникли у серці?

A. *Постінфарктний крупновогнищевий кардіосклероз

B. Ішемічна стадія інфаркту міокарду

C. Некротична стадія інфаркту міокарда

D. Дрібновогнищевий дифузний кардіосклероз

E. Міокардит

7. На розтині хворої 25-ти років, яка померла від уремії: нирки збільшені, пістряві, з осередками крововиливів. Патогістологічно виявляються гематоксилінові тільця, капілярні мембрани клубочків у вигляді дротяних петель, гіалінові тромби та осередки фібриноїдного некрозу, у судинах селезінки – “цибулинний” склероз. Який найбільш вірогідний діагноз?

A. *Системний червоний вовчак.

B. Системна склеродермія.

C. Ревматоїдний артрит.

D. Ревматичний артрит.

E. Вузликовий періартеріїт.

8. У хворого 50 років на протязі багатьох років була ревматична вада серця. При загостренні захворювання розвинулась геміплегія і наступила смерть. Гістологічно в мітральному клапані виявлено виражений склероз, вогнищеві клітинні інфільтрати, фібринозні накладання. Для якої форми ендокардиту характерні виявлені зміни?

A. *Зворотньо-бородавчастого

B. Гострого бородавчастого

C. Дифузного

D. Фібропластичного

E. Виразково-поліпозного

9. При розтині трупа жінки, яка померла з явищами серцевої недостатності, серце збільшене в об’ємі, дрябле; міокард – глинисто-жовтий, тьмяний; з боку ендокарду видно жовто-білу посмугованість (тигрове серце). Мікроскопічно: в групах кардіоміоцитів відсутня поперечна посмугованість, цитоплазма кардіоміоцитів вміщує дрібні краплини, що фарбуються суданом-IV в чорний колір. Ваш діагноз ?

A. *Жирова дистрофія міокарда

B. Кардіосклероз

C. Ревматичний міокардит

D. Ожиріння серця

E. Міомаляція

10. При розтині померлої в комі молодої людини виявлено поширений тромбоемболічний інфаркт лівої півкулі мозку, а також велика септична селезінка, імунокомплексний гломерулонефрит, виразки в стулках аортального клапану, прикриті поліпоподібними тромбами з колоніями стафілококів. Яке захворювання привело до церебральної тромбоемболії?

A. *Септичний бактеріальний ендокардит

B. Септицемія

C. Гострий ревматичний вальвулит

D. Септикопіємія

E. Ревматичний тромбендокардит

11.У хворого, який переніс інфаркт міокарду, після фізичного навантаження відмічаються ознаки недостатності кровообігу: ціаноз та набряк підшкірної клітковини нижніх кінцівок. Які морфологічні зміни розвиваються на місц інфаркту міокарда у особи, що одужала?

A. *Кардіосклероз

B. Внутрішньоклітинна регенерація

C. Мітотичне новоутворення кардіоміоцитів

D. Міокардит

E. Атрофія міокарду

12.При гістологічному дослідженні вушка серця у хворого на стеноз мітрального клапану виявлені гранульоми Ашоффа-Талалаєва. Про який генез вади серця свідчать гістологічні дослідження?

A. *Ревматичний

B. Атеросклеротичний

C. Сифілітичний

D. Природжений

E. Септичний

13.У хворого на гіпертонічну хворобу діагностований багатогодинний гіпертонічний криз. Які патоморфологічні зміни в стінках артеріол виникають під час гіпертонічного кризу?

A. *Фібриноїдний некроз

B. Гіаліноз

C. Склероз

D. Амілоїдоз

E. Кальциноз

14. Хворий переніс повторний інтрамуральний інфаркт міокарду. Після лікування та реабілітації виписаний у задовільному стані під нагляд дільничого терапевта. Через 2 роки загинув у автомобільній катастрофі Встановити характер патологічного процесу в міокарді, який було встановлено на розтині.

A. *Крупновогнищевий кардiосклероз.

B. Дрiбновогнищевий кардiосклероз.

C. Атрофiя.

D. Некроз.

E. Гiперплазiя.

15. У хворого, що довгий час вживав наркотики, лихоманка, явища інтоксикації, піурія, на ехокардіоскопії виявлені масивні напластування на клапанах серця. З крові висеяний стафілокок. Хворий помер від тромбоемболі легеневої артерії. Вкажіть на зміни серця, виявлені у хворого під час розтину.

A. *Поліпозно-виразковий ендокардит.

B. Ендокардит Лібмана-Сакса.

C. Дифузний вальвуліт.

D. Гострий бородавчатий ендокардит.

E. Зворотно-бородавчастий ендокардит.

16. Чоловік 38 років загинув при спробі підйому ваги. Розвинувся колаптоїдний стан. На аутопсії виявлений розрив обширної аневрізми грудного відділу аорти. Протягом життя страждав вісцеральним сифілісом. Який патологічний процес в даному випадку обумовив зменшення міцності стінки аорти, її розширення і розрив.

A. *Зникнення еластичних волокон.

B. Зникнення колагенових волокон.

C. Атрофія м'язового шару.

D. Зміни інтіми по типу "шагреневї шкіри".

E. Новоутворення судин.

17. При розтині тіла хворого 57 років, який страждав на гіпертонічну хворобу та вмер від серцевої декомпенсації, знайдено збільшене серце з розширеними порожнинами (маса його 550 г товща стінки лівого шлуночку 2,5 см). мікроскопічно кардіоміоцити значно збільшени у розмірах, з жировою дистрофією та гіперхромними бочкоподібними ядрами. Який з переличених патологічних процесів найбільш вірогідний у серці?

A. *Ексцентрична гіпертрофія

B. Міокардит

C. Концентрична гіпертрофія

D. Ангіогенний кардіосклероз

E. Кардіоміопатія

18. У хворої після переохолодження розвилася деформація суглобів, болючість та обмеженість рухів у пальцях кінцівок; поряд з суглобами з’явились маленькі щільні вузлики. При біопсії в вузликах знайдені вогнища фібриноїдного некрозу, оточені гістіоцитами. Ваш діагноз:

A. *Ревматоїдний артрит

B. Дерматоміозит

C. Ревматизм

D. Подагра

E. Деформуючий артроз

18. При гістологічному дослідженні стінки аорти виявлено велику кількість ксантомних клітин, які розташовані переважно в інтимі. Для якого захворювання можлива така картина?

A. *Атеросклероз

B. Гіпертонічна хвороба

C. Сифілітичний мезаортит

D. Неспецифічний аортоартеріїт

E. Нодозний періартеріїт

19.У померлого від раптової зупинки серця чоловіка 45 років знайдено симетричний тип ожиріння III ступеню, розрив стінки правого шлуночка з гемоперікардом; під епікардом надлишкові відкладення жиру. Мікроскопічно: жирова тканина з епікарду розповсюджується у міокард з атрофією м'язових волокон. Який процес найбільш вірогідний?

A. *Ожиріння серця

B. Жирова дистрофія міокарда

C. Гострий інфаркт міокарда

D. Iшемічна хвороба серця

E. Гіпертонічна хвороба

20. При розтині чоловіка 70 років, померлого від серцево-судинної недостатності, який страждав на стенокардію, гіперхолестерінемію та ожиріння, знайдено хронічне венозне повнокрів'я органів, гіпертрофія лівого шлуночка серця з дрібноосередковим кардіосклерозом, об'ємні жовтувато-білуваті бляшки в інтимі аорти з дрібнозернистими масами у центрі, які заглиблюються у товщу стінки. Який патологічний процес має місце?

A. *Атероматоз

B. Ліпоїдоз

C. Ліпосклероз

D. Артеріолосклероз

E. Кальциноз

21. У дитини 7 років на шкірі розгинальних поверхонь ліктьових і колінних суглобів з'явились щільні, безболісні вузлики розміром 1-2 мм. В біоптаті вузликів - велике вогнище фібриноїдного некрозу сполучної тканини з лімфоцитами і макрофагами по периферії. При якому захворюванні спостерігаються такі зміни?

A. *Ревматизм

B. Ревматоїдний артрит

C. Склеродермія

D. Вузликовий періартеріїт

E. Системний червоний вовчак

22. На розтині трупа чоловіка 60 років в міокарді передньої стінки лівого шлуночка серця виявлено сіре неправильної форми щільне вогнище 5 х 4 см. з чіткими границями волокнистої структури. Який діагноз найбільш вірогідний?

A. *Післяінфарктний міокардіосклероз

B. Дифузний дрібновогнищевий міокардіосклероз

C. Інфаркт

D. Міокардит

E. Кардіоміопатія

23.При дослідженні коронарних артерій серця виявлені атеросклеротичні бляшки з кальцинозом, що закривають просвіт на 1/3. В м´язі дрібні множинні білуваті прошарки сполучної тканини. Як називається процес, виявлений в міокарді?

A. * Дифузний кардіосклероз

B. Тигрове серце

C. Післяінфарктний кардіосклероз

D. Міокардит

E. Інфаркт міокарда

24.Чоловік 63 років, який протягом 15 років страждав хронічною дифузною обструктивною емфіземою легень, помер від прогресуючої серцевої недостатності. На розтині виявлен мускатний цироз печінки, цианотична індурація нирок та селезінки, асцит, набряки нижніх кінцівок. Для якого типу серцевої недостатності характерні дані зміни у внутрішніх органах?

A. *Хронічна правошлуночкова недостатність

B. Гостра правошлуночкова недостатність

C. Хронічна лівошлуночкова недостатність

D. Гостра лівошлуночкова недостатність

E. Загальна серцева недостатність

25.Під час гістологічного дослідження стулок мітрального клапана серця жінки 30 років було встановлено, що ендотеліальні клітини вогнищево десквамовані, в цих ділянках на поверхні стулки розташовані дрібні тромботичні нашарування, сполучна тканина стулки з явищами мукоїдного набрякання з ділянками склерозу та васкулярізації. Діагностуйте вид клапанного ендокардиту?

A. *Поворотньо-бородавчастий

B. Дифузний

C. Гострий бородавчастий

D. Фібропластичний

E. Поліпозно-виразковий

26.Жінка 54 років мала значну деформацію суглобів пальців рук і стоп. Гістологічно: навколосуглобова сполучна тканина - мукоїдне набухання, ділянки фібриноїдного некрозу, скупчення макрофагів, ділянки склерозу; в синовіальній оболонці - набрякі ворсини, з ознаками мукоїдного та фібриноїдного набухання, в синовіальній порожнині зустрічаються поодинокі "рисові тільця". Діагностуйте захворювання.

A. *Ревматоїдний артрит

B. Ревматизм

C. Хвороба Бехтерева

D. Гематогенний туберкульоз

E. Подагра

27.При мікроскопічному дослідженні міокарду дівчинки, яка померла від дифтерії внаслідок серцевої недостатності зайдені: жирова дистрофія та множинні осередки некрозу кардіоміоцитів, незначні вогнищеві клітинні інфільтрати інтерстицію. Про який міокардит йде мова?

A. *Альтеративній міокардит

B. Дифузний ексудативний міокардит

C. Осередковий ексудативний міокардит

D. Інтерстиціальний міокардит

E. Гранульоматозний міокардит

28.При розтині трупу жінки 48 років, яка раптово померла, в інтимі аорти визначаються ділянки жовтуватого кольору, які не підвищуються над її поверхнею. При гістологічному дослідженні цих ділянок виявляються клітини з пінистою цитоплазмою, а при фарбуванні суданом ІІІ жовтого кольору. Для якої стадії атеросклерозу характерні такі зміни в аорті?

A. *Ліпоїдозу

B. Ліпосклерозу

C. Атероматозу

D. Виразки

E. Атерокальцинозу

29.При розтині трупу чоловіка 62 років, який хворів на атеросклероз і помер від гострої серцевої недостатності, у передній стінці лівого шлуночка серця виявлена щільна ділянка біло-жовтого кольору розміром 6х5 см, неправильної форми, чітко відмежована від навколишніх тканин гемарагічним вінчиком. Встановіть діагноз.

A. *Інфаркт міокарду

B. Постінфарктний кардіосклероз

C. Дрібно-осередковий кардіосклероз

D. Міокардит

30.При розтині дитини, померлої від серцевої недостатності, виявлено: розширення порожнин шлуночків серця. Мікроскопічно в стромі міокарда повнокрів’я, набряк, розповсюдженні інфільтрати з гістіоцитів, лімфоцитів, нейтрофілів, еозинофілів. Який найбільш вірогідний діагноз?

A. *Дифузнопроміжний ексудативний міокардит

B. Осередковий проміжний ексудативний міокардит

C. Вузликовий продуктивний міокардит

D. Проміжний продуктивний міокардит

E. Альтеративний міокардит

31. При розтині трупа чоловіка 47 років померлого раптово, в інтимі черевного відділу аорти знайдені осередки жовтого кольору у вигляді плям та смуг, що не вибухають над поверхнею інтими. При фарбуванні суданом ІІІ спостерігається померанчове забарвлення. Для якої сдадії атеросклерозу характерні такі зміни?

A. *Ліпоїдозу.

B. Ліпосклерозу.

C. Атероматозу.

D. Атерокальцинозу.

E. Стадія утворення атероматозної виразки.

32. При розтині трупа жінки 69 років, підвищеного харчування, померлої від гострого інфаркту міокарду, в інтимі коронарних артерій знайдені багаточисленні білуваті, щільні, вибухаючі в просвіт та різко звужуючі його формування. Для якої стадії атеросклерозу характерні такі зміни?

A. * Ліпосклерозу.

B. Ліпоїдозу.

C. Атероматозу.

D. Атерокальцинозу.

E. Стадія утворення атероматозної виразки.

33. Під час мікроскопічного дослідження аорти виявлено: вогнищева інфільтрація інтими ліпідами, білками. Ліпіди просочують інтиму та накопичуються у міоцитах середнього шару та макрофагах. Встановіть стадію атеросклерозу.

A. *Ліпоїдоз

B. Ліпосклероз

C. Атероматоз

D. Атерокальциноз

E. Доліпідна стадія

34. При гістологічному дослідженні біоптату з ураженої ділянки шкіри виявлено: незначно виражену атрофію епідермісу, в дермі гіаліноз колагенових волокон, скупі периваскулярні лімфоцитарні інфільтрати, у підлеглих скелетних м’язах - виражений інтерстиційний набряк, втрату поперечної посмугованості, дрібновогнищеві некрози з петрифіацією. Діагностуйте виявлене захворювання.

A. *Дерматоміозит

B. Склеродермія

C. Системний червоний вовчак

D. Вузилковий периартериїт

E. Ревматизм

35. При гістологічному дослідженні серця померлого від гострої серцевої недостатності в міокарді лівого шлуночка виявлено ділянка омертвіння, відмежована від неушкодженої тканини зоною повнокровних судин, дрібних крововиливів та лейкоцитарною інфільтрацією. Який діагноз найбільш вірогідний?

A. * Інфаркт міокарду

B. Ішемічна дистрофія міокарду

C. Осередковий ексудативний міокардит

D. Дифузний ексудатиний міокардит

E. Продуктивний міокардит

36.На розтині померлої дівчини 19 років знайдено бородавчастий ендокардит усіх клапанів, Осередкові крововиливи під ендокард. Виразковий стоматит. Виразки стравоходу. Пневмонія. Нефрит. Гіперплазія селезінки з периваскулярним склерозом. Васкуліт і некроз в ділянці зорового бугра Численні фурункули. Яке більш ймовірне захворювання?

A. *Системний червоний вовчак

B. Ревматизм

C. Септичний ендокардит

D. Ішемічна хвороба серця

E. Гіпертонічна хвороба

37. У померлого з підозрою на ідіопатичний міокардит на секції виявлено збільшене, кволе серце; порожнини розтягнуті з незначними тромботичними накладаннями; м’яз на розтині пістрявий, клапани інтактні. Мікроскопічно: переважання гідропічної дистрофії та лізис кардіоміоцитів; реактивні зміни відсутні. В осередках загибелі м’язових клітин колапс ретикулярної строми. Який морфологічний (гістологічний) тип ідіопатичного міокардиту має місце в даному випадку?

A *Дистрофічний (деструктивний) тип

B Запально-інфільтративний тип

C Змішаний тип

D Судинний тип

E Жодний із вказаних

38. Чоловік 44 років поступив в відділення невідкладної кардіології з інтенсивними загрудинними болями, блідістю, гіпотонією до 60/40 мм рт.ст.. Запідозрено гострий інфаркт міокарда. Проведена терапія виявилась неефективною, через 40 хв. зареєстрована зупинка серця. На розтині виявлена розшаровуюча аневризма дуги аорти до 10 см в діаметрі. Інтима аорти у висхідній частині і дузі морщиниста, далі на всьому протязі з жовтуватими плямами і жовтувато-білими бляшками, кількість яких наростає до черевного відділу аорти. Мікроскопічно інтима аорти відшарована; між нею і середнім шаром згортки крові, в середньому шарі крупновогнищева інфільтрація лімфоїдними, плазматичними клітинами, ділянки казеозного некрозу, з руйнуванням еластичних волокон і розростанням сполучної тканини; виражене проліферативне запалення. Який діагноз?

A * Сифілітичний мезаортит

B Ревматичний аортит

C Атеросклеротична аневризма аорти

D Неспецифічний аорто-артеріїт

E Вузликовий поліартеріїт.

39. При розтині померлого виявлено збільшене в размірі серце (вага 850,0). Товщина стінки лівого шлуночка 2 см, трабекулярні і сосочкові м´язи потовщені, порожнина серця не змінена. Яка можлива причина описаних змін?

A *Гіпертонічна хвороба

B Недостатність аортальних клапанів

C Емфізема легень

D Пневмосклероз

E Фіброзуючий альвеоліт

40. У хворої 42 років, померлої при явищах уремії, виявлені ерітема шкіри, особливо виражена на лиці в ділянці виличних дуг, також на шкірі сгинальних поверхонь рук і на стегнах є округлі бляшки яскраво-червоного кольору, які лущаться. При мікроскопічному дослідженні в шкірі спостерігається значне стоншення епідермісу, гіперкератоз, відсутність в дермі сітчатого шару, наявність щільних лімфо-макрофагальних інфільтратів вздовж дермо-епідермального з´єднання, а також навколо судин, волосяних фолікулів, потових залоз. При імунофлюоресцентному дослідженні виявлено наявність імунних комплексів в базальних мембранах дермо-епідермального з´єднання і придатків шкіри. Для якого ревматичного захворювання характерне таке ураження шкіри?

A * Системний червоний вовчак

B Дерматоміозит

C Псоріаз

D Системна склеродермія

E Ревматизм

41. У хворої 56 років, померлої при явищах кахексії, пов´язаної з непрохідністю стравоходу, на розтині виявлені наступні зміни: шкіра на верхніх кінцівках і обличчі щільна, натягнута, блискуча, на тулубі – набрякла, з наявністю телеангіоектазій; стінка стравоходу в нижній третині різко потовщена, м´язевий шар не визначається, слизова оболонка не утворює складок; нирки зменшені в розмірах, кора нирок потоншена; в серці визначаються множинні прошарки сполучної тканини. При мікроскопічному дослідженні в шкірі визначається різкий фіброз дерми, відсутність сосочкового шару дерми, гіаліноз артеріол, різке стоншення епідермісу, вогнища кальцифікації. У внутрішніх органах відмічається виражений фіброз, периваскулярні лімфоцитарні інфільтрати, зменшується просвіт артеріол за рахунок потовщення інтими і проліферації ендотеліоцитів. В стравоході спостерігається розростання щільної сполучної тканини, повна відсутність гладком´язевих волокон в середньому шарі. Ваш діагноз:

A * Системна склеродермія

B Системний червоний вовчак

C Вузликовий периартеріїт

D Дерматоміозит

E Ревматоїдный артрит

42. У хворого з дифтерією на 2-му тижні хвороби з'явилися постійні болі в області серця, які не змінювали інтенсивності при фізичному навантаженні, зростала задишка. Смерть наступила від серцевої недостатності. На розтині виявлена значна ділятація камер серця, при гістологічному дослідженні в міокарді - значна гіперемія капілярів, жирова дістрофія і міоліз кардіоміоцитів, лімфогістіоцитарна інфільтрація міжм'язової строми. Ваш діагноз.

A *Міокардит

B Кардіоміопатія

C Гостра коронарна недостатність

D Інфаркт міокарду

E Стенокардія

43. Хворий, який переніс інфаркт міокарда помер через 3 місяці від хронічної серцевої недостатності. На розтині на боковій стінці лівого шлуночка знайдено щільне мішковидне випинання, стінка шлуночка в цьому місці стоншена, має білий кольор. Для якої патології характерні ці зміни?

A * Хронічна аневризма серця

B Кардіоміопатія

C Інфаркт міокарда

D Міокардит

E Кардіосклероз

44. У хворого діагностована хвороба Карі - облітеруючий ендофлебіт печінкових вен, що одночасно супроводжується розвитком дистрофічного процесу в печінці і розростанням сполучної тканини. Які розлади гемодинаміки спричинюютьрозвиток цієї хвороби?

A * Симптомокомплекс портальної гіпертензії.

B Застій крові у системі нижньої поржнистої вени.

C Порушення кровопостачання кишечника.

D Застій крові у системі верхньої порожнистої вени.

E Гепато-ренальний синдром.

45.На розтині жінки 28 років при огляді виявлено: на шкірі обличчя ділянки червоного кольору, що лущаться та з´єднані на переніссі. Серце збільшене, стулки аортального клапану потовщені з бородавчатими накладаннями; нирки збільшені, строкаті; селезінка збільшена. Мікроскопічно: в селезінці – периартеріальний “цибулиний” склероз; в нирках - потовщення капілярних мембран клубочків, тромби і вогнища фібриноїдного некрозу. Ваш діагноз:

A * СЧВ

B Ревматизм

C Нефропатичний амілоїдоз

D Септичний ендокардит

E Вузликовий периартеріїт

46. При мікроскопічному дослідженні аорти було встановлено накопичення в інтимі великої кількості макрофагів, переповнених жировими включеннями. Для якої стадії атеросклерозу характерні такі структурні зміни?

A *Стадії пінистих клітин

B Стадії ліпідних плям

C Стадії фіброзних бляшок

D Стадії ускладнень

E Завершальної стадії

47. У юнака 18 років, що страждав ревматизмом і помер від серцевої декомпенсації, на аутопсії виявлені вкорочені, потовщені, щільні, білуваті стулки мітрального клапану. Мікроскопічно в тканині мітрального клапану виявлені явища дифузного склерозу і гіалінозу. Який вид ендокардиту найбільш вірогідний?

A * Фібропластичний

B Дифузний

C Гострий бородавчатий

D Зворотньо-бородавчатий

E Поліпозно-виразковий

48. При морфологічному дослідженні ампутованої нижньої кінцівки з приводу гангрени, просвіт стегнової артерії звужений бляшками камянистої щільності, частково звапнені з наявністю на них обтуруючих тромбів. Який з перерахованих діагнозів найбільш вірогідний?

A *Атеросклероз

B Облітеруючий ендартеріїт

C Неспецифічний аортоартеріїт

D Облітеруючий тромбангіт

E Вузликовий периартеріїт

49. При дослідженні біоптату нирок (за даними ультразвукового дослідження збільшених в розмірах) жінки молодого віку на фоні блідо зафарбованих гематоксиліном ядер виявляються інтенсивно зафарбовані внутрішньоядерні включення, потовщення капілярних мембран клубочків, поява гіалінових тромбів і вогнищ фібриноїдного некрозу. Яке захворювання можна запідозрити?

A * Системний червоний вовчак

B Мієломна хвороба

C Гіпертонічна хвороба

D Пієлонефрит

E Первинний амілоїдоз

50. На автопсії тіла чоловіка 49 років, який помер від гострої лівошлуночкової недостатності, у лівому шлуночку серця знайдено сіро-жовте вогнище неправильної форми, оточене темно-червоним вінчиком. Вогнище великих розмірів і охоплює всю товщу стінки шлуночка. Діагностуйте патологічний процес.

A * Інфаркт міокарду.

B Абсцес міокарду.

C Кардіосклероз.

D Флегмона серцевого м’яза.

E Гематома.

51. 54-річний чоловік раптово помер йдучи на роботу. З анамнезу відомо, щопомерлий вважав себе хворим, не лікувався, лише іноді відзначав короткочасні болі за грудиною. При автопсії встановлено обтуруючий тромб лівої передньої міжшлуночкової артерії, який утворився біля атеросклеротичної бляшки, котра стенозувала просвіт на 2/3. Жодних інших морфологічних змін у серці діагностовано не було. Діагностуйте захворювання.

A * Ішемічна хвороба серця, раптова серцева смерть.

B Ішемічна хвороба серця, гострий інфаркт міокарду.

C Ішемічна хвороба серця, нестабільна стенокардія.

D Ішемічна хвороба серця, хронічна форма.

E Ішемічна хвороба серця, гострий повторний інфаркт міокарду.

52. Чоловік 58 років страждає на атеросклероз судин головного мозку. При обстеженні виявлена гіперліпідемія. Вміст якого класу ліпопротеїдів найбільш вірогідно буде підвищений при дослідженні сироватки крові?

A *Ліпопротеїди низької щільності

B Ліпопротеїди високої щільності

C Комплекси жирних кислот з альбумінами

D Хіломікрони

E Ліпопротеїди проміжної щільності

53. У хворого діагностовано стеноз мітрального клапана. До якого патофізіологічноготипу недостатності серця можна віднести цей процес?

A * Недостатність серця від перевантаженням опором

B Недостатність серця через ушкодження міокарда

C Недостатність серця через перевантаження об’ємом

D Недостатність серця через гіпертрофію міокарда

E Змішана форма недостатності серця

54. При проходженні профілактичного огляду у чоловіка, який працює шахтарем, лікар встановив зміни функціонального стану серця, що свідчать про серцеву недостатність в стадії компенсації. Що з нижче перечисленого є головним підтвердженням компенсації діяльності серця?

A * Гіпертрофія міокарда

B Тахікардія

C Збільшення артеріального тиску

D Задишка

E Ціаноз

Хвороби органів дихання.

1. У хворого висока температура, задуха, біль у правій частині грудної клітини. Плевральна пункція дала 700 мл в?язкої рідини жовто-зеленого кольору. Який патологічний процес розвився у плевральній порожнині?

A. *Емпієма плеври

B. Бронхопневмонія

C. Серозний плеврит

D. Геморрагичний плеврит

E. Карциноматоз плеври

2. У результаті гістологічного дослідження біоптату із стінки бронха хворого хронічним бронхітом в слизовому шарі виявлені розростання грануляційної тканини, що виступають над поверхнею слизового шару та містять дифузний запальний інфільтрат. Який вид бронхіту спостерігається у хворого?

A. * Хронічний поліпозний бронхіт

B. Хронічний слизисто-гнійний бронхіт

C. Хронічний слизистий бронхіт

D. Хронічний гнійний бронхіт

E. Хронічний деформуючий бронхіт

3. На розтині померлого від легенево-серцевої недостатності чоловіка 47 років у лівій легені знайдено порожнину розміром 4х4см, заповнену гнієм, стінка фестончаста, нерівна, представлена легеневою тканиною. Найвірогідніше це :

A. *гострий абсцес

B. хронічний абсцес

C. кавернозний туберкульоз

D. ехінококоз

E. фіброзуючий альвеоліт

4. Хворий на протязі 8 років скаржився на кашель з гнійним харкотинням, задуху. Пальці рук нагадували барабанні палички. Він помер з явищами легенево-серцевої недостатності. На розтині бронхи деформовані, з мішковидними випячуваннями стінки та гнійним запаленням в них. Найбільш вірогідний діагноз:

A. *бронхоектатична хвороба

B. туберкульоз

C. хронічний бронхіт

D. Абсцес

E. Гострий броніт

5.У хворого, померлого через 3 тижні від початку пневмонії, нижня доля правої легені різко збільшена, щільна, безповітряна, сіра, з накладеннями фібрину на плеврі. При мікроскопії у всих альвеолах цієї долі виявлено фібрин та сегментоядерні лейкоцити. Ваш діагноз:

A. *Крупозна пневмонія

B. Вогнищева бронхопневмонія

C. Грипозна пневмонія

D. Фібринозний плеврит

E. Інтерстициальна пневмонія

6. У хворого (67 років) з клінічним діагнозом хронічного бронхіту, пневмосклерозу, серцево-легеневої недостат-ності взято біоптат з підозрілої ділянки слизової правого бронха. Гістологічно встановлено клітинний і тканинний атипізм, появу структур у вигляді ”ракових перлин”. Якому патолоргічному процесу відповідають зазначені гістологічні зміни?

A. * Плоскоклітинний рак бронху з ороговінням

B. Хронічний поліпозний бронхіт.

C. Бронхоектаз.

D. Гострий бронхіт.

E. Плоскоклітинна метаплазія слизової бронху

7. На розтині хворого 65 років, що помер від хронічної серцевої недостатності внаслідок ревматичної вади серця легені бурого кольору, збільшені у розмірах, ущільнені. Як звуться такі зміни у легенях?

A. *бура індурація легень

B. мускатні легені

C. сотові легені

D. хронічний бронхіт

E. хронічна емфізема

8. У хворого висока температура, задуха, біль у правій половині грудної клітини. Під час плевральної пункціїї видалено 700 мл вершкоподібної рідини жовтувато-зеленого кольору. Найвірогідніший діагноз:

A. *емпієма плеври

B. карціноматоз плеври

C. серозний плеврит

D. фібринозний плеврит

E. геморагічний плеврит

9. Під час розтину в верхній долі правої легені виявлений крупний клиноподібний осередок темно-червоної, щільної тканини. При гістологічному дослідженні в ній виявлений некроз стінок альвеол, просвіт альвеол щільно заповнений еритроцитами. Який процес розвинувся в легенях?

A. *Геморагічний інфаркт легень

B. Карніфікація легень

C. Гангрена легень

D. Крововилив в легені

E. Ателектаз легень

10. При мікроскопічному дослідженні тканини легенів виявлено ділянку запалення, яка складається з вогнища некрозу, оточеного правильними рядами епітеліоїдних, лімфоїдних клітин, є плазматичні клітини, макрофаги т гігантські багатоядерні клітини Пирогова-Лангханса. Визначте вид запалення.

A. *Туберкульозне запалення

B. Банальне продуктивне запалення

C. Запалення при лепрі

D. Ексудативне запалення

E. Альтеративне запалення

11. При розтині прозектор виявив, що легені збільшені в розмірах, бліді, м'якої консистенції, не спадаються, ріжуться з хрустом. Мікроскопічно - розширення альвеолярних ходів, міжальвеолярні перегородки тонкі, є ознак інтракапілярного склерозу. Для якого захворювання легенів характерна така морфологічна картина?

A. *Емфізема

B. Пневмосклероз

C. Пневмоторакс

D. Ателектаз

E. Пневмонія

12. У хворого з легеневою патологією на розтині виявлена порожнина з ущільненими стінками, виповнена густою рідиною зеленуватого кольору, з непрємним запахом. Поставити діагноз.

A. *Абсцес легень.

B. Гангрена легень.

C. Iнфаркт легень.

D. Каверна легень.

E. Туберкулома легень.

13. У дитини 8 років виражені ознаки легенево-серцевої недостатності, ядуха, пальці у вигляді барабанних паличок, нігті у вигляді годинкових скелець, кашель з значною кількістю слизово-гнійного харкотиння (особливо вранці) При рентгенологічному дослідженні виявлене різке розширення бронхів. Встановіть діагноз.

A. *Бронхоектатична хвороба.

B. Бронхіальна астма.

C. Хронічний бронхіт.

D. Бронхопневмонія.

E. Емфізема легень.

14. 42-річний чоловік захворів гостро після охолодження. Захворювання супроводжувалось серцево-дихальною недостатністю, від якої він помер. Під час розтину виявлено, що уся прав легеня збільшена, печінкової щільності, важка; на плеврі – значні фібрінозні накладення. На розрізі легеня сірого кольору, зерниста, з поверхні стікає каламутна рідина. При гістологічному дослідженні – гостре запалення з наявністю у просвітах альвеол гнійно- фібринозного ексудату. Який з переличених діагнозів найбільш вірогідний?

A. *Крупозна пневмонія

B. Осередкова пневмонія

C. Інтерстиційна міжальвеолярна пневмонія

D. Стафілококова пневмонія

E. Ідиопатичний фіброзуючий альвеоліт

15. У 45-річного хворого, який гостро захворів на пневмонію, на 6-й день хвороби розвинувся набряк легень, який став причиною смерті. На розтині знайдено, що уражена вся верхня частка правої легені : збільшена, щільна, на плеврі з фібринозними нашаруваннями, на розрізі сірого кольору, з поверхні розрізу стікає каламутна рідина. Мікроскопічно : в просвіті альвеол - фібрин, нейтрофіли, макрофаги, гемолізовані еритроцити. Вкажіть, яка пневмонія була у хворого ?

A. * Крупозна пневмонія

B. Стафілококова бронхопневмонія.

C. Вірусна пневмонія

D. Гіпостатична пневмонія

E. Респіраторний дистрес-синдром дорослих

16. На розтині виявлено, що ліва легеня збільшена в розмірах, щільна, на вісцеральній плеврі є накладення фібрину, на розрізі - сірого кольору, з її поверхні зтікає мутна сіро – жовта рідина. Про яке захворювання іде мова?

A. *Крупозна пневмонія

B. Вогнищева пневмонія

C. Інтерстиційна пневмонія

D. Рак легені

E. Бронхоектатична хвороба

17. На розтині виявлено, що вся права легеня збільшена, щільна, на плеврі нашарування фібрину, на розрізі тканина сірого кольору, з якої стікає каламутна рідина. Для якого захворювання легенів характерна така картина?

A. *Крупозна пневмонія

B. Вогнищева пневмонія

C. Інрерстиціальна пневмонія

D. Гангрена легені

E. Фіброзуючий альвеоліт

18. У чоловіка, що за життя страждав на митральний стеноз, під час розтину виявлені ущільненіі легені, бурого кольору. Про який патологічний процес в легенях йде мова?

A. *Гемосидероз

B. Гемохроматоз

C. Жовтяниця

D. Гемомеланоз

E. Ліпофусциноз

19.При розтині трупа чоловіка зі злоякісною пухлиною шлунку, померлого від ракової інтоксикації, в задньонижніх відділах легень виявлені щільні сіро-червоного кольору неправильної форми осередки, які виступають над поверхнею розрізу. Мікроскопічно при цьому у просвіті, стінках дрібних бронхів та альвеолах виявляється ексудат, в якому багато нейтрофілів. Яке захворювання виявлене в легенях у померлого?

A. *Гостра гнійна бронхопневмонія

B. Гострий бронхіт

C. Крупозна пневмонія

D. Проміжна пневмонія

E. Гостра серозна бронхопневмонія

20. На розтині тіла чоловіка віком 58 років, який тривалий час вживав значну кількість алкоголю і помер вдома, було знайдено: макроскопово - права легеня щільна і збільшена в розмірах, тканина на розрізі сіруватого кольору однорідна, плевра вкрита сіруватими плівчастими нашаруваннями; мікроскопово - порожнини альвеол містять нітки фібрину, гемолізовані еритроцити. Діагностуйте захворювання.

A. *Крупозна пневмонія

B. Вогнищева пневмонія

C. Інтерстиційна пневмонія

D. Первинний туберкульоз легень

E. Казеозна пневмонія

21. На розтині трупа чоловіка 50 років виявлено наступні зміни: права легеня помірно у всіх відділах щільна, на розрізі тканина безповітряна, мілко зерниста, сухувата. Вісцеральна плевра з нашаруванням фібрину сіро-коричневого кольору. Визначте діагноз?

A. *Крупозна пневмонія

B. Туберкульоз

C. Бронхопневмонія

D. Інтерстиціальна пневмонія

E. Пневмофіброз

22. При розтині померлого 49 років, який хворів на крупозну пневмонію і помер від пневмококового сепсису, в лівій плевральній порожнині містилось до 700 мл каламутної зеленувато-жовтого кольору з неприємним запахом. Листки плеври тьмяні, повнокровні. Назвати клініко-морфологічну форму запалення в плевральній порожнині.

A * Емпієма

B Хронічний абсцес

C Гострий абсцес

D Флегмона

E Фібринозне запалення

23. Чоловік 68 років, що в останні 15 років страждав на емфізему легень, помер раптово від гострої серцевої недостатності. На розтині тіла померлого легені збільшені в розмірах, прикривають своїми краями переднє середостіння, здуті, бліді, м’які, не спадаються, ріжуться з хрустом; з просвіту бронхів, стінки яких стовщені сірого кольору, видавлюється слизисто-гнійний ексудат. Який найбільш вірогідно вид емфіземи був у хворого?

A *Хронічна обструктивна

B Перифокальна

C Вікарна

D Стареча

E Ідіопатична

24. Хворий помер від наростаючої легенево-серцевої недостатності. При гістологічному дослідженні виявлено дифузна поразка легенів з

інтерстиціальним набряком, інфільтрацією інтерстиціальної тканини лімфоцитами, макрофагами, плазмоцидами; пневмофіброз, панацинарна емфізема. Ваш діагноз.

A *Фіброзуючий альвеоліт

B Хронічний бронхіт

C Бронхопневмонія

D Ателектаз легенів

E Бронхіальная астма

25. Хворий 70 років, який довнго хворів на хронічний бронхіт, помер від наростаючої легенево-серцевої недостатності. При патогістологічному дослідженні виявлено: легені збільшені в розмірах, роздуті, бліді. В слизовій оболонці бронхів запальний інфільтрат, велика кількість келихоподібних клітин, проксимальні відділи ацинусів різко розширені. Ваш діагноз.

A *Емфізема легенів

B Бронхіальная астма

C Бронхопневмонія

D Межуточна пневмонія

E Хронічний бронхіт

26. У хворого з термінальною стадією хронічної ниркової недостатності (сечовина крові – 70 ммоль/л, креатинин – 1,07 ммоль/л) в кліниці з’явились симптоми утрудненого дихання, кашель. При бронхоскопії слизова оболонка бронхів повнокровна, набрякла, з дрібними крововиливами та виразками. В просвіті бронхів багато слизу. Ваш діагноз:

A * Гострий бронхіт

B Часткова пневмонія

C Хронічний бронхіт

D Бронхопневмонія

E Проміжна пневмонія

27. Хворий 68 років важкою формою грипу (штам вірусу грипу А2) з летальним завершенням. На секції зміни в легенях були подібні до змін „великих строкатих легенів”. При мікроскопічному дослідженні виявлено: різке повнокров’я судин, крововиливи, набрак легеневої тканини, в просвіті бронхів та альвеол ексудат, що містить переважно еритроцити. Ваш діагноз:

A *Геморагічна бронхопневмнія

B Катаральна бронхопневмнія

C Гнійна бронхопневмнія

D Десквамативна бронхопневмонія

E Фібринозна пневмонія

28. На автопсії тіла чоловіка 49 років, який помер від серцево-легеневої недостатності, знайдено дифузне ущільнення нижньої частки правої легеня. Вказана частка сіро-жовта, плевра над нею вкрита фібринозними нашаруваннями. При гістологічному дослідженні тканини ураженої частки в усіх альвеолах виявлено нейтрофіли, фібрин, поодинокі сидерофаги, фібринозний плеврит. Діагностуйте захворювання.

A * Крупозна пневмонія.

B Вогнищева гнійна пневмонія.

C Вогнищева фібринозно-гнійна пневмонія.

D Бура індурація легень.

E Пневмоконіоз.

29. У чоловіка 42 р. внаслідок ДТП пошкоджена грудна клітина. Він відчуває інтенсивну біль при диханні. Яке порушення дихання буде у хворого?

A *Тахіпное.

B Брадіпное.

C Гіпериное.

D Інспіраторна задишка.

E Експіраторна задишка.

30. У хворого інсулінзалежним цукровим діабетом прояви кетоацидозу. Які зміни зовнішнього дихання будуть у хворого?

A *Гіперпное.

B Тахіпное.

C Брадіпное.

D Дихання Чейна-Стокса.

E Дихання Біота.

Хвороби органів травлення.

1. У хворого з кривавою блювотою на операції в шлунку знайдена виразка, проникаюча в м’язовий шар шлунка. Краї виразки щільні, в дні – кровоточива судина. При цитобіопсії в краях і дні виразки виявлена рубцева тканина. Яка це виразка?

A. * Хронічна кровоточива виразка

B. Пенетруюча виразка

C. Гостра кровоточива виразка

D. Перфоративна виразка шлунка

E. Малігнізована виразка

2.Апендикс, надісланий до патоморфологічного відділення після апендиктомії, потовщений і збільшений у розмірах, серозна оболонка тмяна, судини повнокрівні, з просвіту відростка на розрізі виділяється рідина жовто-зеленого кольору. При якій формі апендициту розвиваються такі зміни?

A. * Флегмонозний апендицит.

B. Простий катаральний апендицит.

C. Поверхневий катаральний апендицит.

D. Гангренозний апендицит.

E. Апостематозний апендицит.

3. При мікроскопічному дослідженні печінки виявлено: венозне повнокрів’я центру часточок, дистрофія та атрофія гепатоцитів у вогнищах венозного застою, жирова дистофія гепатоцитів по периферії дольки з наявністю розростання сполучної тканини в місцях атрофії гепатоцитів. Про який патологічний процес іде мова?

A. *Мускатна печінка з предциротичними явищами.

B. Біліарний цироз печінки.

C. Гепатит.

D. Жировий гепатоз.

E. Токсична дистрофія печінки.

4. У хворого знайдено асцит, в два рази збільшена селезінка, варикозне розширення вен стравоходу і прямої кишки. При гістологічному дослідженні біоптата печінки виявлений мікронодулярний цироз. Який процес ускладнив цироз печінки?

A. * Синдром портальної гіпертензії

B. Серцева недостатність

C. Гепато-лієнальний синдром

D. Печінково-клітинна недостатність

5. У тканині печінки виявлено округле утворення діаметром 0.5 см. Мікроскопічно воно має наступну будову: у центрі – некротичні маси, їх оточує грануляційна тканина з наявністю у її складі плазматичних, лімфоїдних клітин і кровоносних судин з явищами васкуліту. Який діагноз необхідно поставити на підставі даних мікроскопії?

A. *Солітарна гума печінки.

B. Солітарна аденома печінки.

C. Солітарна лепрома печінки.

D. Хронічний абсцес печінки.

E. Рак печінки.

6.Вилучений при апендектомії хробакоподібний відросток потовщений, покритий фібринозно-гнійним нальотом. Всі шари відростка інфільтровані гнійним ексудатом, слизова оболонка зруйнована. Ваш діагноз?

A. *Флегмонозно-виразковий апендицит з вогнищевою деструкцією слизового та підслизового шару

B. Простий апендицит

C. Флегмонозний апендицит

D. Гангренозний апендицит

E. Поверхневий апендицит

7. Хворий, що страждав на хронічний вірусний гепатит, помер від гострої постгеморагічної анемії, яка виникла на фоні кровотечі з варикозно розширених вен стравоходу. На розтині печінка різко зменшена в розмірах, щільно консистенції, поверхня дрібногорбиста. Мікроскопічна картина однорідна – тонкопетлиста сполучнотканинна сітка і дрібні несправжні часточки. Який морфогенетичний тип цироза має місце у

A. *Портальний цироз.

B. Постнекротичний цироз.

C. Змішаний цироз.

D. Вірусний цироз.

E. Біліарний цироз.

8.При гістологічному дослідженні гастробіоптатів виявлено потоншення слизової оболонки шлунку із зменшенням кількості залоз та значним розростанням сполучної тканини, проток залоз розширені; слизова оболонка інфільтрована лімфоцитами та плазматичеими клітинами. Який з перелічених діагнозів найбільш вірогідний?

A. *Хронічний значний атрофічний гастрит

B. Хронічний поверхневий гастрит

C. Хронічний значний атрофічний гастрит з кишковою метаплазією

D. Хронічний помірний атрофічний гастрит

E. Флегмона шлунку

9.При дослідженні апендиксу спостерігається дифузна лейкоцитарна інфільтрація всіх шарів стінки. Назвіть різновид апендициту.

A. *Гострий флегмонозний

B. Гострий простий

C. Некротичний

D. Гострий поверхневий

E. Гангренозний

10.При гістологічному дослідженні вилученої виразки шлунка в її дні знайдено фібринозно-лейкоцитарний ексудат, поширена зона фібриноїдного некрозу, нижче розташовані шари грануляційної та фиброзної тканини. Ваш діагноз:

A. *Хронічна виразка

B. Гостра виразка

C. Гостра ерозія

D. Малігнізована виразка

E. Флегмона шлунка

11.Апендикс довжиною 9 см, товщиною 0.9 см. Сероза тьмяна, повнокровна. Мікроскопічно - стінка набрякла, стази у капілярах і венулах та дрібни крововиливи; у слизовій і підслизовій оболонках – осередки некрозу с лейкоцитарною інфільтрацією навкруги їх.Який з перелічених діагнозів найбільш вірогідний?

A. * Гострий поверхневий апендицит.

B. Гострий простий апендицит.

C. Гострий флегмонозний апендицит.

D. Гострий флегмонозно-виразковий апендицит.

E. Гострий гангренозний апендицит.

12.При фiброгастроскопiї хворого 48 рокiв, водiя, зі скаргами на бiль в епiгастрiї пiсля їжи, слизова оболонка шлунку гiперемована, зморшки її зменшенi. Мiкроскопiчно в гастробiоптатi: слизова оболонка витончена, кiлькiсть залоз зменшена, розростання сполучної тканини, iнфiльтрованої лiмфоцитами, плазмоцитами. Вкажіть, який з перелічених діагнозів найбільш ймовірний.

A. *Хронiчний атрофiчний гастрит.

B. Гострий катаральний гастрит.

C. Гострий гнiйний гастрит.

D. Хронiчний поверхневий гастрит.

E. Гiгантський гiпертрофічний гастрит.

13. У хворої 46 років з ревматичною вадою серця – стенозом лівого атріовентрикулярного отвору - визначаються задуха при невеликому фізичному навантаженні, серцебиття, ціаноз губ, вологі хрипи в нижніх відділах легень, набряки на нижніх кінцівках. Які гістологічні зміни будуть характерні для печінки?

A. *Некроз гепатоцитів в центрі часточки, жирова дистрофія на периферії

B. Некроз гепатоцитів в центрі часточки, гіаліново-крапельна дистрофія на периферії

C. Некроз гепатоцитів в центрі часточки, гідропічна дистрофія на периферії

D. Жирова дистрофія гепатоцитів в центрі часточки, некроз на периферії

E. Гідропічна дистрофія гепатоцитів в центрі часточки, некроз на периферії

14. Чоловік 59 років мав ознаки паренхіматозної жовтяниці та портальної гіпертензії. Під час гістологічного дослідження пункційного біоптату печінки було знайдено: балково-часточкова будова порушена, частина гепатоцитів має ознаки жирової дистрофії, утворюються порто-портальні сполучнотканинні септи з формуванням псевдочасточок, з наявністю періпортальних лімфо-макрофагальних інфільтратів. Діагностуйте захворювання

A. * Цироз печінки

B. Алкогольний гепатит

C. Хронічний гепатоз

D. Вірусний гепатит

E. Токсичная дисрофія

15. Під час розтину тіла чоловіка 29 років, який тривалий час хворів на виразкову хворобу дванадцятипалої кишки були знайдені ознаки перитоніту, множинні стеатонекрози позаочеревинної жирової тканини та підшлункової залози, а в діляці тіла ії знайдений виразкоподібний дефект діаметром 5 мм і глибиною до 10 мм, краї якого містили некротичні маси. Діагностуйте ускладнення виразкової хвороби дванадцятипалої кишки.

A. *Пенетрація

B. Кровотеча

C. Стеноз

D. Перфорація

E. Малігнізація

16.У чоловіка віком 50 років, який гостро захворів, діагностовано дизентерію. Смерть наступила на 8-й день захворювання. Під час розтину виявлено потовщену стінку сигмовидної та початкових відділів прямої кишки, фібринозну плівку на поверхні слизової. Гістологічно: глибокий некроз слизової оболонки з просяканням некротичних мас фібрином. Який вид коліту мав місце?

A. *Дифтеритичний коліт

B. Катаральний коліт

C. Виразковий коліт

D. Гангренозний коліт

E. Фолікулярний коліт

17. При розтині чоловіка, який тривалий час зловживав алкоголем, печінка малих розмірів, щільна, дрібнобугриста. Мікроскопічно: псевдочасточки дрібні, розподілені вузькими прошарками сполучної тканини з лімфомакрофагальними інфільтратами; гепатоцити у стані великокрапельної жирової дистрофії. Який з перелічених діагнозів найбільш вірогідний?

A. *Алкогольний цироз.

B. Хронічний активний алкогольний гепатит.

C. Хронічний персистуючий алкогольний гепатит.

D. Токсична дистрофія печінки.

E. Жировой гепатоз.

18. При гістологічному дослідженні біоптату зі слизової оболонки шлунку жінки 50 років виявлено: потоншення слизової оболонки, зменшення кількості залоз, фокуси кишкової метаплазії; повнокрів’я, набряк та склероз строми; дифузна лімфоплазмоцитарна інфільтрація зі значними домішками полінуклеарних лейкоцитів. Встановити діагноз.

A. *Хронічний атрофічний гастрит в активній фазі

B. Хронічний атрофічний гастрит у неактивній фазі

C. Хронічний поверхневий гастрит

D. Гострий катаральний гастрит

E. Гострий фібринозний гастрит

19. При морфологічному дослідженні шлунку виявлено глибокий дефект стінки з ураженням м’язової болонки, проксимальний край якого підритий, дистальний - пологий. При мікроскопічному дослідженні: в дні дефекту виявляється зона некрозу, під якою грануляційна тканина та масивна ділянка рубцевої тканини на місці м’язового шару. Встановити діагноз.

A. *Хронічна виразка у стадії загострення

B. Хронічна виразка з малігнізацією

C. Гостра виразка

D. Ерозія

E. Рак-виразка

20. При мікроскопічному дослідженні оперативно видаленого апендикса відмічався набряк, дифузна нейтрофільна інфільтрація стінки з некрозом та наявністю дефекту слизової оболонки з ураженням її м’язової пластинки. Яка форма апендициту розвилась у хворого?

A. *Флегмонозно-виразкова

B. Флегмонозна

C. Гангренозна

D. Поверхнева

E. Апостематозна

21.При ендоскопії шлунку, на гістологічне дослідження взято біоптат слизової оболонки. Його дослідження виявило: слизова оболонка збережена, стовщена, набрякла, гіперемірована, з багато численними мілкими крововиливами, щільно вкрита слизом. Визначити форму гострого гастриту.

A. *катаральний (простий)

B. ерозівний

C. фібринозний

D. гнійний

E. некротичний

22. На гістологічне дослідження прислали видалений червоподібний відросток. Розміри його збільшені, серозна оболонка тускла, повнокровна, вкрита плівками фібрину, стінки стовщені, на розрізі із просвіту виділяється гній. При мікроскопічному дослідженні спостерігається повнокров’я судин, набряк всіх шарів і дифузна інфільтрація їх лейкоцитами. Назвіть форму апендициту.

A. *флегмонозний

B. апостематозний

C. простий

D. поверховий

E. гангренозний

23. Хворому 65 років проведена операція видалення з печінки круглястого утворення (d=9 см) зі щільними волокнистими фіброзними стінками. При розтині останнього всередині знайдено густу, каламутну, жовто-зеленого кольору рідину з неприємним запахом. Мікроскопічно знайдені поліморфні лейкоцити. Назвати процес за описаною морфологією.

A * Хронічний абсцес

B Гострий абсцес

C Кіста жовчних шляхів

D Емпієма

E Флегмона

24. Хворий Н. поступив в стаціонар з діагнозом отруєння грибами, де і помер на 12 день з ознаками гострої печінкової недостатності. На секції – макроскопічно: печінка дрябла, в’яла, жовто-сірого кольору, глинястого вигляду як на поверхні, так і на розтині; мікроскопічно: ділянки некрозу з аутолітичним розпадом і утворенням жиро-білкового детриту в центрі та вузькою смужкою гепатоцитів в стані жирової дистрофії на периферії печінкових часточок. Ваш діагноз?

A *Токсична дистрофія печінки в стадії жовтої дистрофії

B Токсична дистрофія печінки в стадії червоної дистрофії

C Хронічна токсична дистрофія печінки

D Хвороба Вільсона-Коновалова

E Спадковий пігментний гепатоз

25. При операції у хлопчика 12 р. видалений апендикс, який надіслано патологу на дослідження. Макроскопічно: апендикс в дистальному відділі з булавоподібним стовщенням діаметром 3 см, при розрізі якого вилилася прозора жовтувата рідина, стінка апендикса стоншена. Мікроскопічно: атрофія всіх шарів апендикса, ознак запалення немає. Діагноз?

A *Водянка апендикса

B Флегмонозний апендицит

C Емпієма апендикса

D Хронічний апендицит

E Міксоглобульоз апендикса

26. Хвора 38 р. поступила в клініку з температурою тіла 39,4 градуси за Цельсієм, різкими розлитими болями в епігастральній області. При операції прийнято рішення видалити весь шлунок. Гістопатологічне дослідження всіх анатомічних відділів шлунка показало розлите гнійно-деструктивне запалення всіх шарів стінки з утворенням численних дрібних абсцесів. Діагноз?

A *Флегмонозний гастрит

B Хронічний гастрит, тип А

C Хронічний гастрит, тип В

D Хронічний гастрит, тип С

E Хелікобактер-асоційований гастрит

27.Хворий 53 років поступив в клініку з невпинною блювотою "свіжою" кров’ю і через 1 годину помер. На розтині в нижній третині стравоходу -варикозно-розширені вени з кількома розривами. Патологія якого органа спричинала зміни у судинах стравоходу?

A * Печінки

B Нирок

C Підшлункової залози

D Легень

E Серця

28. До патогістологічої лабораторії доставлено червоподібний відросток товщиною до 2,0 см. Серозна оболонка його тьмяна, потовщена, вкрита жовто-зеленими плівковими нашаруваннями. Стінка в’яла, сіро-червона. Просвіт відростка розширено, заповнено жовто-зеленими масами. При гістологічному дослідженні виявлено, що: стінку інфільтровано нейтрофілами. Визначте захворювання апендикса, його форму.

A *Гострий флегмонозний апендицит

B Гострий гангренозний апендицит

C Гострий поверхневий апендицит

D Гострий простий апендицит

E Хронічний апендицит

29. Хворий 42 років потрапив до стаціонару зі скаргами на жовтуху, біль у правому підребір’ї. Занедужав після прийому великої дози алкоголю. При пункційній біопсії печінки: жирова дистрофія гепатоцитів, наявність в їх цитоплазмі еозинофільних включень, інфільтрація строми поліморфноядерними лейкоцитами; часточкова будова печінки збережена. Яке захворювання печінки у хворого?

A *Гострий алкогольний гепатит

B Хронічний алкогольний гепатит

C Жировий гепатоз

D Дрібновузловий портальний цироз

E Хвороба Вільсона

30. Після отруєння грибами у хворого розвинулися ознаки гострої печінкової недостатності, що призвело до його смерті. На розтині тіла померлого макроскопічно печінка зменшена, в’яла, капсула зморшкувата, на розрізі тканина охряно-жовта. Мікроскопічно: жирова дистрофія гепатоцитів, центральні відділи часточок представлені тканинним детритом. Виявлені зміни характерні для:

A *Масивного прогресуючого некрозу

B Жирового гепатозу

C Гострого ексудативного гепатиту

D Гострого продуктивного гепатиту

E Гепатолентікулярної дегенерації

31. Після проведеної холецистектомії у жінки 50 років жовчний міхур відправли до патогістологічної лабораторії для дослідження. Макроскопічно: міхур значно збільшений за розмірами, протока його закрита каменями, стінка повнокровна, рихла. В розширеній порожнині міхура скопичення каламутної густої в’язкої рідини жовто-зеленого кольору. Ці зміни найбільш характерні для:

A *Емпієми

B Мукоцеле

C Кісти

D Флегмони

E -

32. При терміновій операції у хворого було видалено червоподібний відросток, який був різко потовщений, впродовж всієї довжини сірувато-чорний, в дистальному відділі знайдено дефект стінки, через який з просвіту апендикса виділяються сірувато-бурі маси з неприємним запахом. При гістологічному дослідженні стінка апендикса некротизована з осередками крововиливів, просвіт брижової артеріїзаповнений тромбом. Визначте форму апендициту:

A *Гострий гангренозний

C Гострий простий

D Гострий поверхневий

E Хронічний

33. Чоловік 58 років помер від гострого малокрів´я внаслідок стравохідно-шлункової кровотечі. На розтині вени підслизового шару нижньої третини стравоходу і кардіального відділу шлунку варікозно розширені. Печінка маленька, щільна, бугриста, діаметр вузликів 2-3 мм; мікроскопічно часточкова будова печінки порушена, виражені дистрофічні зміни і вогнищевий некроз гепатоцитів. Значна кількість “псевдочасточок”, розростання в стромі прошарків сполучної тканини. Морфологічні зміни в печінці характерні для:

A * Дрібновузлового цирозу печінки

B Крупновузлового цирозу печінки

C Змішаного цирозу печінки

D Хронічного активного гепатиту

E Біліарного цирозу печінки

34. При гістологічному дослідженні видаленого під час операції апендикса виявлені поширені дісциркуляторні зміни з фокусами ексудативного гнійного запалення і поверхневими дефектами епітелію. Назвіть форму апендициту.

A *Гострий поверхневий

B Флегмонозний

C Гангренозний

D Вторинний

E Простий

35. Хворому з хронічною виразковою хворобою без загострення виконана фіброгастроскопія з біопсією слизової оболонки. Які морфологічні зміни в біоптаті свідчитимуть про стадію ремісії виразкової хвороби?

A * Розростання сполучної тканини

B Фібриноїдний набряк

C Фібриноїдні зміни стінок судин

D Наявність гнійно-некротичного детриту

E Зона фібриноїдного некрозу в дні виразки

36. Хворому з діагнозом рак шлунку проведено його видалення. При патогістологічному дослідженні виявлено: стінка шлунка отовщена, щільна, нерухома. Пухлина росте ендофітно в слизовому, підслизовому і м'язовому шарах по ходу сполучно-тканинних прошарків. Назвіть клініко-анатомічну форму раку.

A *Дифузний

B Інфільтративно-виразковий

C Полипозний

D Фунгозний

E Блюдцеподібний

37. Через 1,5 місяця після татуювання, при виконанні якого використовувалась одна голка для кількох чоловік, у хворого з’явилась жовтяниця, печінка збільшена у розмірах. При пункцій ній біопсії виявлена балонна дистрофія і коагуляцій ний некроз гепатоцитів, проліферація ендотеліоцитів. Поставте діагноз.

A *Вірусний гепатит

B Жировий гепатоз

C Медикаментозний гепатит

D Токсична дистрофія печінки

E Цироз печінки

38. Розтинаючи труп жінки, яка тривалий час страждала жовчево-кам’яною хворобою, і померла від печінкової недостатності внаслідок цирозу печінки, патологоанатом виявив: печінка збільшена, зеленого забарвлення, щільної консистенції, жовчні протоки розширені. Назвіть вид цирозу

A *Біліарний

B Постнекротичний

C Портальний

D Змішаний

E Алкогольний

39. При гістологічному дослідженні печінки відмічається різке порушення часточкової будови з інтенсивним фіброзом і формуванням вузлів регенерації. Мікроскопічно: проліферація гепатоуитів, поява псевдочасточок, дистрофія і некроз гепатоцитів. Ваш діагноз?

A * Жировий гепатоз

B Алкогольний гепатит

C Цироз печінки

D Вірусний гепатит

E Прогресуючий масивний некроз печінки

40. У хворого під час операції з приводу хронічного апендициту виявлені розростання слизоподібної тканини на очеревині. Червоподібний відросток кістозно розширений, заповнений слизовими масами у виді глобул. Вкажіть найбільш імовірне захворювання:

A *Мукоцеле червоподібного відростку

B Флегмонозний апендицит

C Апостематозний апендицит

D Первинний гангренозний апендицит

E Простий апендицит

41. На автопсії людини 56 років, яка довго страждала на виразкову хворобу шлунку виявлена виразка на малій кривизні з численними розростаннями щільної рубцевої тканин та потовщеної слизової оболонки біля виразки. При гістологічному дослідженні виявляється різко виражений клітинний атипізм. Ваш діагноз:

A *Малігнізована хронічна виразка шлунку

B Виразковий рак шлунки

C Метастаз раку легені у шлунок

D Перфорація виразки шлунку

E Пенетрація виразки шлунку

42. Жовчний міхур, який прислали до патоморфологічного відділення після холецистектомії збільшений у розмірах, повнокровний, стінка набрякла, дифузно інфільтрована нейтрофільними лейкоцитами. При якій формі холециститу розвиваються такі зміни:

A *Флегмонозному

B Катаральному

C Фібринозному

D Гангренозному

E Апостематозному

43. У чоловіка 42 років, який страждає на виразкову хворобу шлунку при ендоскопічному дослідженні діагностовано хронічний гастрит типу С. Такий тип гастриту називається:

A *Еозинофільний

B Аутоімунний

C Бактеріальний

D Хімічний

E Гранулематозний

44. Жінка 63 років поступила в стаціонар с явищами геморагічного шоку, в результаті якого хвора померла. На розтині в цибулині 12-палої кишки виявлено дефект 2х2 см з бурим дном. Мікроскопічно: некроз слизового, підслизового шару з відкладанням солянокислого гематину в дні і лейкоцитарною інфільтрацією країв.

A *Гостра виразка 12-палої кишки.

B Ерозія 12-палої кишки

C Хронічна виразка 12-палої кишки

D Гострий дуоденіт

E Хронічний дуоденіт.

45. При ендоскопічному дослідженні шлунка у хворого, який довго приймав сульфаніламідні препарати, на малій кривизні на внутрішній поверхні щік знайдені дефекти, що мають буровате дно. Мікроскопічно встановлено, що вони не проникають за межі м´язевої пластинки слизової оболонки, краї дефектів дифузно інфільтровані лейкоцитами. Який з перерахованих патологічних процесів найбільш вірогідний?

A * Гостра ерозія

B Гостра виразка

C Хронічна виразка в стані ремісії

D Хронічна виразка в стані загострення

46. При дослідженні гастробіоптату хворого 59 років, що страждав протягом 23 років виразковою хворобою шлунка, виявлено наявність гіперхромних клітин з вираженим атипізмом, які лежать серед пластів і тяжів грубоволокнистої сполучної тканини, з переважанням стромального компоненту. Який з перерахованих діагнозів найбільш вірогідний?

A * Скір

B Перстневидно-клітинний рак

C Медулярний рак

D Папілярна аденокарцинома

E Солідний рак

47. Під час мікроскопічного дослідження печінки чоловіка 56 років, що помер від печінкової недостатності, було знайдено: балкова будова часточок порушена, гепатоцити з ознаками гідропічної дистрофії, зустрічаються вогнищеві некрози гепатоцитів, множинні тільця Каунсільмена, періпортально розміщуються поліморфноклітинні інфільтрати з переважанням лімфоцитів. Діагностуйте захворювання.

A * Гострий вірусний гепатит „В”.

B Хронічний вірусний гепатит „В”.

C Алкогольний гепатит.

D Портальний цироз печінки.

E Біліарний цироз печінки

48. При гістологічному дослідженні апендиксу, який був резекований при операції апендектомії, виявлено множинні виразки у слизовій оболонці відростка, дифузну інфільтрацію сегменто-ядерними нейтрофілами всіх шарів стінки апендиксу, набряк строми, повнокрів’я судин. На серозній оболонці – нашарування фібрину і лейкоцитів. Вкажіть, якій формі гострого апендициту відповідають приведені патоморфологічні зміни.

A * Флегмонозно-виразковий.

B Апостематозний.

C Флегмонозний.

D Гангренозний.

E Поверхневий.

49. 54-річній жінці проведено лівобічну геміколектомію з приводу перфорації кишки. При гістологічному дослідженні операційного матеріалу виявлено множинні епітеліоїдно-клітинні гранульоми, щілиноподібні виразки, виражений набряк підслизового шару. Діагностуйте захворювання.

A * Хвороба Крона.

B Туберкульоз кишки.

C Неспецифічний виразковий коліт.

D Хвороба Уіпла.

E Лімфома кишки.

50. У хворого з печінковою комою при вірусному гепатиті в крові підвищилась концентрація Na+ та знизилась концентрація К+, тобто почались прояви гіперальдостеронізму. Чим обумовлено підвищення дії альдостерону у хворого?

A *Зменьшенням його метаболізму.

B Підвищенням його сінтезу альдостерону.

C Збільшенням кіоткості рецепторів до нього.

D Порушенням його транспорту.

E Зменьшенням його виведення.

51. У чоловіка віком 65 років, у якого виник цироз печінки, спостерігалось значне зниження артеріального тиску крові. Який механізм може обумовити артеріальну гіпотензію у цьому випадку?

A *Зменшення синтезу ангіотензиногену в печінці.

B Зменшення утворення жовчних кислот в печінці.

C Зменшення синтезу сечовини в печінці.

D Зменшення антитоксичної функції печінки.

E Зменшення синтезу транспортних белків в печінці.

52. В експерименті при моделювання печінкової патології у тварин виникали брадикардія, зниження артеріального тиску, ознаки пригнічення нервової системи. Для якої патології печінки характерна така сукупність ознак?

A *Холемічний синдром.

B Ахолічний синдром.

C Синдром портальної гіпертензії.

D Гепаторенальний синдром.

E Дисхолія.

53. У хворого після отруєння грибами різко погіршився загальний стан: спостерігається втрата свідомості, випадіння рефлексів. Порушення якої функції печінки стало причиною даних змін?

A * Зниження антитоксичної

B Порушення фагоцитарної

C Підвищення метаболічної

D Зниження секреторної

E Порушення гемодинамічної

54. У хворого 39 років при фіброгастроскопічному обстеженні в пілоричному відділі шлунка виявлена виразка розміром 1,5х1 см. Проба на наявність Helicobacter pylori позитивна. Роль бактеріального обсіменіння слизової шлунка в патогенезі виразки пов’язана з

A *Порушенням слизового бар'єру

B Підвищенням секреції кислоти

C Підвищенням секреції ферментів

D Розвитком запалення

E Ушкодженням клітин слизової

55. При сонографії у хворого виявлено дифузний цироз печінки. Вміст цукру в крові становить 3,6 ммоль/л. Причиною розвитку гіпоглікемії було зниження

A *Глікогену в печінці

B Глікогенолізу

C Гліконеогенезу

D Гліколізу

E Засвоєння глюкози

Хвороби нирок і сечового міхура.

1. Чоловік, 19 років страждав з раннього дитинства бронхо-ектатичною хворобою. Помер від ниркової недостатності. На розтині окрім множинних бронхоектатичних каверн, заповнених гнійним ексудатом виявлено збільшені в розмірах нирки щільної консистенції, кірковий шар потовщений, білого кольору, щільний. Піраміди нирки анемічні, чіткі. Назвати процес, який розвився в нирках ?

A. *Вторинний амілоїдоз

B. Гломерулонефрит

C. Хронічний пієлонефрит

D. Вроджений кістоз нирок

E. Вторинний нефросклероз

2. Хворий 67 років протягом 20 років страждав гіпертонічною хворобою. Помер від хронічної ниркової недостатності. Який вид мали нирки при розтині трупа?

A. *Маленькі, щільні, поверхня дрібнозерниста.

B. Великі строкаті

C. Великі червоні

D. Великі білі

E. Великі з множинними тонкостінними кістами

3. На розтині у померлого хворого виявлена аденома передміхурової залози і великі нирки з різко збільшеними баліями і чашечками, заповненими прозорою рідиною. Назвіть процес у нирках.

A. *Гідронефроз

B. Гломерулонефрит

C. амілоїдоз

D. Туберкульоз

E. Пієлонефрит

4. Хворий на цукровий діабет помер від хронічної ниркової недостатності ( ХНН ) з розвитком уремії. Під час секціїї виявлені всі прояви уремії з змінами в нирках. Які найімовірніші мікроскопічні зміни мають місце в нирках при цьому ?

A. * Гіаліноз та склероз капілярів клубочків.

B. Спазм приносних артеріол.

C. Гіалінові циліндри в просвіті канальців нефрону.

D. Некроз епітелію канальців нефрону.

E. Вогнищевий некроз коркової речовини нирок.

5. У молодої особи на протязі року прогресувала ниркова недостатність зі смертельним наслідком. При розтині виявлені великі строкаті нирки з червоними дрібними краплинами в жовто-сірому корковому шарі. Гістологічно клубочках виявлені "півмісяці" з проліферуючого нефротелія. Ваш діагноз?

A. *Підгострий гломерулонефрит

B. Амілоїдоз

C. Крововилив в нирки

D. Гнійний нефрит

E. Рак нирки

6. У 60-річної померлої, яка протягом тривалого часу хворіла на гіпертонічну хворобу, на розтині нирки значно зменшені ( вага обох нирок 80 г), поверхня дрібнозерниста, на розрізі кора рівномірно потоньшена. Як можна назвати зміни в нирках ?

A. * Первинно - зморщені нирки

B. Пієлонефритично-зморщені нирки

C. Вторинно- зморщені нирки

D. Амілоїдно-зморщені нирки

E. Диабетичний нефросклероз

7. При мікроскопічному дослідженні нирок виявлений некроз епітелія звивистих канальців, тубулорексис, набряк строми, ішемія коркового шару і повнокрів’я мозкового. Назвіть цей патологічний процес.

A. *Некротичний нефроз

B. Гломерулонефрит

C. Пієлонефрит

D. Полікистоз

E. Сечокам’яна хвороба

8. На секції виявлено значне збільшення об’єму правої нирки. На розрізі в ній міститься камінь. Просвіт ниркової миски розтягнутий сечею, яка накопичується. Паренхіма нирки різко потоншана. Який з діагнозів є найбільш вірним?

A. * Гідронефроз.

B. Пієлоектазія

C. Гідроуретронефроз.

D. Кіста нирки

9. На аутопсії померлої, яка тривалий час хворіла цистітом і дискінезією сечоводів, виявлено морфологічні ознаки уремії. Нирка була нерівномірно рубцево-зморщена. У просвіті мисок містилися дрібні уратні камені і пісок. Гістологічно виявлено “щитовидну нирку”, вогнища інтерстиційного запалення. Який із нижчеперерахованих діагнозів є найбільш вірогідний?

A. *Хронічний пієлонефрит

B. Гострий пієлонефрит

C. Атеросклеротично зморщена нирка

D. Первинно зморщена нирка

E. Амілоїдно зморщена нирка

10. На розтині прозектор знайшов зміни в нирках: збільшення в розмірах, корковий шар широкий, жовто-сірий з червоним крапом, добре відмежований від мозкової речовини, яка має темно-червоний колір. При гістологічном дослідженні - розростання нефротелія і подоцитів в капсулі клубочків з утворенням “ півмісяців”, явища склерозу і гіалінозу клубочків, фіброз строми. Про яке захворювання нирок йде мова?

A. *Екстракапілярний продуктивний гломерулонефрит

B. Інтракапілярний продуктивний гломерулонефрит

C. Екстракапілярний ексудативний гломерулонефрит

D. Інтракапілярний ексудативний гломерулонефрит

E. Інтерстеційний нефрит

11. Хворий тривалий час хворів на гіпертонічну хворобу і помер від крововиливу в мозок.При розтині нирки потончены в розмірах, щільної консистенції, поверхня їх дрібногранулярна, кіркова речовина дещо стоншена. Як мікроскопічні зміни характерні для нирки?

A. *Первинно зморщена нирка

B. Вторинно зморщена нирка

C. Амілоїдоз нирки

D. Пієлонефритично зморщена нирка

E. Інфаркт нирки

12. У хворого цукровим діабетом з'явились ознаки нарастаючої ниркової недостатності. Який процес в нирках є причиною

A. *Гломерулосклероз.

B. Некроз епітелію звивистих канальців.

C. Амілоїдоз.

D. Інтерстиційним

E. Мембранозна нефропатія.

13. Хворий на фіброзно-кавернозний туберкульоз легень помер від хронічної легенево-серцевої недостатності. Протягом останніх місяців спостерігалась виражена протеїнурія. На розтині: нирки збільшені, щільні, восковидні поверхні і на розтині. Які зміни в нирках при цій формі туберкульозу могли виникнути?

A. *Амілоїдоз нирок.

B. Туберкульоз нирок.

C. Гломерулонефрит.

D. Нефролітіаз.

E. Некротичний нефроз.

14. Хворий, який страждав хронічним гнійним остеомієлітом, помер від хронічної ниркової недостатності. При розтині знайдені великі щільні нирки біло-жовтого кольору з сальним блиском на розрізі. Ваш діагноз:

A. * Амілоїдоз нирок

B. Хронічний гломерулонефрит

C. Підгострий гломерулонефрит

D. Септичний нефрит

E. Гострий некротичний нефроз

15. Хворий на фіброзно-кавернозний туберкульоз помер при зростаючих явищах ниркової недостатності. На розтині – запах сечі, гіпертрофія лівого шлуночка, фібринозний перикардит, фібринозно-геморагічний ентероколіт. Нирки дещо зменшені в розмірах, дуже щільні, з множинними втягненнями. Гістологічно на препаратах, забарвлених Конто-рот - рожеві маси у клубочках та стінках судин, загибель і атрофія більшості нефронів, нефросклероз. Дайте характеристику нирки при даній картині?

A. *Амілоїдно зморщені нирки

B. Первинно зморщені нирки

C. Вторинно зморщені нирки

D. Атеросклеротично зморщені нирки

E. Пієлонефритично зморщені нирки

16. При розтині тіла жінки 40 років, померлої від уремії, знайдено: збільшені пістряві нирки, у нирках – потовщення капілярних мембран клубочків у вигляді “дротяних петель”, осередки фібриноїдного некрозу їх стінок та “гіалінові” тромби у просвітах, у ядрах – “гематоксилінові тільця”; у серці – ендокардит Лібмана-Сакса. Яке ураження нирок найбільш вірогідне?

A. *Вовчаковий нефрит

B. Ревматичний гломерулонефрит

C. Холерний гломерулонефрит

D. Склеротична нирка

E. Термінальний гломерулонефрит

17. На розтині виявлено, що нирки збільшені в розмірах, поверхня крупно-горбиста за рахунок наявності числених порожнин з гладкою стінкою, заповнених прозорою рідиною. Про яке захворювання йде мова?

A. *Полікістоз

B. Некротичний нефроз

C. Пієлонефрит

D. Гломерулонефрит

E. Інфаркт

18.Жінка віком 49 років, тривалий час хворіла хронічним гломерулонефритом, який привів до смерті. На розтині встановлено, що нирки мають розміри 7х3х2.5 см, масу 65,0 г, щільні, дрібнозернисті. Мікроскопічно: фібринозне запалення серозних і слизових оболонок, дистрофічні зміни паренхіматозних органів, набряк головного мозку. Яке ускладнення привело до вказаних змін?

A. *Уремія

B. Анемія

C. Сепсис

D. ДВЗ-снндром

E. Тромбоцитопенія

19. Чоловік хворів на цукровий діабет 15 років, помер від крововиливу у мозок. На розтині нирки зменшені у розмірах, поверхня дрібнозерниста. Епіпелій канальців дистального сегменту високий, зі світлою пінявою цитоплазмою, при забарвленні карміном Беста та при ШИК - реакції - яскраво-червоного кольору. Зміни в епітелії зумовлені накопиченням:

A. *Глікогену

B. Ліпідів

C. Гіаліну

D. Протеїнів

E. Амілоїду

20. При розтині чоловіка, померлого від опікової хвороби, знайдено набряк головного мозку, збільшення печінки, а також нирок, корковий шар яких широкий, блідо-сірий, мозковий – повнокровний. Мікроскопічно: некроз канальців головних відділів з деструкцією базальних мембран, набряк інтерстицію з лейкоцитарною інфільтацією та крововиливами. Який з перелічених діагнозів найбільш вірогідний?

A. *Некротичний нефроз.

B. Тубулоінтерстиціаальний нефрит.

C. Пієлонефрит.

D. Подагрична нирка.

E. Мієломна нирка.

21. В біоптаті нирки: склероз, лімфо-плазмоцитарна інфільтрація стінок мисок та чашок; дистрофія та атрофія канальців. Збережені канальці розширені, розтягнені колоїдоподібними масами, епітелій сплюснутий (“щитовидна” нирка). Який діагноз найбільш вірогідній?

A. * Хронічний пієлонефрит

B. Гострий пієлонефрит

C. Гломерулонефрит

D. Нефросклероз

E. Тубуло-інтерстиціальний нефрит

22. У чоловіка 60 років, який тривалий час хворів на гіпертонічну хворобу і помер від хронічної ниркової недостатності, при автопсії виявлено, що обидві нирки значно зменшені у розмірах, поверхня їх дрібно-зерниста; гістологічно - більшість клубочків гіалінізована, частина склерозована, окремі гіперплазовані; в стромі поля склерозу, артеріоло- та артеріосклероз, еластофіброз великих гілок ниркових артерії. Назвіть виявлені зміни.

A. *Артеріолосклеротичний нефросклероз

B. Атеросклеротичний нефросклероз

C. Хронічний гломерулонефрит

D. Хронічний пієлонефрит

E. Вторинно зморщена нирка

23. У чоловіка 42 років, що хворів на важку форму черевного тифу, розвинулась гостра ниркова недостатність, від якої він помер. При автопсії нирки збільшені у розмірах, набряклі, фіброзна капсула знімається легко; на розрізі кора – блідо-сіра, піраміди темно-червоні. При гістологічному дослідженні - у більшості канальців просвіт звужений, епітеліальні клітини збільшені у розмірах без ядер; клубочки колабовані; у стромі набряк, невелика лейкоцитарна інфільтрація, дрібні геморагії. Про яку патологію нирок йде мова?

A. *Некронефроз

B. Гострий пієлонефрит

C. Гострий гломерулонефрит

D. Піонефроз

E. Гідронефроз

24. При мікроскопічному дослідженні нирок спостерігається проліферація нефротелія капсули Боумена, подоцитів і макрофагів з утворенням в її просвіті “півмісяцевих” структур, які стискують клубочок. Капіляри клубочків зазнають некрозу, в їх просвіті спостерігаються фібринові тромби. Деякі клубочки склерозуються або гіалінізуються. Спостерігається виражена дистрофія нефроцитів, набряк та інфільтрація строми нирок. Назвіть патологію нирок.

A. * Швидкопрогресуючий гломерулонефрит

B. Післяінфекційний гломерулонефрит

C. Хронічний гломерулонефрит

D. Хронічний пієлонефрит

E. Амілоїдоз нирок

25. У хворого, який помер від хронічної ниркової недостатності на грунті хронічного гломерулонефрита нирки зменшені в розмірах, щільної консистенції, капсула знімається важко, оголяючи зернисту поверхню. На розрізі кірковий та мозковий шари тонкі, тканина нирок суха, недокрівна, сірого кольору. Як називається така нирка?

A. * Вторинно-зморщені нирки

B. Первинно-зморщені нирки

C. Атеросклеротично-зморщені нирки

D. Амілоїдно-зморщені нирки

E. Пієлонефротично-зморщені нирки

26. У трьохрічної дівчинки, після перенесеної гострої респираторної вірусної інфекції з’явилися розповсюджені набряки (анасарка), масивна протеїнурія, гіпоальбумінемія, гіперліпідемія. При дослідженні біоптату нирки виявлено: відсутність малих відростків подоцитів судинних клубочків. Який найбільш вірогідний діагноз?

A. *Ліпоїдний нефроз

B. Післяінфекційний гломерулонефрит

C. Швидкопрогресуючий гломерулонефрит

D. Фокальний сегментарний гломерулосклероз

E. Мембранозний гломерулонефрит

27. На аутопсії хворого, померлого від отруєння етиленгліколем, нирки дещо збільшені у розмірах, набряклі, їх капсула знімається дуже легко, кіркова речовина широка, блідо-сіра, мозкова речовина - темно-червона. Яка патологія нирок розвинулась у хворого?

A. *Некротичний нефроз

B. Гострий пієлонефрит

C. Гострий гломерулонефрит

D. Гострий тубуло-інтерстиціальний нефрит

E. Ліпоїдний нефроз

28. У дитини 15 років через 14 днів після перенесеної ангіни з’явилися набряки на обличчі зранку, підвищення артеріального тиску, сеча у вигляді “м’ясних помиїв”. Імуногістохімічне дослідження біоптату нирки виявило відкладання імунних комплексів на базальних мембранах капілярів та у мезангії клубочків. Яке захворювання розвинулось у хворого?

A. *Гострий гломерулонефрит

B. Гострий інтерстіційний нефрит

C. Ліпоїдний нефроз

D. Гострий пієлонефрит

E. Некротичний нефроз

29. Хворий 30 р. поступив до лікарні зі скаргами на головний біль, запаморочення. Протягом останнього року визначаються високі показники артеріального тиску. Лікарські засоби майже не допомагали. У біоптаті нирки виявлено екстракапілярну проліферацію з утворенням “півмісяців” Для якого захворювання це характерно?

A. * Швидкопрогресуючий гломерулонефрит

B. Гострий гломерулонефрит

C. Вовчаковий гломерулонефрит

D. Гранулематоз Вегенера

E. Хронічний гломерулонефрит

30. При розтині померлого від ниркової недостатності спостерігались наступні зміни: нирки збільшені, дряблі, корковий шар широкий, набряклий, жовто-сірий, тьмяний, з червоним крапом, добре відмежований від темно-червоної мозкової речовини. Мікроскопічно: пролиферациія єпітелія капсули клубочків, подоцитів та макрофагів з формуванням „півмісяців”. Для якого захворювання характерні вищезазначені зміни?

A. * Підгострий гломерулонефрит.

B. Гострий гломерулонефрит.

C. Хронічний гломерулонефрит.

D. Гострий пієлонефрит.

E. Амілоїдоз нирок.

31. При морфологічному дослідженні видаленої нирки виявлено наявність в проксимальному відділі сечовіду конренкремента, який обтурує його просвіт, нирка різко збільшена, перенхіма атрофована, чашки та миска значно розширені. Мікроскопиічно: дифузний склероз, атрофія клубочків та канальців, збережені канальці кістозно розширені. Яке ускладення нирковокам’яної хвороби виникло у хворого?

A. * Гідронефроз.

B. Піонефроз.

C. Пієлонефрит.

D. Гломерулонефрит.

E. Хронічний паранефрит.

32. У хворого, який помер від ускладнень гіпертонічної хвороби на розтині виявлені маленькі, щільні нирки з дрібнозернистою поверхністю, паренхіма та коркова речовина атрофічні. Дайте назву таким ниркам.

A. *Первинно-зморщені нирки

B. Амилоїдно-зморщені нирки

C. Вторинно-зморщені нирки

D. Пієлонефритично-зморщені нирки

E. -

33. На розтині чоловіка 62 років шкіра сіро-земельного кольору з дрібноточковими крововиливами, обличчя мов припудрене білуватим порошком., фібринозно-геморагічний ларингіт, трахеїт, фібринозний перікардит, катаральний гастроентероколіт. Для якого синдрому характерний даний комплекс морфологічних

A. * хронічної ниркової недостатності

B. гострої печінкової недостатності

C. гострої ниркової недостатності

D. хронічної серцевої недостатності

34. Чоловік 40 років протягом 10 років після перелому великої гомілкової кістки страждав на хронічний остеомієліт. Три роки назад з’явився нефротичний синдром. Смерть наступила від уремії. На секції: нирки, щільні, білі, з рубцями в кірковому шарі, на розрізі з сальним блиском. Вкажіть патологію нирок, яка розвинулася.

A. *Вторинний амілоїдоз

B. Первинний амілоїдоз

C. Ідеопатичний амілоїдоз

D. Хронический гломерулонефрит

E. Хронічний пієлонефрит

35. На аутопсії померлого від ниркової недостатності відмічено, що нирки збільшені, кволі, широка кора жовто-сірого кольору з червоним крапом. При мікроскопічному дослідженні виявлено: епітелій капсули клубочків проліферує з появою “напівмісяців”, капілярні петлі з осередками некрозу та фібриновими тромбами у просвітах. Який з перелічених діагнозів найбільш вірогідний?

A. *Підгострий гломерулонефрит

B. Гострий гломерулонефрит

C. Ліпоїдний нефроз

D. Хронічний гломерулонефрит

E. Амілоїдоз нирок

36. На аутопсії померлого від хронічної ниркової недостатності відмічено, що нирки збільшені, щільні, воскоподібні, з великою кількістю западень на їх поверхні. Мікроскопічно: багато клубочків заміщені конго-рот-позитивною речовиною, в інших – ця речовина присутня на базальних мембранах капілярів, в мезангії, а також в стінках артерій та стромі. Який з перелічених діагнозів найбільш вірогідний?

A. *Амілоїдоз нирок

B. Гострий гломерулонефрит

C. Хронічний гломерулонефрит

D. Підгострий гломерулонефрит

E. Ліпоїдний нефроз

37. На розтині жінки віком 56 років, що померла від ниркової недостатності, нирки нерівномірно зменшені у розмірах, великобугриті з поверхні; на розрізі ділянки рубцової тканини чергуються з незмінною паренхимою, миски розширені, стінки їх потовщені. Мікроскопічно в стінках мисок, чашок та в інтерстиції – явища склерозу та лімфо-плазмоцитарної інфільтрації. Який з перелічених діагнозів найбільш вірогідний?

A. *Хронічний пієлонефрит

B. Гострий пієлонефрит

C. Гострий гломерулонефрит

D. Тубулоінтерстиційний нефрит

E. Хронічний гломерулонефрит

38. У хворого, який знаходиться у клініці з ознаками отруєння ртуттю, у нирках відмічаються наступні процеси: вогнищеві некротичні зміни канальців головних відділів, набряк, лейкоцитарна інфільтрація та геморагії інтерстицію, венозний застій. Який стан розвився у хворого?

A. *Гострий некротичний нефроз

B. Гострий гломерулонефрит

C. Хронічна ниркова недостатність

D. Гострий пієлонефрит

E. Хронічний пієлонефрит

39. При мікроскопічному дослідженні нирок померлої від ниркової недостатності жінки 36 років, в клубочках виявлено проліферацію нефротелію капсули, подоцитів та макрофагів з утворенням “півмісяців”, некроз капілярних петель, фібринові тромби в їх просвітах, а також склероз та гіаліноз клубочків, атрофію канальців та фіброз строми нирок. Який з перелічених діагнозів найбільш вірогідний?

A. *Підгострий гломерулонефрит

B. Гострий гломерулонефрит

C. Хронічний гломерулонефрит

D. Фокальний сегментарний склероз

E. Мембранозна нефропатія

40. Хворий помер при явищах уремії. На розтині нирки збільшені, в’ялі, кірковий шар широкий, набряклий, з червоним крапом, мозкова речовина темно-червона. Мікроскопічно в порожнині капсули клубочків виявлені “півмісяці”, які стискають капіляри, дистрофія нефроцитів, набряк та інфільтрація строми. Яке захворювання у померлого?

A. *Гломерулонефрит

B. Пієлонефрит

C. Нефролітіаз

D. Нефротичний синдром

E. Амілоїдоз нирок

41. У хворого 27 років з вираженим нефротичним синдромом, втратою білка більше 3 г на добу, при пункційній біопсії знайдено мінімальну проліферацію мезангіальних клітин, потовщення базальної мембрани капілярів клубочків, різка дистрофія епітелію канальців, до некрозу їх поодиноких клітин. Виберіть правильний діагноз:

A * Мембранозний гломерулонефрит

B Ідіопатичний некротичний синдром

C Мезангіальний проліферативний гломерулонефрит

D Мембранозно-проліферативний гломерулонефрит

E Амілоїдоз нирок.

42. У хворого з гломерулонефритом в біоптаті нирок виявлений 21 клубочок, 3 з яких повністю гіалінізовані, в інших виражена проліферація ендотеліальних і мезангіальних клітин, значне потовщення і розщеплення базальної мембрани капілярів клубочка, зрощення їх з капсулою клубочка. В канальцах значні дистрофічні зміни до некрозу епітелія, атрофія великої кількості канальців. В стромі виражена лімфо-гістіоцитарна інфільтрація, склероз строми. Які з названих змін в даному випадку мають найбільш важливе прогностичне значення?

A *Вираженість тубуло-інтерстиціального компоненту

B Гіаліноз клубочків

C Вираженість проліферації клітин клубочків

D Потовщення базальних мембран капілярів клубочків

E Зрощення капілярів клубочків з їх капсулою.

43. Прводячи розтин трупа чоловіка 46-ти років, який тривалий час хворів на хронічний гломерулонефрит, патологоанатом виявив: нирки 6,5 на 3 на 2,5 см., масою 75гр., щільної консистенції, поверхня дрібнозерниста; набряк головного мозку; фібринозне запалення серозних слизових оболонок; дистрофічні зміни паренхіматозних органів. Як називається описана макроскопічна артина нирок?

A *Вторинно зморщені нирки

B Первинно зморщені нирки

C Артеріолонефросклероз

D Амілоїднозморщені нирки

E Артеріонефросклероз

44. У хворої 59-ти років, яка тривалий час хворіла ревматоїдним поліартритом і померла від уремії, на розтині виявлено: нирки щільної консистенції, на розрізі мають „сальний” вигляд, кора біла, піраміди червоні, розміри нирок збільшені.Назвіть ускладнення ревматоїдного артриту.

A *Амілоїдоз нирок

B Гострий гломерулонефрит

C Хронічний гломерулонефрит

D Хронічний пієлонефрит

E Тубуло-інтерстиціальний нефрит

45. Хворий 60-ти років тривалий час страждав на фіброзно-кавернозний туберкульоз легень, що ускладнився емпіємою плеври. На протязі останнього року з’явилась і наростала ниркова недостатність. Помер хворий від масивної легеневої кровотечі. Яка причина розвитку хронічної ниркової недостатності:

A *Вторинний амілоїдоз

B Туберкульоз сечо-статевої системи

C Хронічний гломерулонефрит

D Нефросклероз

E Хронічний пієлонефрит

46. У хворого з масивними опіками розвинулась гостра недостатність нирок, що характеризується значним і швидким зменшенням швидкості клубочкової фільтрації. Який механізм його розвитку?

A *Зменшення ниркового кровоплину

B Ушкодження клубочкового фільтра

C Зменшення кількості функціонуючих нефронів

D Збільшення тиску канальцевої рідини

E Емболія ниркової артерії

47. У дитини, хворої на дифтерію розвинулася гостра ниркова недостатність. При розтині макроскопічно нирка нерівномірного кровонаповнення: блідий кірковий шар і повнокрівні піраміди. Мікроскопічно: некроз епітелію проксимальних і дистальних канальців, тубулорексис. Вкажіть ускладнення, яке спричинило смерть.

A *Гостра ниркова недостатність

B Гострий гломерулонефрит

C Гострий пієлонефрит

D Первинна нефропатія

E Нефротичний синдром

48. У підлітка через 2 тиждня після перенесеної стрептококової ангіни розвинулась клініка гострої ниркової недостатності, в результаті якої наступила смерть. При аутопсії нирки збільшені в розмірах, набряклі з сіруватим крапом. Мікроскопічно – фібриноїдний некроз і тромбоз капилярів клубочків, інфільтрація їх нейтрофілами. Яке захворювання має місце?

A * Гострий гломерулонефрит.

B Тубуло-інтерстиціальний нефрит.

C Некротичний нефроз

D Підгострий гломерулонефрит

E Гострий пієлонефрит

49. З анамнезу хворого 65 років, померлого від уремії, відомо, що він близько 15 років тому переніс гострий гломерулонефрит. На розтині нирки різкозменшені в розмірах, щільні, поверхня їх дрібнозерниста, на розрізі корковий шар тонкий, тканина нирок сіруватого кольору, суха, анемічна. Який з перерахованих діагнозів найбільш вірогідний?

A * Хронічний гломерулонефрит

B Гіпертонічна хвороба

C Атеросклероз

D Хронічний пієлонефрит

E Діабетичний гломерулосклероз

50. У хворого на хронічний тонзилит з’явились набряки під очима, підвищена втомлюваність. Артеріальтний тиск 170/110 мм рт.ст. У сечі виявлено білок – 0,990/00. Про яку патологію слід думати?

A * Нефрит

B Пієліт

C Цисти

D Уретрит

E Гепатит

51. У чоловіка віком 57 років, який страждав хронічним пієлонефритом, була виявлена артеріальна гіпертензія. Який механізм підвищення артеріального тиску є головним у цьому випадку?

A *Збільшення секреції реніну нирками.

B Збудження вегетативних центрів гіпоталамуса.

C Підвищення рівня катехоламінів в крові.

D Збудження барорецепторів синокаротидної зони.

E Збудження кори великих півкуль.

52. Хвора 58 років скаржиться на періодичну біль в поперековій області, що супроводжується лихоманкою, слабкістю, помірною поліурією та набряками. В сечі: лейкоцити 30-40 в колі зору, еритроцити 10-15 в колі зору, білок 0,4 г/л, бактеріурія. Яка патологія нирок найбільш вірогідна у хворої?

A *Пієлонефрит.

B Гострий гломерулонефрит.

C Хронічний гломерулонефрит.

D Нефротичний синдром.

E Нефролітіаз.

53. Внаслідок масивної крововтрати у хворого споспостерігається зниження АТ 80/50 мм рт.ст., збільшені ЧСС та дихання. Лікар запідозрив розвиток постгеморрагічного шоку. Яка функція нирок порушена в першу чергу?

A *Фільтраційна.

B Секреторна.

C Концептраційна.

D Еритропоетична.

E Азотовидільна.

54. У хворого при обстеженні виявлена глюкозурія, гіперглікемія. Скарги на сухість в роті, свербіння шкіри, часте сечовиділення, спрагу. Поставлен діагноз: цукровий діабет. Чим обумовлена поліурія у даного хворого?

A *Збільшенням осмотичного тиску сечі.

B Зменьшенням онкотичного тиску плазми.

C Збільшенням фільтраційного тиску.

D Зменьшенням серцевого вибросу.

E Збільшенням онкотичного тиску плазми.

55. У хворого з’ясовано отруєння кислими речовинами, у цьому випадку застосовують закономірності неіонної дифузії у нирках. У цьому разі треба створити лужну реакцію сечі, яка б прискорювала виділення токсичної речовини. Які процеси підтримки кислотно-лужного стану крові відбуваються у канальцях нирок?

A Збільшується секреція Н+

B Збільшується фільтрація

C Збільшується реабсорбція солей

D Збільшується секреція НСО?-

E Збільшується швидкість ниркового кровообігу

56. При захворюваннях нирок, які супроводжуються порушенням екскреторної функції клубочків (гострий і хронічний гломерулонефрит) часто спостерігається анемія, що носить нормоцитарний, нормохромний та гіпорегенеративний характер. Яка найбільш вірогідна причина анемії?

A *Підвищення секреції інгібітора еритропоезу.

B Дефіцит заліза.

C Пригнічення функцій кісткового мозку азотвмісними речовинами.

D Гематурія.

E Геморагічний діатез.

Хвороби статевих органів та молоч-ної залози. Хвороби вагітності, пологів та післяпологового періоду.

1. В ході гістологічного дослідження вишкрібку стінок порожнини матки 45-річної жінки з порушеннями оваріально-менструального циклу виявлено збільшення кількості ендометріальних залоз, деякі пилообразно извитые, деякі залози - кистозно розширені. Діагностуйте захворювання.

A. * Залозисто-кистозна гіперплазія ендометрію

B. Плацентарный полип

C. Атипова гіперплазія ендометрію

D. Залозистий поліп ендометрію

E. Аденокарцинома ендометрію

2. У жінки 28 років народився мертвий плід з гестаційним віком понад 43 тижні. Шкіра плоду суха лупиться. На аутопсії - загальна гіпотрофія і наявність ядер окостеніння проксимального епіфіза великогоміл-кової і плечової кісток. навколоплідні води, пуповина та оболонки посліду забарвлені меконієм. Проявом якого періоду є описані морфологічні зміни:

A. *антенатального періоду

B. Пренатального періоду

C. Інтранатального періоду

D. Постнатального періоду

E. Періоду прогенезу

3. У молодої жінки в зв’язку з гострими болями в клубній області, вилучена маткова труба з локальним розширенням її середньої третини, заповненої кров’ю. При гістологічному дослідженні в отворі труби знайдені хоріальні ворсини, великі поля еритроцитів з домішком лейкоцитів. Ваш діагноз:

A. *Трубна вагітність

B. Гострий гнійний сальпінгіт

C. Крововилив в маткову трубу

D. Геморагічний сальпінгіт

E. Гнійний сальпінгіт

4.У літньої жінки з метрорагією в менопаузі при патогістологічному дослідженні в зіскобі слизової оболон-ки шийки матки виявлені розростання атипового епітелію з утворенням так званих "ракових перлин". Ваш діагноз?

A. *Плоскоклітинний рак зі зроговінням

B. Аденокарцинома

C. Плоскоклітинний рак без зроговіння

D. Слизовий рак

E. Недиференційований рак

5. У померлої 42 років оперованої у минулому з приводу пухлини шлунку при розтині яєчник різко збільшений в розмірах, щільної консистенції, білуватого кольору. Гістологічно: в препараті крайньо атипові епітеліальні клітини, розміщені серед пластів і тяжів сполучної тканини. Яке захворювання має місце у хворої?

A. *Крукенбергівський рак яєчника.

B. Серозна цистаденокарцинома.

C. Псевдомуцинозна цисткарцинома.

D. Злоякісна текома.

E. Злоякісна гранульзоклітинна пухлина.

6. Хвора оперована з приводу пухлини матки. Макропрепарат: губчастий строкатий вузол в міометрії. Гістологічно- наявні великі світлі епітеліальні клітини, серед яких багато темних поліморфних клітин. Строма відсутня, судин мають вигляд порожнин, вистелених клітинами пухлини. Визначаються множинні крововиливи. Як називається виявлена пухлина?

A. *Хоріонепітеліома.

B. Деструюючий (злоякісний) міхурцевий занесок.

C. Аденокарцинома.

D. Кавернозна гемангіома.

E. Медулярний рак.

7. У жінки 39 років при операції були видалені збільшена у розмірах маточна труба та частина яєчника з великою кістою. При гістологічному дослідженні стінки труби виявлені децидуальні клітини, ворсини хоріону. Який найбільш імовірний діагноз було встановлено при дослідженні маточної труби?

A. *Трубна вагітність

B. Плацентарний полип

C. Хоріонкарцинома

D. “Паперовий “ плід

E. Літопедіон

8. При гістологічному дослідженні слизової оболонки матки знайдено звивисті залози, пилко- та штопороподібні, подовжені, розростання строми з проліферацією її клітин. Ваш діагноз?

A. * Залозиста гіперплазія ендометрія

B. Гострий ендометрит

C. Лейоміома

D. Пухирний занос

E. Плацентарний поліп

9. У жінки 45 років при дослідженні біоптату шийки матки виявлено значний поліморфізм багатошарового плоского епітелію, наявність патологічних мітозів та рогових "перлин" в глибині епітеліальних шарів. Ваш діагноз:

A * Плоскоклітинний рак з ороговінням

B Залозистий рак

C Недиференційований рак

D Перехідноклітинний рак

E Плоскоклітинний рак без зроговіння

10. На патологоанатомічному розтині тіла жінки 24 років, яка поступила в лікарню після автомобільної катастрофи і померла через дві доби, в порожнині матки виявлені темночервоні згортки крові. Як правильно назвати знайдені зміни?

A *Гематометра

B Метрорагія

C Менометрорагія

D Менорея

E Аменорея

11. У вагітної жінки після кесаревого розтину в післяопераційному періоді виникла масивна маткова кровотеча, яку вдалося зупинити, але розвинулась анурія. Гемодіаліз виявився неефективним і хвора померла на 4-ту добу після втручання. На розтині нирки трохи збільшені за розміром, набряклі, на розрізі коркова речовина бліда, піраміди і проміжна зона повнокровні. Яка патологія нирок ускладнила операцію?

A *Некротичний нефроз

B Ішемічний інфаркт

C Ціанотична індурація

D Підгострий гломерулонефрит

E Гострий пієлонефрит

12. У хворої, 24 років видалена пухлина яєчника, яка представлена кістою, діаметром 8 см. При патогістологічному дослідженні виявлено, що кіста містить жирову, хрящову та нервову тканину, волосся. До якого виду пухлин відноситься видалене новоутворення?

A *Тератома

B Епітеліальна органоспецифічна

C Епітеліальна органонеспецифічна

D Пухлина жирової тканини

E Пухлина сполучної тканини

13. У жінки 25-ти років через 4 тижні після аборту з’явився кашель, температура тіла підвищилась до 37,8*С. На рентгенограмах у легенях виявлені вогнища затемнення з обох сторін. На протязі 10-ти днів хвора отримувала антибіотики, але без ефекту. Виконана крайова резекція легені. В препараті знайдена пухлина, що складається з клітин трофобласта та поліморфних гігантських елементів синцитіотрофобласта з високою мітотичною активністю: строма відсутня, множинні крововиливи. Заключення по гістологічній картині:

A *Хоріонкарцинома

B Світлоклітинний рак

C Плоско клітинний рак

D Трофобластична пухлина плацентарного ложа

E Аденокарцинома

14. При гістологічному дослідженні вмісту порожнини матки, отриманого шляхом діагностичного вишкрібання, виявлено хоріонепітеліому. З яких тканинних елементів розвивається ця пухлина?

A *Тканинних елементів плаценти

B Слизової оболонки тіла матки

C Мязової оболонки тіла матки

D Тканини яєчника

E Слизової оболонки цервікального каналу

15. У жінки 25 років вагітність закінчилась смертю плода на ранніх сроках. При гістологічному досліджені вмісту матки виявлено гідропічне та кістозне перередження ворсин хоріона, конгломерати яких нагадують грона винограда. Який патологічний процес розвинувся в плаценті?

A *Міхуровий занесок

B Плацентарний поліп

C Хоріонепітеліома

D Фолікулярні кісти

E Залозисто-кістозна гіперплазія

16. Під час розтину тіла жінки 32 років, яка померла від внутрішньої кровотечі, виявлено розрив маткової труби і мертвий плід в черевній порожнині. Які зміни призвели до руйнування маткової труби і кровотечі?

A *Вростання ворсин хоріона

B Гострий сальпінгіт

C Децидуальна реакція слизової оболонки

D Міхуровий занесок

E Залозиста гіперплазія слизової оболонки матки

17.У жінки через добу після аборту, зробленого поза лікувальним закладом, зявилась гектична температура, та гноєвидні виділення із статевих органів. Назвіть патологічний процес, який розвинувся в матці після аборту.

A *Гострий ендометрит

B Гострий сальпінгіт

C Пухирний занесок

D Хоріонепітеліома

E Плацентарний поліп

18. При огляді мертвонародженого, який помер від інтеркуррентного захворювання, звертає на себе увагу косий розріз очей, спинка носу, що западає, низьке розташування маленьких вушних раковин. При розтині виявлені ознаки комбінованої уродженої вади серця (тетрада Фалло). Про яке хромосомне захворювання необхідно подумати?

A *Хвороба Дауна

B Синдром Патау

C Синдром Шерешевського-Тернера

D Синдром Едвардса

E Синдром триплоїдії

18. При огляді мертвонародженого виявлені загальна гіпоплазія, низький скошений лоб, вузькі очні щілини, переносся, що запало, гіпотелоризм, дефекти скальпу, деформовані вушні раковини, розщілина верхньої губи і піднебіння, полідактилія. Про яку вроджену ваду необхідно подумати?

A * Синдром Патау

B Хвороба Дауна

C Синдром Шерешевського-Тернера

D Синдром триплоїдії

E Синдром Едвардса

19. При огляді мертвонародженого звертає на себе увагу поєднання низького зросту, коротка шия зі складками, яка нагадує шию сфінкса, лімфонабряк кистей и стоп. На розтині виявлені коарктація аорти, недорозвиток гонад, підковоподібна нирка. Про яку ваду розвитку необхідно подумати?

A * Синдром Шерешевського-Тернера

B Хвороба Дауна

C Синдром Патау

D Синдром триплоїдії

E Синдром Едвардса

20. Через дві доби після пологів у жінки розвинулась клініка шоку з ДВЗ-синдромом, в результаті чого хвора померла. На аутопсії знайдено гнійний ендоміометрит, регіональний гнійний лімфангіт і лімфаденіт, гнійний тромбофлебіт. В паренхіматозних органах – дистрофічні зміни і проміжне запалення. Ваш діагноз:

A * Сепсис

B Сифіліс

C Туберкульоз статевих органів

D Деструюючий міхурцевий занесок

E Міхурцевий занесок

21. У вишкрібку з порожнини матки жінки 32 років знайдено: залози ендометрію у фазі глибокої секреції, строма – децидуальна тканина, присутні множинні ворсини хоріону та великі скупчення елементів цито- та синцитіотрофобласту, клітини цитотрофобласту мають значний клітинний поліморфізм та ядерну гіперхромію, зустрічаються множинні вогнища некрозу. Діагностуйте ураження.

A * Хоріонепітеліома.

B Частковий міхурцевий занесок.

C Повний міхурцевий занесок.

D Плацентарний поліп.

E Децидуальний поліп.

22. У 39-річної жінки в правій грудній залозі діагностовано множинні дрібні щільні вузлики в лівій грудній залозі. При дослідженні біоптату з грудної залози виявлено обширні ділянки гіалінізованої сполучної тканини та зони ліпоматозу, серед яких розташовані поодинокі атрофовані ацинуси та кистозно розширені протоки. Діагностуйте захворювання:

A * Непроліферативна форма фіброзно-кістозної хвороби

B Проліферативна мастопатія.

C Інвазивний протоковий рак.

D Інтраканалікулярна фіброаденома.

E Періканалікулярна фіброаденома.

23. При гістологічному дослідженні легеневої тканини померлого від дихальної недостатності недоношеного хлопчика виявлені ділянки ателектазів, а в просвіті розправлених альвеол визначаються набрякова рідина, крововиливи, пристінково розташовані еозинофільні маси - “гіалінові” мембрани. Діагностуйте захворювання.

A * Респіраторний дистрес синдром новонароджених.

B Асфіксія новонароджених.

C Бронхолегенева дисплазія.

D Внутрішньоутробна пневмонія.

E Аспірація навколоплідних вод.

24. Новонароджена дитина відмовляється від їжі, в неї періодичне блювання, пронос, а з часом виникло помутніння кришталика. Про яку патологію слід думати лікарю?

A *Галактоземія

B Тирозиноз

C Фенілкетонурія

D Цистинурія

E Алкаптонурія

25. Хвора дитина бліда, пастозна, м'язова тканина розвинута слабо, лімфовузли збільшені. Часто хворіє на ангіну, фарингіт, у крові лімфоцитоз. Схильний до автоалергічних захворювань. Який вид діатезу можна запідозрити в даному випадку?

A *Лімфатико-гіпопластичний.

B Ексудативно- катаральний

C Нервово-артритичний

D Астенічний

E Геморагічний

26. В другій половині вагітності у жінки відмічалась гіпертензія, протеїнурія, гипопротеїнемія, з'явилися набряки. Головна роль в розвитку набряків в даному випадку належить:

A *Пониженню онкотичного тиску крові

B Альдестеронемії

C Венозної гіперемії

D Лімфостазу

E Запаленню

27. Жінка 52 р., хвора на рак млочної залози, пройшла курс променевої терапії. Розмір пухлини зменшився. Який з наведених механізмів ушкодження клітини найбільш обумовлює ефективність променевої терапії?

A *Утворення вільних радикалів

B Гіпертермія

C Лізис NK-клітинами

D Тромбоз судин

E Мутагенез

Хвороби залоз внутрішньої секреції.

1. У молодої людини виявлений надлишок соматотропного гормону, збільшені розміри носа, губ, ушей, нижньої щелепи, кистей і стоп. Ваш діагноз.

A. *Акромегалія

B. Гіпофізарний нанізм

C. Хвороба Іценко-Кушинга

D. Адісонова хвороба

E. Адіпозогенітальна дистрофія

2. У хворого виникло збільшення щитовидної залози в 2 рази. При пальпації залоза щільна, поверхня нерівномірно горбиста. При гістологічному дослідженні - дифузна інфільтрація тканини залози лімфоцитами, плазматичними клітинами з утвореннм фолікулів та посилене розростання сполучної тканини. Яке захворювання має місце у хворого?

A. *Зоб Хасімото.

B. Ендемічний зоб.

C. Спорадичний зоб.

D. Дифузний токсичний зоб.

E. Зоб Ріделя.

3. У хворого, який раніше хворів на гематогенний туберкульоз, з'явилась гіперпігментація шкіри (меланоз) та слизових оболонок, кахексія, недостатність серцево-судинної системи. Яке захворювання викликало такі зміни?

A. *Аддісонова хвороба

B. Феохромоцитома

C. Хвороба Симмондса

D. Хвороба Іценко-Кушинга

E. Хвороба Грейвса

4. У хворого з гірського району Середньої Азії виявлене збільшення щитовидної залози, що затруднило ковтання. Спостерігались збільшення маси тіла, сповільненість, сонливість, одутле обличчя. При мікроскопічному дослідженні у щитовідній залозі - різних розмірів фолікули з гіпохромним колоїдом. Який з діагнозів найбільш ймовірний?

A. *Ендемічне воло.

B. Базедове воло.

C. Тиреоідіт Хошимото.

D. Спорадичне воло.

E. Воло Риделя.

5. При розтині жінкі 45 років, яка при жітті страждала ожирінням за верхнім типом, стероїдним цукровим діабетом, артеріальною гіпертонією, вторинною дисфункцією яєчніків виявлено: гіпертрихоз, гірсутизм, стрії на шкірі стегон та живота. В передній долі гіпофізу - пухлина (мікроскопічно: базофільна аденома); у наднирниках - гіперплазія пучкового шару. Який з перелічених діагнозів найбільш вірогідний?

A. *Хвороба Іценко-Кушинга

B. Синдром Іценко-Кушинга

C. Хвороба Симондса

D. Адіпозогенітальна дістрофія

E. Гіпофізарний нанізм

6. При розтині новонародженої дитини, віком 3 діб, померлої від внутрішньоутробної пневмонії, виявлено зменшення розмірів та маси вилочкової залози. При мікроскопічному дослідженні її у часточках не виявляється чітка межа між корою та мозковою речовиною в наслідок лімфоїдного спустошення обох зон; у мозковій речовині знайдені численні, різних розмірів тільця вилочкової залози (тільця Гассаля), з явищами кератозу та кальцинозу; міжчасточкова сполучна тканина розвинута надмірно. Який з перелічених патологічних процесів у тимусі найбільш вірогідний?

A. *Акцидентальна інволюція тимусу

B. Атрофія

C. Гіпотрофія

D. Аплазія

E. Тімомегалія

7. При гістологічному дослідженні щитовидної залози виявлено помірну атрофію паренхіми, склероз, дифузну інфільтрацію строми лімфоцитами і плазматичними клітинами з формуванням лімфоїдних фолікулів. Ваш діагноз?

A. *Аутоімунний тиреоідит

B. Паренхіматозний зоб

C. Тіреотоксичний зоб

D. Тіреоїдит

E. Зоб Ріделя

8. При розтині тіла жінки 50 років, померлої від уремії, знайдено аденому паращитовидної залози, деформацію кінцівок, хребта, ребер. Кісткі м'які, на розрізі підвищеної порозності, з пухлиноподібними вузликами, які на розрізі мають пістрявий вигляд, а також з кистами. При мікроскопічному дослідженні виявляється значна перебудова кісткових структур, осередки лакунарного розсмоктування остеоїдної та фіброзної тканини. Яка хвороба найбільш вірогідна?

A. *Хвороба Реклінгхаузена

B. Хронічний остеомієліт

C. Остеопороз

D. Фіброзна дисплазія

E. Хвороба Педжета

9. При гістологічному дослідженні передміхурової залози, оперативно видаленої у чоловіка 72 років, який скаржився на утруднення сечовипускання, виявлено: збільшення кількості залозистих та м'язових елементів. Часточкова будова залози порушена. Який процес у передміхуровій залозі найбільш вірогідний?

A. *Змішана форма простатопатії

B. Залозиста гіперплазія

C. М'язово-фіброзна гіперплазія

D. Простатит

E. Аденокарцинома

10. У хворої людини 42 років виявлено значне збільшення в розмірах носа, вух, нижньої щелепи та стоп. Яке захворювання можна запідозрити?

A. *Акромегалія

B. Гігантизм

C. Нанізм

D. Церебральна кахексія

E. Адіпозогенітальна дистрофія

11. Жінці 40 років проведено операцію тироїдектомії. При гістологічному дослідженні щитоподібної залози виявлено, що її фолікули різновеликі, містять пінистий колоїд, епітелій фолікулів високий, місцями формує сосочки, в стромі - осередкова лімфоцитарна інфільтрація. Діагностуйте захворювання щитоподібної залози.

A. *Зоб Базедов

B. Тироїдит Хашімото

C. Тироїдит Ріделя

D. Тироїдит де Кервена

E. Вузловий зоб

12. У хворого 45 років поступово почали пропорційно збільшуватись в розмірах стопи, права кисть, ніс і губи. Виявлена аденома гипофізу. Яке захворювання можна запідозрити?

A. * Акромегалія

B. Нанізм

C. Хвороба Базєдова

D. Аддисонова хвороба

E. Цукровий діабет

13. При гістологічному дослідженні щитовидної залози визначається значна інфільтрація лімфоцитами зутворенням лімфоїдних фолікулів, руйнування паренхіматозних елементів, розростання волокон сполучної тканини. Для якого захворювання характерна така картина?

A. * Зоб Хосімото

B. Колоїдний зоб

C. Ендемічний зоб

D. Дифузний токсичний зоб

E. Паренхіматозний зоб

14. На розтині померлого від кахексії виявлено аденому паращитовидних залоз, деформацію кісток, особливо кінцівок, хребетного стовпа, ребер. Кістки м?які, порозні, легко ріжуться ножем. Який з перелічених діагнозів найбільш вірогідний?

A. *Паратиреоїдна остеодистрофія

B. Остеопетроз

C. Хондродисплазія

D. Остеомієліт

E. Фіброзна дисплазія

15.В операційній біопсії щитовидної залози гістологічно виявлені серед фолікулів, заповнених колоїдом, лімфоїдні структури із центрами росту. Яке захворювання у хворого?

A. *Зоб Хашимото

B. Ендемічний зоб

C. Спорадичний зоб

D. Базедов зоб

E. Зоб Риделя

16. У померлого хворого на розтині виявлено бронзове забарвлення шкіри та слизових оболонок порожнини рота. В наднирниках визначались казеозні маси. Яким захворюванням страждала людина?

A. *Адісонова хвороба

B. Синдром Іценко-Кушинга

C. Базедова хвороба

D. Акромегалія

E. Феохромоцитома

17. На розтині чоловіка 67 років, померлого при явищах гіпоглікемічної коми, в підшлунковій залозі спостерігаються ділянки розростання сполучної тканини та некротичні осередки, атрофія острівків Лангенгарса. Яке захворювання зумовило такі зміни в підшлунковій залозі?

A. *Цукровий діабет.

B. Муковісцидоз.

C. Гострий панкреатит.

D. Рак головки підшлункової залози.

E. Гіпоплазія підшлункової залози.

18. У людини з пухлиною передньої долі гіпофизу в результаті підвищеного вироблення кортикотропіну розвивається гіпертрофія кори наднирників. Як називається такий вид гіпертрофії?

A *Кореляційна

B Регенераційна

C Вакатна

D Вікарна

E Робоча

19. У хворого 30 років видалена пухлина щитовидної залози. Гістологічно пухлина складається з сосочкових утворень різної величини, які виходять з внутрішньої поверхні кістозно розширених фолікулів і вкриті атипічним епітелієм. Ваш діагноз.

A *Папілярний рак

B Вузловий колоїдний зоб

C Макрофолікулярна аденома

D Фолікулярний рак

E Медулярний рак

20. У чоловіка 40 років, який скаржиться на періодичні болі голови, помітне збільшення розмірів кистей та ступнів, при рентгенограмі відмічено збільшення розмірів „турецького сідла”. Яке захворювання у хворого?

A *Акромегалія

B Хвороба Сімондса

C Хвороба Іщенко-Кушинга

D Хвороба Бабінського-Фреліха

E Гіпофізарний нанізм

21. У чоловіка 30-ти років спостерігається помітне збільшення розмірів носа, губ, вух, нижньої щелепи, збільшення щитовидної залози, атрофія статевих залоз. Який діагноз найбільш вірогідний?

A *Акромегалія

B Гігантизм

C Адипозогенітальна дистрофія

D Гіпофізарний нанізм

E Хвороба Грейвса

22. У жінки 45 років відсутні симптоми діабету, але натще визначається підвищений вміст глюкози в крові (7,2 ммоль/л). Який наступний тест необхідно провести?

A *Визначення толерантності до глюкози

B Визначення залишкового азоту в крові

C Визначення глюкози крові

D Визначення глюкози у сечі

E Визначення гліколізованого гемоглобіну

23. У хворого 40 років ожиріння, зниження температури тіла, малорухливість. Обличчя пастозне з бідною мімікою, стовщеним носом і губами. Спостерігається ламкість нігтів, випадіння волосся таінші трофічні порушення. Статева функція зменшена, погіршилась пам’ять. Для якої ендокринної патології характерна така клінічна картина?

A * Мікседема

B Кретинізм

C Ендемічний зоб

D Тиреотривна кахексія

E Дифузний токсичний зоб

24. Для якої стадії загального адаптаційного синдрому характерні гіпертрофія кори наднирників і секреція великої кількості гормонів, активація анаболічних процесів, посилення гліконеогенезу?

A * Резистентності

B Тривоги

C Виснаження

D Аварійної

E Завершальної

25. Пацієнт, який хворіє на цукровий діабет, доставлений в лікарню без свідомості, артеріальний тиск знижений, дихання типу Кусмауля, з рота чути запах ацетону. Провідним механізмом для розвитку цієї коми є:

A *Накопичення в крові кетонових тіл

B Накопичення іонів калію

C Накопичення іонів натрію

D Накопичення іонів хлору

E Накопичення сечовини

26. Хвора звернулася до лікаря зі скаргами на схуднення, швидку фізичну і психічну втомлюваність, поганий апетит, зміну кольору шкіри, артеріальну гіпотензію. При огляді виявлено гіперпігментацію шкіри. Механізм її пов’язаний з

A *Посиленням меланоцитстимулюючої активності гіпофіза

B Зниженням меланоцитстимулюючої активності гіпофіза

C Зниженням рівня адреналіну в крові

D Переважанням катаболізму білків

E Підвищенням концентрації іонів натрію

27. Хворий звернувся до лікаря із скаргами на збільшення розмірів носа, підборіддя, пальців кінцівок. При об’єктивному обстеженні виявлено: розширення меж серця, збільшення розмірів печінки, селезінки. Причиною розвитку вказаних порушень є:

A *Підвищення секреції соматотропного гормону

B Підвищення секреції тиреотропного гормону

C Підвищення секреції адренокортикотропного гормону

D Підвищення секреції альдостерону

E Підвищення секреції адреналіну

28. Чому у деяких географічних регіонах України кількість людей з гіпертрофією щитовидної залози значно більша ніж в інших регіонах?

A * Низький рівень йоду у питній воді викликає компенсаторну реакцію залози – її розростання

B Високий рівень кальцію у воді стимулює ріст щитовидної залози.

C Низький рівень кальцію у воді стимулює ріст щитовидної залози.

D Радіоактивний фон стимулює ріст щитовидної залози.

E Високий рівень йоду у воді стимулює ріст щитовидної залози.

29. Під час експерименту тварині протягом тривалого часу вводили токсичні речовини. Після чого появились симптоми гіперглікемії, поліурії, постійної спраги, надмірного апетиту, які зникали на деякий час після введення гормону інсуліну. Ураження якого органу викликали токсичні речовини?

A * Підшлункової залози.

B Прищитовидних залоз.

C Наднирників.

D Щитовидної залози.

E Гіпоталамусу.

Інфекційні хвороби.

1. При мікроскопічному дослідженні шийного лімфатичного вузла виявлено скупчення епітеліоїдних клітин, лімфоцитів і гігантських клітин Пірогова-Ланганса. В центре казеозний некроз. Вкажіть найбільш вірогідну патологію.

A. * Туберкульоз

B. Риносклерома.

C. саркоидоз

D. Сап.

E. Сифіліс.

2. У померлого 67 років на розтині знайдені ознаки фібринозного запалення в товстому кишківнику. Ваш діагноз:

A. *Дизентерія

B. Амебіаз

C. Черевний тиф

D. Холера

E. Балантидіаз

3. У чоловіка 30 років при гістологічному дослідженні біоптату з шийного лімфатичного вузла виявлені гранульоми, які складаються з епітеліоїдних, лімфоїдних, багатоядерних гігантських клітин типу Пірогова-Лангханса. У центрі гранульом визначається некроз. Який збудник потрібно виявити в зоні некрозу для підтвердження діагнозу туберкульоз?

A. *Мікобактерію Коха

B. Бліду трепонему

C. Стафілокока

D. Бацили Волковича-Фріша

E. Сальмонели

4. У хворого М. 14 років, діагностована тріада Гетчинсона: зуби діжкоподібної форми, паренхіматозний кератит та глухота.. Для якої хвороби характерні виявлені зміни?

A. * Сифіліс.

B. Токсоплазмоз.

C. Проказа.

D. Туберкульоз.

E. Опісторхоз.

5. Смерть хворого К, 16 років, настала від розлитого (поширеного) фібринозно-гнійного перитониту. На розтині, в нижній ділянці тонкої кишки виявлена виразка, яка повторювала форму пейерової бляшки, з перфорацією стінки кишки. Мікроскопічне дослідження виявило стертість малюнка лімфоїдної тканини, витіснення її проліферуючими моноцитами, які формують гранульоми. Ускладненням якого захворювання є причина смерті?

A. * Черевного тифу.

B. Дизентерії.

C. Холери.

D. Бруцельоза.

E. Неспецифічного виразкового коліту.

6. При гістологічному дослідженні біоптата перегородки носу хворого, який страждав утрудненим носовим диханням, в слизовій оболонці знайдено гранульоматозне запалення з наявністю в гранульомах клітин Микуліча. Ваш діагноз.

A. *Риносклерома

B. Сифіліс

C. Туберкульоз

D. Сап

E. Лепра

7. При розтині трупа чоловіка, який хворів на черевний тиф, знайдені зміни у тонкій кишці: групові лімфоїдні фолікули збільшені, виступають над поверхнею слизової оболонки, вони сіро-червоні, соковиті, їх поверхня має вигляд звилин та боріздок. При мікроскопічному дослідженні відмічається утворення черевнотифозних гранульом. Вкажіть, яка з перелічених стадій черевного тифу найбільш вірогідна?

A. *Мозкоподібного набухання.

B. Некрозу.

C. Утворення виразок.

D. Чистих виразок.

E. Загоювання.

8. При розтині померлої дитини, яка страждала на діарею, виявлено ексикоз і поширений фібринозний коліт. В мазку-відбитку слизової оболонки виявлені грам-негативні палички. Ваш попередній діагноз?

A. * Дизентерія

B. Холера

C. Стафілококова кишкова інфекція

D. Черевний тиф

E. Сальмонельоз

9. Дитина у 3-х річному віці померла при явищах глибокої церебральної коми. На розтині виявлено гнійні нашарування на поверхні м’якої мозкової оболонки в передніх відділах головного мозку, набряк мозку. Мікроскопічно – повнокрів’я і нейтрофільна інфільтрація м’яких мозкових оболонок. Вкажіть ймовірне захворювання?

A. *Менінгококовий менінгіт

B. Менінгіт при сибіркі

C. Грипозний менінгіт

D. Туберкульозний менінігіт

E. -

10. При дослідженні некропсії чоловіка 57 років в II-ому сегменті правої легені виявлено туберкульозний панбронхіт, з вогнищем казеозної бронхопневмонії, навколо якого спостерігається вал епітеліоїдних клітин з домішком лімфоїдних та гігантських клітин Пірогова-Лангханса. Яка форма туберкульозу легень у даному випадку?

A. *Гострий вогнищевий туберкульоз легень

B. Первинний туберкульозний комплекс

C. Ріст первинного афекту

D. Гострий міліарний туберкульоз легень

E. Інфільтративний туберкульоз

11. Чоловік віком 42 роки помер при явищах вираженої інтоксикації і дихальної недостатності. На розрізі тканина легень у всіх відділах строката, з множинними дрібновогнищевими крововиливами та вогнищами емфіземи. Гістологічно у легенях: геморагічна бронхопневмонія з абсцедуванням, у цитоплазмі клітин епітелію бронхів еозинофільні і базофільні включення. Діагностуйте виявлене на секції захворювання.

A. * Грип

B. Парагрип

C. Аденовірусна інфекція

D. Респіраторно-синцитіальна інфекція

E. Стафілококова бронхопневмонія

12. На розтині чоловіка віком 34 роки діагностовано туберкульоз з наявністю у верхній частці правої легені зони казеозної пневмонії, казеозного лімфаденіту у збільшених лімфатичних вузлах середостіння та множинні міліарні вогнища у багатьох органах. Назвіть виявлену форму туберкульозу.

A. * Первинний змішаної форми генералізіції

B. Первинний гематогенно-генералізований

C. Первинний лімфогенно-генералізований

D. Первинний з ростом первинного афекту

E. Гематогенний

13. У дитини після перенесеної ангіни різко збільшилися лімфатичні вузли: паратрахеальні, біфуркаційні, шийні. При мікроскопічному дослідженні шийного лімфатичного вузла виявлені вогнища некрозу, обмежовані лімфоцитами, епітеліоїдними клітинами, та клітинами Пірогова-Лангханса. Вкажіть найбільш вірогідну патологію.

A. *Туберкульоз

B. Саркоїдоз

C. Риносклерома

D. Сап

E. Сифіліс

14. При розтині померлого хворого від розповсюдженого перитоніту в дистальних відділах тонкої кишки виявлено численні виразки овальної форми, які розташовані вздовж кишки. Дно виразок чисте, гладеньке, утворене м'язовою або серозною оболонкою, краї виразок рівні, закруглені. У двох виразках є перфоративні отвори діаметром до 0,5 см. Яке захворювання треба запідозрити?

A. *Черевний тиф

B. Дизентерія

C. Холера

D. Туберкульоз

E. Висипний тиф

15. У хворого на дифтерію на 2-му тижні розвинулась гостра серцева недостатність з летальним наслідком. При розтині: шлуночки серця розширені, міокард в'ялий і строкатий. Гістологічно виявлені чисельні дрібні вогнища міолізу слабкою перифокальною лімфоїдною інфільтрацією. Що стало причиною смерті?

A. *Міокардит, зумовлений бактеріальним екзотоксином

B. Бактеріальний міокардит

C. Септичний міокардит

D. Інфаркт міокарду

E. Метаболічні некрози міокарду

16. На розтині трупа чоловіка 44 років, який помер від легенево-серцевої недостатності, на розтині виявлено пневмосклероз, емфізему легень, гіпертрофію правого шлуночка серця. В обох легенях, переважно субплеврально множинні вогнища діаметром до 1 см. Гістологічно: в центрі вогнища – зона некрозу, на периферії – вал з епітеліоїдних клітин і лімфоцитів з домішкою макрофагів і плазматичних клітин. Визначаються гігантські клітини Пірогова-Лангханса. Невелике число кровоносних капілярів визначається на периферії вогнища. Яке захворювання має місце у хворого:

A. *Гематогенний туберкульоз.

B. Актиномікоз легень.

C. Саркоїдоз.

D. Сифіліс.

E. Силікоз.

17. Хворий помер на 3-й день після операції з приводу перфорації стінки товстої кишки з явищами розлитого гнійного перитоніту. На розтині: слизова оболонка стінки товстої кишки потовщена, вкрита фібриновою плівкою, поодинокі виразки проникають на різну глибину; гістологічно: некроз слизової, фібрину, інфільтрація лейкоцитами з фокусами геморагій. Ускладнення якого захворювання стало причиною смерті хворого?

A. *Дизентерія.

B. Черевний тиф.

C. Неспецифічний виразковий коліт.

D. Хвороба Крона.

E. Амебіаз.

18. У хворої 18 років пахові лімфатичні вузли збільшені у розмірах, не болючі, ущільнені при пальпації. В ділянціслизової оболонки геніталій невеликих розмірів виразка з ущільненими краями та "лакованим"дном сіруватого кольору. Поставити діагноз.

A. *Сифiліс.

B. Туберкульоз.

C. Лепра.

D. Трофiчна виразка.

E. Гонорея.

19. При розтині у дитини виявлений первинний кишковий туберкульозний комплекс: первинна аффект-виразка порожньої кишки, лімфангіт і регіонарний казеозний лімфаденіт. Смерть наступила від перфорації виразки і розвитку розлитого перитоніту. Назвіть шлях зараження дитини туберкульозом.

A. *Аліментарний.

B. Трансплацентарний.

C. Аерогенний.

D. Змішанний.

E. Контактний

20. Чоловік 36 років знаходився в інфекційній лікарні з профузною діареєю, ознаками ексикозу, падінням температури тіла. Помер від уремії. Під час розтину знайдено: у просвіті тонкої кишк безкольорова рідина у вигляді рисового відвару; слизова оболонка набрякла. При мікроскопічному дослідженні тонкої кишки – повнокрів'я судин, осередкові крововиливи, десквамація ентероцитів, гіперсекреція келихаподібних клітин та лімфо-лейкоцитарна інфільтрація строми слизової оболонки. Ваш попередній діагноз.

A. *Холера

B. Сальмонельоз

C. Дизентерія

D. Черевний тиф

E. Хвороба Крона

21. Після позалікарняного аборту у жінки пргресував гнійний ендо-міометрит зі смертельним наслідком. При розтині померлої виявлені чисельні абсцеси легень, субкапсулярні гнійнички в нирках, гіперплазія селезінки. Яка форма сепсису виникла у хворої?

A. *Септикопіємія

B. Септицемія

C. Хроніосепсис

D. Легеневий сепсис

E. Уросепсис

22. При розтині тіла чоловіка 48 років, померлого від туберкульозу легень, знайдені частково випорожнені симетричні туберкульозні каверни у кортико-плевральних зонах з перевагою продуктивної тканинної реакції; у обох легенях - ніжний сітчастий пневмосклероз та емфізема; “легеневе серце”; у біфуркаційних лімфовузлах - туберкульозний казеозний лімфаденіт. У ІІІ сегменті правої легені - вогнище Гона. Вкажіть, який з перелічених діагнозів найбільш вірогідний?

A. *Гематогенно-дисемінований туберкульоз.

B. Гематогенна генералізація первинного туберкульозу.

C. Хроничний первинний туберкульоз.

D. Хроничний міліарний туберкульоз.

E. Циротичний туберкульоз.

23. Хворий інфекційного відділення скаржився на слабкість, відсутність апетиту, підвищення температури до $38^o$С. На 7 добу – різкий біль у правому підребер'ї та пожовтіння шкіри. При мікроскопії біоптату печінки: порушення балкової будови, у гепатоцитах – гідропічна та балонна дистрофія, в деяких гепатоцитах – некроз, тільця Каунсільмена, на периферії часточок – збільшена кількість багатоядерних гепатоцитів. Яка форма вірусного гепатиту найбільш вірогідна?

A. *Циклічна жовтянична

B. Злоякісна

C. Хронічна

D. Холестатична

E. Безжовтянична

24. У хворого з субфебрильною температурою в біоптаті збільшеного лімфатичного вузла виявлені численні гранульоми, які містять у центрі казеозний некроз, оточений епітеліоїдними клітинами, велетенськими багатоядерними клітинами Пирогова-Лангханса і лімфоцитами. Для якого захворювання характерні такі патогістологічні зміни?

A. *Туберкульоз

B. Лімфогранульоматоз

C. Лімфосаркома

D. Лімфолейкоз

E. Банальний лімфаденіт

25. У хворого зі скаргами на задуху проведена біопсія слизової оболонки носової порожнини. Встановлено дiагноз: риносклерома. Якi клiтини типовi для даного захворювання при мiкроскопiчному дослiдженi?

A. *Клiтини Мiкулiча

B. Клiтини Пирогова-Лангханса

C. Плазмоцити

D. Лімфоцити

E. Тiльця Шаумана

26. На розтині у померлого в сигмоподібній та прямій кишці множинні червоні виразки неправильної форми, між якими слизова вкрита брудно-сірою плівкою. Етіологія захворювання:

A. *Шигелла

B. Амеба

C. Мікобактерія туберкульозу

D. Сальмонела

E. Стафілокок

27.Хворий 46р. скаржився на утрудненя дихання носом. В біоптаті потовщеної слизової носа знайдені клітини Мікуліча, скупчення епітеліоїдних клітин, плазмоцити, лімфоцити, гіалінові кулі. Ваш діагноз?

A. *Склерома

B. Аденовірусний риніт

C. Алергічний риніт

D. Риновірусна інфекція

E. Менінгококовий назофарингіт

28. У померлого, який прижиттєво отримував чисельні ін’єкції наркотиків, в печінці гістологічно знайдена гідропічна дистрофія гепатоцитів, “матовосклоподібні гепатоцити”, ацидофільні тільця Каунсільмена, лімфоцитарно-макрофагальні скопичення в портальних трактах. Найбільш вірогідна етіологія захворювання:

A. *Вірусна

B. Бактеріальна

C. Токсична

D. Паразитарна

E. Грибкова

29. На розтині хворого виявлено: м’яка мозкова оболонка верхніх відділів півкуль головного мозку різко повнокровна, жовто-зеленого кольору, просочена гнійним та фібринозним ексудатом, що нагадує чіпець. Для якого захворювання характерна така картина?

A. *Менінгококового менінгіту

B. Туберкульозного менінгіту

C. Грипозного менінгіту

D. Менінгіту при сибірці

E. Менінгіту при висипному тифі

30. Робітник тваринницької ферми гостро захворів і при наростаючих явищах інтоксикації помер. На розтині встановлено, селезінка збільшена, в’яла, на розрізі темно-вишневого кольору, зішкріб пульпи обільний. М’які мозкові оболонки на склепінні та основі мозку набряклі, просякнуті кров’ю мають темно-червоний колір (“шапочка кардинала”). Мікроскопічно: серозно-геморагічне запалення оболонок і тканин головного мозку з руйнуванням стінок дрібних судин. Поставте діагноз.

A. *Сибірка

B. Туляремія

C. Бруцельоз

D. Чума

E. Холера

31. При гістологічному дослідженні біоптату шкіри виявлені гранульоми, які складаються з макрофагальних вузликів з наявністю лімфоцитів та плазматичних клітин. Крім того, зустрічаються великі макрофаги з жировими вакуолями, які містять запакованих у вигляди куль збудників захворювання (клітини Вірхова). Грануляційна тканина добре васкуляризована. Для якого захворювання характерна описана картина?

A. *Лепри

B. Туберкульозу

C. Сифілісу

D. Риносклероми

E. Сапу

32. При мікроскопічному дослідженні біоптату шкіри виявляються гранульоми, які складаються з епітеліоїдних клітин, оточених в основному Т-лімфоцитами. Серед епітеліоїдних клітин розташовуються поодинокі гігантські багатоядерні клітини типу Пирогова-Лангхаінса. В центрі деяких гранульом виявляються ділянки казеозного некрозу. Кровоносні судини відсутні. Для якого захворювання характерні описані гранульому?

A. *Туберкульозу

B. Сифілісу

C. Лепри

D. Риносклероми

E. Сапу

33. Дитина віком 8 років захворіла гостро. Через дві доби від початку захворювання наступила смерть. На аутопсії виявлено, що м’які мозкові оболонки різко повнокровні, просякнуті густим каламутним жовтувато-зеленуватим ексудатом на базальній поверхні головного мозку. Тканина мозку набрякла, повнокровна. Ваш діагноз.

A. *Менінгококова інфекція

B. Скарлатина

C. Коклюш

D. Дифтерія

E. Кір

34. На розтині трупа чоловіка віком 56 років у термінальному відділі тонкої кишки виявлено декілька виразок розмірами від 4 до 5 см. Краї виразок піднімаються над поверхнею слизової оболонки, стінки виразок покриті сірувато-жовтуватими масами, які кришаться. Реакція Відаля позитивна. Поставте діагноз.

A. *Черевний тиф

B. Паратиф

C. Поворотний тиф

D. Дизентерія

E. Хвороба Крона

35. Під час розтину тіла дівчики 9 років у верхівці правої легені субплеврально було зайдене вогнище казеозного некрозу діаметром 15 мм, біфуркаційні лімфатичні вузли були збільшені, містили дрібні вогнища некрозу коагуляційного типу. Мікроскопічно - у легеневому вогнищі та в лімфатичних вузлах навколо некротичних мас були розташовані епітеліоїдні клітини, лімфоцити та поодинокі багатоядерні гігантські клітини. Діагностуйте захворювання.

A. *Первинний туберкульоз

B. Гематогенний туберкульоз з переважним ураженням легень

C. Гематогенний генералізований туберкульоз

D. Вторинний вогнищевий туберкульоз

E. Вторинний фіброзно-вогнищевий туберкульоз

36. На розтині тіла чоловіка 63 років, були знайдені множинні патологічні порожнини в обидвох легенях, мікроскопічно у внутрішньому шарі стінки порожнини знайдені некротичні маси, які дифузно інфільтровані нейтрофілами, середній - містить інфільтрат з епітеліоїдних клітин, лімфоцитів та багатоядерних гігантів, зовнішній шар складається із зрілої сполучної тканини. Діагностуйте форму вторинного туберкульозу.

A. *Фіброзно-кавернозний

B. Гострий вогнищевой

C. Фіброзно-вогнищевий

D. Гострий кавернозний

E. Циротичний туберкульоз

37. Жінка 32 років померла в післяпологовому періоді. На розтині були знайдені ознаки гнійного ендометриту, гнійний тромбофлебіт вен матки, множинні абсцеси легень, абсцеси нирок і селезінки, апостематозний міокардит та гнійний менінгіт. Діагностуйте вид сепсису.

A. *Септикопіємія

B. Септицемія

C. Хроніосепсис

D. Затяжний септичний ендокардит

38.У чоловіка 40 років в ділянці шиї виникло почервоніння та набряк шкіри і з часом розвинувся невеликий гнійник. На розрізі осередок щільний, жовто-зеленого забарвлення. В гнійних масах видно білі крупинки. Гістологічно виявлено друзи грибка, плазматичні та ксантомні клітини, макрофаги. Вкажіть найбільш ймовірний вид мікозу.

A. *Актиномікоз

B. Аспергільоз

C. Кандидоз

D. Споротрихоз

E. Кокцидіоїдомікоз

39. На розтині чоловіка, який помер на 5-у добу захворювання черевним тифом, виявлені наступні зміни: групові фолікули клубової кишки збільшені, повнокровні і виступають над слизовою оболонкою, на їх поверхні видно борозди та звивини. Гістологічно: повнокрів’я і набряк тканини, наявність гранульом, які складаються з великих клітин зі світлою цитоплазмою і містять черевнотифозні палички. Про який період місцевих змін при черевному тифі можна думати?

A. *Стадія мозкоподібного набухання

B. Стадія некрозу

C. Стадія загоювання виразок

D. Стадія чистих виразок

E. Стадія утворення виразок

40. При розтину трупа чоловіка 40 років виявлено різко розширений просвіт тонкої кишки, переповнений рідиною, яка нагадує “рисовий відвар”. Стінка кишки набрякла, на слизовій велика кількість дрібнокрапкових крововиливів. Для якого інфекційного захворювання характерний описаний ентерит?

A. *Холера

B. Дизентерія

C. Сальмонельоз

D. Амебіаз

E. Черевний тиф

41. Під час розтину в печінці померлого був виявлений утвір у вигляді міхура округлої форми з гладкою поверхнею діаметром 5 см. В його порожнині знаходилася велика кількість дрібних міхурців з прозорим безбарвним вмістом. Тканина печінки навколо міхура склерозована. Який діагноз найбільш вірогідний?

A. *Гідатидозний ехінококоз

B. Альвеококоз

C. Цистицеркоз

D. Опісторхоз

E. Шистосомоз

42. У чоловіка 43 років, який тривалий час хворів на туберкульоз, розвинулась кровотеча з легень, що призвела до смерті. На розтині в легенях виявлено декілька порожнин овальної або круглої форми, стінка яких створена некротичними масами та тканиною легень. Для якої форми туберкульозу характерні такі зміни

A. *Гострий кавернозний

B. Фіброзно-кавернозний

C. Туберкулома

D. Казеозна пневмонія

E. Гострий вогнищевий

43. При мікроскопії біоптату печінки виявлені гранульоми, які складаються з плазматичних, лімфоїдних, гігантських багатоядерних клітин типу Пирогова-Лангхарса, дрібних судин з ознаками ендо- і периваскуліту, зустрічаються осередки казеозного некрозу. Такі гранулеми характерні для:

A. *Сифілісу

B. Туберкульозу

C. Лепри

D. Риносклероми

E. Сапу

44. У дівчинки 8 років раптово підвищилась температура, з’явились катаральні явища дихальних шляхів. На 5-й день хвороби виник параліч м’язів нижніх кінцівок і приєднались дихальні розлади. В передніх рогах спинного мозку виявлена проліферація глії навколо загиблих нейронів. Про яке захворювання слід думати?

A. *Поліомієліт

B. Кір

C. Дифтерію

D. Менінгококцемію

E. Скарлатину

45. У 30-річного чоловіка, який хворів на гостре респіраторне захворювання та загинув при явищах гострої легенево-серцевої недостатності, під час розтину знайдено фібринозно-геморагічне запалення в слизовій оболонці гортані та трахеї, деструктивний панбронхит, збільшені легені, яки мають пістрявий вигляд за рахунок абсцесів, крововиливів, некрозу. Який з перелічених діагнозів найбільш вірогідний?

A. *Грип.

B. Парагрип.

C. Респіраторно-синцитіальна інфекція.

D. Кір.

E. Аденовірусна інфекція.

46. Під час розтину чоловіка, у якого після поранення кінцівки виникло тривале нагноєння рани, і який помер при явищах інтоксікації, знайдено загальне виснаженя, зневоднення, бура атрофія печінки, міокарду, селезінки, поперечно-смугастої мускулатури та амілоїдоз нирок. Який з перелічених діагнозів найбільш ймовірний?

A. *Хронісепсіс.

B. Септикопіємія.

C. Септицемія.

D. Хвороба Чорногубова.

E. Бруцельоз.

47. При розтині померлого, який хворів на туберкульоз, у верхній долі правої легені знайдено порожнину розмірами 3х2 см, яка сполучається з бронхом. Стінка порожнини щільна, має три шари: внутрішній – піогенний, середній – шар туберкульозної грануляційної тканини, зовнішній – сполучнотканинний. Який з перелічених діагнозів найбільш вірогідний?

A. *Фіброзно-кавернозний туберкульоз.

B. Фіброзно-осередковий туберкульоз.

C. Туберкульома.

D. Гострий осередковий туберкульоз.

E. Гострий кавернозний тубеокульоз.

48. У померлого 44 років, громадянина України, за результатами розтину встановлено поєднання пневмоцистної пневмонії, саркоми Капоші та В-клітинної лімфоми. В анамнезі невпорядковані статеві контакти. Що є найбільш вірогідним?

A. *Інфекція вірусом імунодефіциту людини, стадія СНІДу

B. Інфекція вірусом імунодефіциту людини, стадія преСНІДу

C. Вторинний імунодефіцит внаслідок первинної В-клітинної лімфоми

D. Вторинний імунодефіцит внаслідок саркоми Капоші

E.

49.Патологоанатом в біоптаті шкіри побачив гостре серозно-геморагічне запалення і ділянку некрозу. З анамнезу: захворювання почалося з появи невеликої червоної плями, в центрі якої утворився міхур із серозно-геморагічною рідиною. Згодом центральна частина стала чорною. Що є більш вірогідним?

A. *Карбункул при сибірці

B. Актиномікоз шкіри

C. Алергічний дерматит

D. Карбункул стрептококовий

E. Хімічний дерматит

50. На розтині померлого 18 років селезінка вагою 580 грамів, темно-червоного кольору на розрізі, дає рясний зскрібок пульпи. Гістологічно: виражена проліферація ретикулярних клітин, наявність великої кількості зрілих нейрофілів в сінусоїдних капілярах. Як правильно називати таку селезінку?

A. *“Септична”

B. “Сагова”

C. “Порфірова”

D. “Ціанотична”

E. “Лейкозна”

51. У юнака 18 років, який захворів гостро і помер від інфекційно-токсичного шоку, на автопсії виявлено збільшені мигдалики, вкриті сіро-білими плівками, що поширюються на піднебінні дужки, набряк м’яких тканин шиї. При гістологічному дослідженні: некроз епітелію мигдаликів та дужок, підлеглі тканини просякнуті фібринозним ексудатом, який утворює масивні нашарування на поверхні. Діагностуйте захворювання.

A. *Дифтерія

B. Скарлатина

C. Аденовірусна інфекція

D. Інфекційний мононуклеоз

E. Стафілококова інфекція

52. На слизовій оболонці правого піднебінного мигдалика спостерігається безболісна виразка з гладким лакованим дном та рівними хрящеподібної консистенції краями. Мікроскопічно: запальний інфільтрат що складається з лімфоцитів, плазмоцитів, невеликої кількості нейтрофілів та епітеліоїдних клітин та наявність єндо та периваскуліту. Про яке захворювання йде мова?

A. * Сифіліс

B. Актиномікоз

C. Туберкульоз

D. Дифтерія зеву

E. Виразково-некротична ангина Венсана

53. При розтині хворого, померлого через тиждень від початку профузної діареї, виявлений різко виразимий ексікоз, всі тканини сухі, кров густа. При бактеріологічному дослідженні вміста тонкої кишки, що нагадує рисовий відвар, знайдені вібріони. Яке захворювання привело хворого до смерті?

A. * Холера

B. Дизентерія

C. Черевний тиф

D. Сальмонельоз

E. Харчова токсикоінфекція

54. У хворого з дизентерією при колоноскопії виявлено, що слизова оболонка товстої кишки гіперемірована, набрякла, її поверхня покрита сіро-зеленими плівками. Назвіть морфологічну форму дизентерійного коліту.

A. *Фібринозний коліт

B. Катаральний коліт

C. Виразковий коліт

D. Гнійний коліт

E. Некротичний коліт

55. У хворого на 5-й день від початку діареї при колоноскопії в запаленій слизовій оболонці прямої кишки знайдені сіро-зелені плівчасті накладення, щільно фіксовані з належною тканиною. Який діагноз найбільш вірогідний?

A. * Дизентерія

B. Черевний тиф

C. Неспецифічний виразковий коліт

D. Сальмонеллез

E. Хвороба Крону

56. При розтині, померлого від чуми хворого, на фоні геморагічного синдрому знайден геморагічний некроз шкіри стегна, лимфангіт, паховий геморагічний лімфаденіт. Назвіть форму чуми.

A. * Шкірно-бубонна

B. Бубонна

C. Первинно-септична

D. Первинно-легенева

E. Геморагічна

57. На розтині, померлого від інтоксикації хворого в тонкій кишці знайден набряк групових лімфатичних фолікулів, виступаючих над поверхнею слизової оболонки у вигляді м'яко-еластичних бляшок з нерівною поверхнею у вигляді борозен і звивин, що нагадує поверхню мозку. Який діагноз найбільш можливий?

A. * Черевний тиф

B. Дизентерія

C. Сальмонельоз

D. Гострий ентерит

E. Холера

58. Хворий помер від інтоксикації на 4-у добу після вживання сирих яєць. На розтині слизова оболонка шлунку і тонкої кишки запалена, вкрита слизовим ексудатом; в легенях, головному мозку і печінці знайдені абсцеси. Який діагноз найбільш ймовірний?

A. * Сальмонельоз (септична форма)

B. Дизентерія

C. Сальмонельоз (черевнотифозная форма)

D. Сальмонельоз (інтестінальна форма)

E. Черевний тиф

59. У дитини з помірно вираженою загальною інтоксикацією виявлена дифтерія з поразкою крупних бронхів. Який вид запалення характерний для цього виду дифтерії?

A. * Крупозне

B. Діфтеритичне

C. Альтеративне

D. Геморагічне

E. Продуктивне

60. У дитини 10 р. З’явилися різкий біль при ковтанні, набряк шиї, температура тіла підвищилась до 390С, з’явились яскраво-червоні дрібноточкові висипання по всьому тілу. У зіві і мигдаликах – різке повнокрів’я (“палаючий зів”), “малиновий язик”. На поверхні мигдаликів – поодинокі сіруваті вогнища некрозів. Про яке захворювання може йти мова?

A. * Скарлатина

B. Менінгококовий назофарингіт

C. Дифтерія

D. Грип

E. Кір

61. При розтині чоловіка 40 р., який 10 років хворів на туберкульоз, у І і ІІ сегментах правої легені виявлена порожнина зі щільною стінкою. Внутрішня поверхня порожнини нерівна, з балками, які являють собоюоблітеровані бронхи або тромбовані судини. Середня та нижня долі щільні, на розрізі з жовтуватими вогнищами. Яку форму туберкульозу можна запідозрити у померлого?

A. * Фіброзно-кавернозний

B. Гострий кавернозний

C. Фіброзно-вогнищевий

D. Туберкулома

E. Циротичний

62..У дитини 7 років з`явилась біль у горлі. підвищилась температура тіла. На 2-ий день від початку захворювання виявлені висипання червоного кольору у вигляді дрібних, густо розташованих плям, величиною з макове зерно. Вони покривають все тіло, за виключенням носогубного трикутника. При огляді порожнини рота – в зіві яскраве почервоніння, мигдалики збільшені, язик малиново – червоний. Ваш діагноз?

A. *Скарлатина.

B. Аденовірусна інфекція.

C. Дифтерія зіва.

D. Кір.

E. Стрептококова ангіна.

63. 40-річний ув’язнений помер у виправно-трудовій колонії від туберкульозу. При автопсійному дослідженні тіла померлого встановлено деформацію і зменшення розмірів верхівок обох легень, множинні порожнини зі щільними стінками, товщиною 2-3 мм у верхніх частках обох легень, дисеміновані вогнища діаметром від 5 мм до 2 см казеозного некрозу у нижніх частках легень. Діагностуйте форму туберкульозу.

A. *Вторинний фіброзно-кавернозний

B. Вторинний фіброзно-вогнищевий

C. Гематогенний великовогнищевий з ураженням легень

D. Первинний, ріст первинного афекту

E. Вторинний циротичний

64. У 30-річного померлого наркомана, який страждав на ВІЛ-інфекцію при патоморфологічному дослідженні виявлено, що обидві легені ущільнені, темно-бордово-сірі, мало повітряні, міжальвеолярні перетинки густо інфільтровані лімфоцитами, частина альвеолоцитів трансформовані у великі клітини, з центрально розташованим круглим ядром зі світлим обідком, і які нагадують “совине око”. Яка опортуністична інфекція викликала пневмонію у даного хворого?

A. *Цитомегаловірус

B. Пневмоциста карінії

C. Атипова мікобактерія

D. Герпес-вірус

E. Токсоплазма

65. 10-річна дівчинка поступила в травматологічне відділення із симптомами патологічного перелому правої стегнової кістки. В ході гістологічного дослідження операційного матеріалу виявлено, що в кістковому мозку видаленого фрагменту стегнової кістки розташовані множинні осередки казеозного некрозу, оточені по периферії епітеліоїдними клітинами та поодинокими клітинами Пирогова-Ланхганса; зони казеозного некрозу поширюються і на прилягаючі ділянки кісткової тканини, руйнуючи її. Діагностуйте захворювання.

A. *Туберкульозний остеомієліт

B. Туберкульозний спондиліт

C. Туберкульозний коксит

D. Загальний велико-осередковий туберкульоз

E. Вторинний гострий осередковий туберкульоз

66. На розтині тіла хлопчика 8 років, що хворів на дифтерію зіва та мигдаликів і помер на другий тиждень від початку захворювання, виявлено зміни в міокарді у вигляді дрібновогнищевих некрозів міокардиоцитів, набряку строми з незначною лімфоцитарною інфільтрацією. Діагностуйте вид міокардиту.

A. *Альтеративний

B. Септичний

C. Гранулематозний

D. Інтерстиційний

E. Вогнищевий проміжний ексудативний

67. На розтині тіла чоловіка віком 46 років у прямій і сигмовидній кишці на слизовій оболонці виявили множинні коричнево-зелені нашарування, крововиливи, у просвіті кишки слиз, невелику кількість крові, гістологічно - фібринозний коліт. При бактеріологічному дослідженні вмісту кишки висіяно S. Sonnе. Діагностуйте виявлене на секції захворювання.

A. *Дизентерія

B. Холера

C. Сальмонельоз

D. Ієрсініоз

E. Хвороба Крона

68. Хворий скаржився на лихоманку, сильний головний біль, задишку, серцебиття. При огляді виявлено педікульоз, розеули і петехії на шкірі грудної клітки, пролежні гомілок, гангрену стопи. Помер при ознаках серцевої недостатності. Гістологічно в тканині довгастого мозку спостерігаються гіперемія, стази, периваскулярні муфти із плазматичних клітин і осередки проліферації мікроглії (гранульоми Попова). Ваш діагноз.

A. *Епідемічний висипний тиф

B. Сибірковий менінгоенцефаліт

C. Менінгококовий менінгіт

D. Черевний тиф

E. Краснуха

69. У померлого внаслідок серцевої недостатності на шкірі відзначаються сліди висипу у вигляді плям і крапок. В області крижи, остистих відростків хребців –пролежні. При мікроскопічному дослідженні ЦНС, шкіри, надниркових залоз в судинах мікрорциркуляторного русла і дрібних артеріях – деструктивно-проліферативний ендотромбоваскуліт з наявністю гранульом Попова, в серці – інтерстиційний міокардит. Який з перелічених діагнозів найбільш вірогідний?

A. *Висипний тиф

B. Ку-гарячка

C. Черевний тиф

D. Вузликовий періартеріїт

E. ВІЛ-інфекція

70. На розтині 17-річної дівчини, померлої внаслідок легеневої недостатності, знайдено: у ніжній долі правої легені зливні зони казеозного некрозу, в бронхопульмональних, бронхіальних та біфуркаційних лімфатичних вузлах – явища казеозного некрозу. Який з перелічених діагнозів найбільш вірогідний?

A. *Ріст первинного афекту при первинному туберкульозі

B. Гематогенна форма прогресування первинного туберкульозу

C. Гематогенний туберкульоз з переважним ураженням легень

D. Туберкульома

E. Казеозна пневмонія при вторинному туберкульозі

71. У чоловіка 50 років, що хворів на туберкульоз та помер при явищах легенево-серцевої недостатності, при розтині виявлено лобарний характер ураження легень: верхня доля правої легені збільшена, щільна, на розрізі жовтого кольору, крихкого вигляду, на плеврі – фібринозні нашарування. До якої з форм вторинного теркульозу відноситься вказана патологія?

A. *Казеозна пневмонія

B. Фіброзно-осередковий туберкульоз

C. Інфільтративний туберкульоз

D. Туберкульома

E. Гострий осередковий туберкульоз

72. У жінки 20 років при гістологічному дослідженні шийного лімфатичного вузла виявлені вузлики, які складаються з валів епітеліоїдних, лімфоїдних клітин та багатоядерних гігантських клітин Пірогова-Лангханса, розташованих між ними. У центрі вузликів визначається казеозний некроз. Який збудник потрібно виявити у зоні некрозу для встановлення остаточного діагнозу?

A. *Мікобактерії Коха

B. Мікобактерії лепри

C. Бліду трепонему

D. Рикетсії

E. Гриби

73. У хворого на слизовiй оболонці щоки виразка овальної форми з припiднятими краями з хрящеподiбним iнфiльтратом. Дно виразки м'ясисто-червоного кольору, покрите сiруватим сальним налетом. При мiкроскопiчному дослiдженнi - лiмфо-плазмоцитарний iнфiльтрат, переважно довкола дрiбних судин. В останнiх - пролiферацiя ендотелiю. Яке захворювання слід запідозрити?

A. * Сифiлiс

B. Травматична виразка

C. Ерозивно-виразкова лейкоплакiя

D. Виразка-рак

E. Виразково-некротичний стоматит Венсана

74. У хворого, який помер від туберкульозу виявлено: в І сегменті правого легеня вогнище біло-сірого кольору, оточене капсулою, діаметром 3 см. Мікроскопічно: вогнище некрозу з капсулою, відсутність пері фокального запалення. Назвіть форму туберкульозу.

A. *Туберкульома

B. Гострий кавернозний туберкульоз

C. Фіброзно-кавернозний туберкульоз

D. Циротичний туберкульоз

E. Казеозна пневмонія

75. Померлій жінці було виставлено клінічний діагноз: хронічний абсцес легенів. При розтині виявлено порожнину в 2-гому сегменті правої легені розміром 5 см, округлої форми. Внутрішня поверхня порожнини була сформована казеозними масами, зовнішня – щільною тканиною легенів, піогенна мембрана відсутня. Як на вашу думку про який процес йде мова?

A. *Гострий кавернозний туберкульоз.

B. Фіброзно-кавернозний туберкульоз.

C. Гострий абсцес.

D. Хронічний абсцес.

E. Первинна легенева каверна.

76. При мікроскопії біоптату з виразки порожнини рота виявленні осередки казеозного некрозу, оточені плазмоцитами, епітеліоїдними та лімфоїдними клітинами, зрідка гігантськими багатоядерними клітинами типу Пірогова- Лангганса, зустрічаються дрібні судини з ознаками ендо- та пері васкуліту. Вкажіть захворювання.

A. *Сифіліс

B. Туберкульоз

C. Лепра

D. Риносклерома

E. Сап

77. У хворого на рентгенограмі легень виявлено затемнення. Під час діагностичної експрес-біопсії лімфатичного вузла бронха виявлено: сирний некроз, навколо якого розташовані епітеліоїдні та лімфоїдні пласти з домішками багатоядерних гігантських клітин. Вкажіть на причину лімфаденіту.

A. *Туберкульоз

B. Пневмонія

C. Сифіліс

D. Метастази раку

E. Аденовірусна інфекція

78. При розтині трупа хлопця 19 років виявили, що легені дещо вздуті, пухкі, повнокровні, зовні та на розрізі усіяні безліччю просоподібних вогнищ сірого кольору. Гістологічно ці вогнища складались із лімфоїдних, епітеліоїдних, гігантських клітин Пірогова- Ланхганса. Про яке захворювання йде мова?

A. *Туберкульоз

B. Крупозна пневмонія

C. Бронхопневмонія

D. Інтерстиціальна пневмонія

E. Бронхоектатична хвороба

79. У ВІЛ-позитивного хворого в шкірі дистальних відділів нижніх кінцівок виявлено множинні, багряно-червоного кольору вузлики і бляшки, які зливаються. Гістологічне дослідження виявило проліферацію веретеноподібних клітин, які формують пучки і утворюють щілиноподібні структури вистелені ендотелієм. Який Ваш діагноз?

A * Саркома Капоші

B Саркома Юінга

C Фібросаркома

D Дерматофібросаркома

E Вузликовий периартериїт

80. На секції трупа виявлено воскоподібний некроз прямих м’язів живота. У термінальному відділі тонкої кишки виявлені виразки розмірами 3-5 см у діаметрі. Стінки виразок покриті крихкими сірувато-жовтими масами. Краї виразок помірно піднімаються над поверхнею слизової оболонки. Позитивна реакція Відаля. Про яке захворювання йде мова?

A *Черевний тиф з позакишковими ускладненнями

B Черевний тиф з кишковими ускладненнями

C Поворотний тиф

D Хвороба Крона

E Дизентерія

81. При судово-медичному розтині трупа чоловіка, що помер у потязі, виявлено різковиражене трупне задубіння (“поза гладіатора”). Шкіра, слизові і серозні оболонки, м’язи сухі. Між петлями тонкої кишки визначається липкий прозорий слиз. В просвіті тонкої кишки міститься велика кількість безбарвної рідини типу рисового відвару, яка взята на бактеріологічне дослідження. Який попередній діагноз найбільш імовірний?

A *Холера

B Дизентерія

C Сальмонельоз

D Черевний тиф

E Ієрсиніоз

82. Хворий 46 років, що страждав на туберкульоз протягом 6 років, помер від масивної легеневої кровотечі. При розтині: в легенях визначаються осередки склерозу і казеозного некрозу різних розмірів, у верхній частині правої легені – порожнина розміром 5,0 см в діаметрі зі щільними стінками сірого кольору, у порожнині міститься рідка кров та згустки крові. Який різновид туберкульозу?

A *Фіброзно – кавернозний

B Гострий кавернозний

C Інфільтративний

D Фіброзно - осередковий

E Гострий осередковий

83. Хворий 47 років скаржиться на утруднене дихання, заніміння кінцівок і появу вузликів м´якої консистенції на шкірі обличчя та рук. При гістологічному дослідженні біопсії вузликів виявлені лімфоїдні, плазматичні, макрофагальні клітини і клітини Вірхова. Який діагноз найбільш вірогідний?

A * Лепра

B Сап

C Сифіліс

D Риносклерома

E Туберкульоз

84. Хлопчик 7 років доставлений в лікарню з ознаками асфіксії, спричиненої тенозом зіву фібринозними плівками і набряком. Була зроблена трахеотомія. Яке ускладнення дифтерії розвинулось у дитини?

A *Справжній круп

B Несправжній круп

C Динамічна лімфатична недостатність

D Токсичний тонзиліт

E Низхідний круп

85. Дитина, яка 2 місяці назад перехворіла на дифтерію, раптово померла від серцевої недостатності. Патологоанатом виставив діагноз пізнього паралічу серця. Назвіть причину пізніх паралічів серця після перенесеної дифтерії?

A *Паренхіматозний неврит блукаючого нерва

B Токсичний міокардит

C Паренхіматозний неврит III шийного ганглію

D Тромбоз камер серця

E Постдифтерійний кардіосклероз

86. У хлопчика 5 років зявився біль при ковтанні, набряк шиї, висока температура. При огляді на мигдаликах виявлені плівки сіро-білого кольру, які знімаються з великим зусиллям. Про яке захворювання йде мова?

A *Дифтерія

B Кір

C Скарлатина

D Менінгококова інфекція

E Скарлатина

87. У помкрлого в інфекційному відділенні при аутопсії слизова оболонка прямої і сигмовидної кишки покрита масивними плівчастими коричнувато-зеленуватими нашаруваннями, тісно з´єднаними з підлеглою тканиною з дрібновогнищевими крововиливами і ділянками некрозу. При бактеріологічному дослідженні виділена шигела Зоне. Який з перерахованих діагнозів найбільш вірогідний?

A *Дизентерія

B Черевний тиф

C Сальмонельоз

D Холера

E Хвороба Крона

88, При розтині тіла померлого чоловіка віком 48 років в в ділянці 1-го сегменту правої легені виявлено круглий утвір діаметром 5 см з чіткими контурами, оточений тонким прошарком сполучної тканини, виповнений білими крихкими масами. Діагностуйте форму вторинного туберкульозу.

A. *Туберкулома

B. Казеозна пневмонія

C. Гострий кавернозний туберкульоз

D. Гострий вогнищевий туберкульоз

E. Фіброзно-кавернозний туберкульоз

Професійні хвороби.

1. Мікроскопійно у видаленому сегменті легені шахтаря знайдеені багаточисельні, округлі вузлики, побудовані з концентричних гіалінізованих пучків сполучної тканини. Більша частина їх розташована периваскулярно та перибронхіально. Найбільш вірогідний діагноз:

A. *сілікоз

B. туберкульоз

C. бронхіт

D. фіброзуючий альвеоліт

E. рак легень

2. При розтині тіла шахтаря та гістологічному дослідженні у легенях було знайдено численні тонкі тяжі сполучної тканини з осередками склерозу навколо бронхів та судин легень; розростання сполучної тканини у альвеолярних перегородках з розвитком бронхіоліту та бронхоектазів. У біфуркаційних лімфовузлах – поодинокі лімфовузли із склерозом їх тканини та великою кількістю частинок пилу та коніофагів. Яка форма силікозу має місце?

A. *Дифузно-склеротична

B. Вузликова

C. Змішана

D. Силікотична кавернозна

E. Силікотуберкульоз

3. На розтині хворого, який багато років працював на шахті і помер від хронічної легенево-серцевої недостатності, виявлено, що легені малоповітряні, значно ущільнені, склерозовані, верхівки емфізематозно змінені, поверхня сіро-чорного кольору, на розрізі тканина легенів аспідно-чорного кольору. Від якої хвороби настала смерть?

A. *Антракоз

B. Силікоз

C. Талькоз

D. Асбесто з

E. Алюміноз

4. При розтині чоловіка, який служив на підводному атомному човні виявили наступні патології: спустошення в кістковому мозку (панмієлофтиз), анемію, лейкопенію, тромбоцитопенію, розпад лімфоцитів у лімфатичних вузлах, селезінці, лімфатичному апараті шлунково-кишкового тракту, крововиливи в наднирники. Яка хвороба розвинулась у даному випадку?

A. *Гостра променева хвороба

B. Кесонна хвороба

C. Гострий лейкоз

D. Гостра анемія

E. Вібраційна хвороба

5. На розтині чоловіка, що багато років працював на підприємстві з високим рівнем вільного двоокису кремнію у повітрі, знайдені збільшені та ущільнені в об(ємі легені зі значною кількістю міліарних і більш великих осередків склерозу округлої та овальної форми сірого або сіро-чорного кольору. Який з перелічених діагнозів найбільш вірогідний?

A. *Вузловата форма силікозу

B. Дифузно-склеротична форма силікозу

C. Антракосилікоз

D. Силікоантракоз

E. Азбестоз

6. У померлого, що понад 20 років працював на шахті зі здобуття кам(яного вугілля, при розтині знайдені ущільнені легені сіро-чорного кольору зі значними ділянками новоутвореної сполучної тканини. та наявністю великої кількості макрофагів з пігментом чорного кольору у цитоплазмі. Який з перелічених діагнозів найбільш вірогідний?

A. *Антракоз

B. Антракосилікоз

C. Силікоантракоз

D. Талькоз

E. Сидероз

Хвороби кістково-м’язовової системи

1. У хворого при рентгенологічному обстеженні в плоских кістках виявлені множинні вогнища остеопорозу і остеолізісу. У трепанобіоптаті виявлено високий зміст пухлинних плазматичних клітин. Ваш діагноз.

A. *Мієломна хвороба

B. Гострий моноцитарний лейкоз

C. Хронічний мієлолейкоз

D. Лімфогрануломатоз

E. Гістіоцитоз

2. При дослідженні стегнової кістки виявлено хронічне гнійне запалення компактної речовини та кісткового мозку, утворення секвестрів. При якому захворюванні розвиваються такі зміни?

A. *Остеомієліт

B. Ретикулосаркома

C. Мієломна хвороба

D. Остеобластокластома

E. Периостит

3. У чоловіка 72 років, на секції виявлено збільшення та деформація правого колінного суглобу. При гістологічному дослідженні тканин суглобу та прилягаючих ділянок знайдено: масивні вогнища казеозного некрозу, які оточені валоми епітеліоїдних, лимфоідних клітин з наявністю гігантських макрофагів. Діагностуйте захворювання.

A. *Туберкульозний артрит

B. Сифілітичний артрит

C. Ревматоїдний артрит

D. Гонорейний артрит

E. Деформуючий остеоартроз

4. У хворої дівчинки 12 років мають місце птоз, швидка стомлюваність мовних м’язів і м’язів стегна. Біопсія останніх виявила ділянки дистрофії, атрофії та некрозу поперечно-смугастої мускулатури з осередковим накопиченням лімфоцитів. В тимусі знайдено тимому. Який з перелічених діагнозів найбільш вірогідний?

A. *Міастенія.

B. Міозит.

C. Прогресуюча м’язова дистрофія.

D. Боковий аміотрофічний склероз.

E. Дерматоміозит

5. У померлого від інтоксикації чоловіка під час розтину виявлено збільшення стегна в об?ємі, на поверхні шкіри - свищі з виділенням з них в'язкої жовто-зеленої рідини; стегнова кістка потовщена та деформована, має місце утворення секвестрів; при мікроскопічному дослідженні секвестральні порожнини оточені грануляційною та сполучною тканиною з наявністю нейтрофілів, кістково-мозкові канали облітеровані, компактна пластинка потовщена. Який з перелічених діагнозів найбільш вірогідний?

A. *Хронічний гнійний остеомієліт

B. Гострий гематогенний остеомієліт

C. Туберкульозний остеомієліт

D. Паратиреоїдна остеодистрофія

E. Остеопетроз

6. На розтині померлого від сепсису в стегновій кістці нижньої кінцівки виявлено флегмонозне запалення, що охоплює кістковий мозок, гаверсові канали та періост, під періостом – множинні абсцеси, в навколишніх м'яких тканинах стегна – також флегмонозне запалення. Вкажіть, який патологічний процес має місце?

A. *Гострий гематогенний остеомієліт

B. Остеопороз

C. Хронічний гематогенний остеомієліт

D. Остеопетроз

7. При розтині чоловіка 60 р. виявлено потовщення кісток склепіння черепа, тіл хребців, множинні спіралеподібні викривлення кісток передпліч, гомілок. Мікроскопічно тканина уражених кісток мозаїчна: представлена фрагментами лакунарного розсмоктування та новоутворення кісткової тканини, з’єднаними базофільними лініями „склеювання”. Діагноз?

A * Хвороба Педжета

B Саркома Юінга

C Мієломна хвороба

D Фіброзна дисплазія

E Хронічний остеомієліт

8. Хвора 50-ти років, за короткий період часу (6 міс.) тричі лікувалась з приводу переломів отриманих побутово. При мікроскопічному дослідженні кісткової тканини – осередки лакунарного розсмоктування, гігантоклітинні гранульоми в пухлино подібних утворах, кісти, кісткова тканина заміщується фіброзною сполучною. Лабораторно – гіперкальційемія. Ваш діагноз?

A *Паратиреоїдна остеодистрофія

B Мієломна хвороба

C Остеомієліт

D Остеопетроз

E Хвороба Педжета

9. При патологоанатомічному розтині тіла жінки 45 років в хребті, кістках черепа та ребрах знайдені дефекти кісткової тканини, по краях яких багато пухлинних вузлів. Проведена мікроскопія пухлинних утворень в кістковому мозку та плоских

кістках виявила проліферацію в основному клітин плазмоцитарного ряду. Діагноз?

A * Мієломна хвороба

B Метастаз раку легень в кістки

C Остеосаркома

D Остеомієліт

E Саркома Юінга

10. У хворого 60 років з спонтанним переломом ребра рентгенологічно відмічені вогнища множинного двобічного остеопорозу ребер. В біоптаті із зони перелому виявлено однорідну проліферацію клітин лімфоцитарно-плазмоцитарного типу з

ознаками клітинного атипізму. Назвіть захворювання:

A * Мієломна хвороба

B Остеосаркома

C Туберкульозний остеомієліт

D Гострий остеомієліт

E Метастаз раку легень

Соседние файлы в предмете [НЕСОРТИРОВАННОЕ]